122
Pool of Argument Topics Argument No. 1 Woven baskets characterized by a particular distinctive pattern have previously been found only in the immediate vicinity of the prehistoric village of Palea and therefore were believed to have been made only by the Palean people. Recently, however, archaeologists discovered such a "Palean" basket in Lithos, an ancient village across the Brim River from Palea. The Brim River is very deep and broad, and so the ancient Paleans could have crossed it only by boat, and no Palean boats have been found. Thus it follows that the so-called Palean baskets were not uniquely Palean. Write a response in which you discuss what specific evidence is needed to evaluate the argument and explain how the evidence would weaken or strengthen the argument. Based on the very fact, that the Brim River is broad and deep, the author insists, that only by boat can one across the river. As no boat has been found in Palean so far, therefore, the only plausible explanation of a unique Palean basket being discovered in Lithos has to be that the basket belongs to Lithos. In other words, it was, from the author’s perspective, impossible to carry a Palean basket to Lithos without boat. The first hidden assumption made by the author is the river never dry up, or not being frozen at any time, or the water lever has been remained the same as always, under all of which circumstances human beings are not able to walk across the river. So we need more evidence to show whether the Brim River changes to be possible to cross by walking. If any document or other geological materials indicate that the Brim River had once been dried up, then it will certainly weaken the author’s claim. The second hidden assumption is the author assumes the Brim River had been shaped before the particular basket came to being. We have to, therefore, identify both of the forming time of the Brim River and the Palean basket. If the Brim River formed before the basket, it definitely strengthens the author’s claim. Otherwise, such basket being carried to Lithos without boat is consequently reasonable. Another crucial assumption, according to the author’s argument, is Palean people and only Palean people could ever carry the basket to Lithos. From the fact that no boat was found in Palean, the author derived an inference that such basket was impossible to be carried to the rival village. However, we need to see whether there’s a boat as old as the basket in Lithos. If the answer is yes, then the

Y9 Argument Topics

  • Upload
    aakash

  • View
    223

  • Download
    1

Embed Size (px)

DESCRIPTION

Gre arguments examples

Citation preview

Page 1: Y9 Argument Topics

Pool of Argument Topics

Argument No. 1

Woven baskets characterized by a particular distinctive pattern have previously been found only in the immediate vicinity of the prehistoric village of Palea and therefore were believed to have been made only by the Palean people. Recently, however, archaeologists discovered such a "Palean" basket in Lithos, an ancient village across the Brim River from Palea. The Brim River is very deep and broad, and so the ancient Paleans could have crossed it only by boat, and no Palean boats have been found. Thus it follows that the so-called Palean baskets were not uniquely Palean.

Write a response in which you discuss what specific evidence is needed to evaluate the argument and explain how the evidence would weaken or strengthen the argument.

Based on the very fact, that the Brim River is broad and deep, the author insists, that only by boat can one across the river. As no boat has been found in Palean so far, therefore, the only plausible explanation of a unique Palean basket being discovered in Lithos has to be that the basket belongs to Lithos. In other words, it was, from the author’s perspective, impossible to carry a Palean basket to Lithos without boat.

The first hidden assumption made by the author is the river never dry up, or not being frozen at any time, or the water lever has been remained the same as always, under all of which circumstances human beings are not able to walk across the river. So we need more evidence to show whether the Brim River changes to be possible to cross by walking. If any document or other geological materials indicate that the Brim River had once been dried up, then it will certainly weaken the author’s claim.

The second hidden assumption is the author assumes the Brim River had been shaped before the particular basket came to being. We have to, therefore, identify both of the forming time of the Brim River and the Palean basket. If the Brim River formed before the basket, it definitely strengthens the author’s claim. Otherwise, such basket being carried to Lithos without boat is consequently reasonable.

Another crucial assumption, according to the author’s argument, is Palean people and only Palean people could ever carry the basket to Lithos. From the fact that no boat was found in Palean, the author derived an inference that such basket was impossible to be carried to the rival village. However, we need to see whether there’s a boat as old as the basket in Lithos. If the answer is yes, then the basket was probably carried by Lithos people when they sailed to Palean for trade, and accidentally brought the distinctive basket back to Lithos. In that case, the claim raised by the author was weakened.

Lastly, the author preconceived that the basket must have been brought by human beings. However, natural force also could be involved in the translocation of the basket. One single basket found in Lithos was hardly able to exclude the non-human factors as a cause of that. It’s perhaps the waves pushing it to the bank near Lithos. Thus, given that the more Palean baskets were discovered in Lithos, the more likely the basket belongs to Lithos. Of course it should be concluded when the above three assumptions turn out to be true.

Page 2: Y9 Argument Topics

Argument No. 2

“Recent study of eighteen rhesus monkeys provides clues as to the effects of birth order on an individual's levels of stimulation. The study showed that in stimulating situations (such as an encounter with an unfamiliar monkey), firstborn infant monkeys produce upto twice as much of the hormone cortisol, which primes the body for increased activity levels, as do their younger siblings. Firstborn humans also produce relatively high levels of cortisol in stimulating situations (such as the return of a parent after an absence). The study also found that during pregnancy, first-time mother monkeys had higher levels of cortisol than did those who had had several offspring.”

The author of the letter introduction a investigate result which perhaps reveal impact of birth order on an individual's levels of stimulation. To support the claim, the author cites three examples about the firstborn infant, firstborn humans and first-time mother monkey sequentially to prove that a positive relationship may be exists with birth order and the level of the hormone cortisol. Close scrutiny of each of the evidence, however, none of them is credible.

First at all, as for the first example, producing twice as much of the hormone cortisol doesn't indicate that there is a relation with birth order and the level of hormone, perhaps it may be caused by other factors such as gender: as I know, monkeys of different gender will have different level of hormone even if they share the same circumstance. According to it, there is a flaw in the research that it doesn't remind gender and so that they can't rule out a following situation that all the first infant monkeys in experiment are female who will actually secrete more hormones. Even if they are not first infant , they can also produce the highest level of hormones. In short, the factors that have impact on the level of stimulation may be monkey's gender rather than birth order.

Second, about the firstborn humans, the experiment is also not credible. It's entirely possible that the firstborn infant doesn't have playmates at home, so parents are their only friends. After a period time doesn't see their parents, they will be so excited and then produce more hormones when parents come home. On the contrast, as for the later born infant, they use to play with their sisters or brothers, so maybe they don't mind whether their parents are at home or not.

Third, about the last experiment, the author assume unfairly that first-time mother monkeys who have higher levels of hormone will deliver an infant monkey that also have relatively more hormone. However, the author use no evidence to support the assumption. Lacking such evidence it is equally possible that the hormone level of mothers don't influence the offspring's. Thus, the author's claim is not credible.

In sum, all these three examples can't attest the relationship with birth order and the level of hormone of cortisol. The results from the experiments may be influent by other factors, such as gender. So to strength the claim, the author should rule out all other possible factors.

Page 3: Y9 Argument Topics

Argument No.3

"Over the past two years, the number of shoppers in Central Plaza has been steadily decreasing while the popularity of skateboarding has increased dramatically. Many Central Plaza store owners believe that the decrease in their business is due to the number of skateboard users in the plaza. There has also been a dramatic increase in the amount of litter and vandalism throughout the plaza. Thus, we recommend that the city prohibit skateboarding in Central Plaza. If skateboarding is prohibited here, we predict that business in Central Plaza will return to its previously high levels."

Write a response in which you discuss what questions would need to be answered in order to decide whether the recommendation is likely to have the predicted result. Be sure to explain how the answers to these questions would help to evaluate the recommendation.

Witnessing the number of shoppers in Central Plaza decrease simultaneously with the increase of the popularity of skateboarding, the author comes to the recommendation that skateboarding should be prohibited so as to refresh the business in Central Plaza. To substantiate the conclusion, the arguer also provides evidence concerning the increasing amount of litter and vandalism throughout the plaza. However, this alone neither constitutes a logical argument in favor of the conclusion nor provides compelling support making the argument sound. The prohibition seems at first glance reasonable, but a careful examination would reveal the Achilles' heel of the article, which can be listed as follow.

First and for most, Why it is skateboard users that keeps customers from shopping the Central Plaza? The author just lists two facts, but provides no interpretation reasoning the link between them. It is ridiculous to assume that it is due to skateboarding that more litter and vandalism is found in the Plaza. Are there any evidence showing that it is skateboard users that pollute the environment and damage the facilities? Perhaps it has something to do with the special activities or exhibitions hold in the plaza.

What's more, the author only provides information concerning the opinion of these shoppers who blame the poor business to these skateboard users without taking some interior factors into account. I doubt if the price of these products increase drastically during the past two years. Price is the major concerning for customers, even nuance may result in the great in behavior. Is service as good as they used to? Do they hold activity or advertise as they used to? As their income cut, it is not likely to be the case.

Last but not least, the author also fails to offer data concerning the outside atmosphere. Are there any new plazas nearby? Perhaps a large shopping mall opened two years ago providing all kinds of product with high cost effective. What if the number of habitants living nearby falls as more people enjoy living in suburban area away from air pollution? It is hasty to arrive to the conclusion without taking these exterior factors into account.

In summary, though the conclusion seems to be plausible, in fact, it lacks credibility because the evidence cited in the analysis does not lend strong support to what the arguer maintains. The author fails to establish a causal relationship between falls in sales in Central Plaza with the

Page 4: Y9 Argument Topics

increasing popularity of skateboarding. To make it logically acceptable, the author would have to provide more evidence concerning the interior and exterior factors as well as overall survey result about attitudes towards the problem.

Argument No.3

In this letter, the writer suggested that business stagnation is correlated to skateboarding in Central plaza. Vandalism and poor social behavior of users are also underlined as exaggerating the problem.

In this claim, one main obscurity is other concomitant conditions that may have influenced the popularity of Central Plaza as a shopping center. For example if a new shopping mall has been recently opened nearby, it is highly plausible that a portion of previous shoppers have been attracted there. Business is a dynamic competition and the number of customers directly depends on how pleasant the goods or services be presented to them. A new shopping mall with a relatively more friendly atmosphere than Central Plaza which address the needs of consumers more efficiently can be a strong rival for it. Easy Access and having a big parking lot also influence the inclination of shoppers to the new center. Losing a considerable number of customers, it is not surprising if skateboarders prefer to play around Central Plaza as it is now less crowded than before.

Another situation might be a change in general attitude toward special merchants. It should be cleared that which type of goods or services are provided in this shopping center. Business is a competitive field and should be regulated according to the needs and requirements of general population. It should be defined that which part of general needs are responded in Central Plaza in order to rule out the unpopularity of its provisions and bolstering the role of skateboarders.

Moreover, we must know the overall condition of national economy. It is obvious that in recession periods people tend to spend their money on their basic requirements and consequently, business may undergo a period of stagnation. Few numbers of shoppers will appear not in Central Plaza but also in all other shopping centers of the country.

However, until not having a query from the customer to know why they do not like to do their shopping at Central Plaza, any argument around this problem will be faulty. Customers are the pivotal factor in economical activities and their comments should be considered the most valuable information in this regard.

Page 5: Y9 Argument Topics

Argument No.4

PROMPT:” Of the two leading real estate firms in our town—Adams Realty and Fitch Realty—Adams Realty is clearly superior. Adams has 40 real estate agents; in contrast, Fitch has 25, many of whom work only part-time. Moreover, Adams’ revenue last year was twice as high as that of Fitch and included home sales that averaged $168,000, compared to Fitch’s $144,000. Homes listed with Adams sell faster as well: ten years ago I listed my home with Fitch, and it took more than four months to sell; last year, when I sold another home, I listed it with Adams, and it took only one month. Thus, if you want to sell your home quickly and at a good price, you should use Adams Realty.”

Write a response in which you examine the stated and/or unstated assumptions of the argument. Be sure to explain how the argument depends on these assumptions and what the implications are for the argument if the assumptions prove unwarranted.

The author concludes that Adams Realty will sell a home quickly and at a good price as it is superior of the two leading real estate firms. However, the author’s conclusion is based upon some unwarranted assumptions.

The author states that while Adams has 40 real estate agents, Fitch has only 25, many of whom work only part-time. With this statement the author assumes that having a larger workforce guaranties better work. Although, the assumption maybe correct, still the author fails to point out to the number of real estate agents that work round the clock in Adams Realty. If, for instance, out of 25 agents of Fitch, 20 work full time, and out of Adams’ 40 workers, 15 work full time, then the figure may weaken author’s argument.

Further, the author continues to state that Adams’ revenue last year was twice as high as that of Fitch and included home sales that averaged $168,000, compared to Fitch’s $144,000. With this statement, the author assumes that since revenue and average home sales are greater for Adams, hence Adams is a more established firm then Fitch. However, generating more revenue in total does not prove so. Adams may have sold 40 houses and earned $400,000 ($10,000 average profit per sale), while Fitch may have sold only 10 houses and still earned $200,000 ($20,000 average profit per sale). Although the revenue earned by Adams is more, still Fitch Realty is a more efficient firm according to the assumed statistics. Furthermore, Adams’ average revenue may be more then Fitch’s but since the author fails to establish that the locality where the house was sold was the same, this may not prove a valid argument for author’s assumption. For instance, if the average home sales of Adams is $168,000 in a very good neighbourhood where the profit margin that Adams gain is $168 (10%), while Fitch sells a house in a comparatively poor neighbourhood and earn a profit of $288 (20%), still Fitch would be a better

Page 6: Y9 Argument Topics

real estate firm. Even if both the figures are combined, then too the author fails to establish that Adams will will fetch a better price for a home sold.

Additionally, the author states that homes listed with Adams sell faster. This is based on author’s own experience wherein it took 4 months for Fitch to sell the house 10 years ago, while Adams sold in one month this year. Herein, the author fails to prove that the conditions were similar 10 years ago while selling the house. If 10 years ago, the demand for house was not as much as now, and the economy of the place was worse then today’s, then comparing the time periods of the two sales is not a parameter to decide the superiority of the firms. Also, 10 years ago, Fitch Realty may not be as good firm as it is now. Hence, basing the argument on an event that took 10 years ago, and concluding an assumption in present will not support author’s conclusion. In conclusion, although the author has stated facts and figures in support of his argument, still, they are insufficient to conclude the author’s assumption that Adams Realty will sell a house quickly and will fetch at a good price for it.

In the letter, the homeowner argues that a estate firm---Adams, is superior to Fitch, which is another leading firm in his town. To bolster it, the homeowner cites various evidence to support it. However, none of these is convincing because the claims relies on a series of unsubstantiated assumptions which render it unconvincing on it stands.

First, the argument unfairly rests on the assumption that the agents from these two firms have equal professional skills, thus, the author argues that more agents indicates greater efficiency. However, absent evidences to support it. So, it is as likely that agents in Fitch have much better abilities to deal with matters like promoting the sales of houses though the number of agents in there is relatively less than Adams'. Besides, many of the agents in Fitch work only part-job indicates that they have wider circle of people in other fields. Thus, they will gain more clients when they work as agents, which is a beneficial factor for the development and economical efficiency of the firm.

Second, the homeowner also unfairly assumes that these two estate firms sold the same amount of houses last year. According to it, he claim that higher sale price equate to much more income. However, the letter contain no convincing evidence to support it. Lacking the evidences that this is the case, it is entirely possible that in last years, Adams sold 5 houses, for each of them is $168000, whereas Fitch sold 15 houses, for each of them is $144000. In this case, it's clear to see that Fitch contains more economic benefits compared with Adams. Thence, the argument of the homeowner that Adams is superior to Fitch will be unconvincing.

Thirdly, the homeowner assumes that the conditions of estate market of recent are similar to ten years ago. So, the author judges Adams is better for it took relatively shorter time to sale a houses last year while ten years ago, Fitch spent 4 month to sale one. However, the assumption is unsound regarding of everything is changing. Beside, no convincing evidences in the letter are used to support the assumption. Due to it, it is as likely that with the improvement of people's lives, the richer become more and it's just a cake for them to buy a house while ten year ago, a large part of people could not afford it. Moreover, rather than buying houses for living, they prefer to invest on them so as to earn more money.

In sum, the argument is unpersuasive. The author need to offer more evidences to substantiate these following assumptions:1)the agents in the two firms have the same professional abilities.2) The two firm sold the same amount of houses last year.3)the conditions of estate market are constant. Without these evidence, the homeowner cannot convince me.

Page 7: Y9 Argument Topics

Argument No.5

"The population of Balmer Island increases to 100,000 during the summer months. To reduce the number of accidents involving mopeds and pedestrians, the town council of Balmer Island should limit the number of mopeds rented by each of the island's six moped and bicycle rental companies from 50 per day to 30 per day during the summer season. By limiting the number of rentals, the town council is sure to attain the 50 percent reduction in moped accidents that was achieved last year in the neighboring island of Torseau, when Torseau's town council enforced similar limits on moped rentals."

The writer of the letter argues that limiting the number of mopeds that can be rented will lead to reducing the number of accidents in the Balmer Island in the summer months. The arguer cites the example of the neighboring island of Torseau as evidence in support of his claim. There has been a reduction in the number of accidents in Torseau after similar limits were enforced by the authorities in Torseau. The evidence provided by the arguer fails to establish a link between the occurrence of accidents and limiting the number of mopeds that can be rented out.

The arguer has based his contention on the reduced number of accidents in Torseau. However, the arguer fails to provide substantial evidence that can link the reduction of accidents to the limitation imposed on the moped rentals in Torseau. It is likely that the authorities in Torseau have clubbed some other activities with the limitation on moped rentals. These activities could be conducting classes for the tourists on road safety, enforcing stringent traffic rules, separating lanes for the bicycles and mopeds, ensuring that the footpath for the pedestrians is well away from the roads, etc. Such measures could have resulted in the reduction of accidents in Torseau. Additionally, it is likely that the influx of tourists in both these islands is not the same. There is a possibility that the number of tourists visiting Balmer is more than those visiting Torseau. Therefore, since the tourists do not have an idea of the roads and directions to their destinations, they tend to get careless on the roads leading to a higher number of accidents in Balmer. Furthermore, it is likely that Torseau saw a lesser number of tourists last year leading to a lesser number of accidents. Therefore, it is unlikely that the limitation on the moped rentals had a major influence on the reduction of accidents of pedestrians with mopeds in Torseau.

Even if one assumes that the restrictions on moped rentals has indeed led to a reduction in the number of accidents in Torseau, it cannot be believed that implementing similar restrictions will have the same effects in both the islands. This is because the arguer has not provided any concrete evidence in support of the similarities between these two islands. There is no indication of the geographical structure of the two islands. It is likely that Balmer has uneven or hilly terrain and the downhill drive is the major cause for moped accidents. Hence, it is unlikely that implementing restrictions on moped rentals will have the desired effect in Balmer as the islands of Balmer and Torseau cannot be compared in light of the inadequate evidence provided by the arguer.

The arguer fails to address the issue of how the tourists will travel if they are denied access to bicycles and mopeds. It is obvious that a reduction in the number of mopeds that can be rented out will increase the number of pedestrians leading to a greater risk of accidents on roads involving pedestrians. Moreover, imposing a restriction on moped rentals may lead to each of the moped rental companies diversifying into two or more companies and the overall number of mopeds will remain the same and the restriction will be rendered ineffective.

Page 8: Y9 Argument Topics

In view of the above, it can be concluded that the argument has failed to convince the reader. The arguer could have substantiated his claim by proving similarities between Balmer and Torseau and convincing the reader that the implementation of similar laws would have the same effects in both the islands. Therefore, in its present form, the given argument is flawed and unconvincing due to lack of adequate evidence in support of its claim.

Argument 5

The author's recommendation that by limiting the moped's rented by rental companies on the Balmer islands from 50 to 25 per day, the accident rate will decrease by 50 percent in summer months, may seem logical at first glance. The author's assumptions might be correct if the premises on which they are based on, were to be true. However, the author has made certain assumptions in reaching his conclusions. Some important questions arise from these assumptions, which need to be answered clearly, if the recommendation is to give the predicted result.

The author has stated that the population of Balmer island increases in summer months. Since mopeds are the main form of transport, there are more moped accidents in summer months. The author has propsed that a reduction in the number of moped rentals from 50 to 25 per day in summer months will bring down the accident rate by 50 percent. His conclusion is based on the fact that a similar reduction in moped accidents was achieved last year in the neighbouring island of Seaville, by applying the same control measure. However, the author has not answered some pertinent questions while proposing the solution. First question that arises is, does the island of Seaville has the same population increase in the summer months and like Balmer island, does the island of Seaville also has mopeds as the popular form of transport? The author has stated that by limiting moped rentals a reduction in moped accidents was achieved on island of Seaville during summer months. However, if the demographic statistics of Balmer and Seaville were to differ significantly in summer months, the same effect might not be achieved in Balmer. Suppose, as opposed to Balmer, large number of people from Seaville might be going to mainland for summers. In this case, the number of moped accidents will automatically come down as there will be less number of drivers and pedestrians. The author has failed to state, whether the moped is the popular form of transport in Seaville or not. In Seaville, due to geographical or other reasons, some other means of transport like dirt bikes, cars etc may have gained popularity over the last year. So a reduction in number of moped users might have brought down the accident rate and the reduction of rentals may not have played such a significant role in the same.

The author has not given any information regarding the number of rented mopeds involved in accidents on the Balmer island. Since, moped is the popular form of transport on Balmer, it is natural to assume that many residents of the island may be owning mopeds. The author has given a figure that the population of Balmer becomes 100,00 in summers, but he hasn't stated the percentage increase in population. So if we are to assume that most of the visitors rent mopeds for transport, there is no evidence to suggest that there is a large percentage of increase in moped rentals during summers. The author has not given any particular evidence to connect the accidents with rented mopeds. So, if a majority of population owns mopeds in Balmer, a decrease in rentals will not have much effect on number of accidents.

Page 9: Y9 Argument Topics

Summing up, the author may be right in claiming that, "By limiting the number of rentals, the town council will attain the 50 percent annual reduction in moped accidents. "However based on this article, his conclusion depends on some unstated assumptions. Some pertinent questions arise due to these assumptions, which have not been answered in this article. Unless the author explicitly answers the questions pointed out in the above paragraphs, his recommendations will not have the predicted effects.

Argument no. 6

"Arctic deer live on islands in Canada's arctic regions. They search for food by moving over ice from island to island during the course of the year. Their habitat is limited to areas warm enough to sustain the plants on which they feed and cold enough, at least some of the year, for the ice to cover the sea separating the islands, allowing the deer to travel over it. Unfortunately, according to reports from local hunters, the deer populations are declining. Since these reports coincide with recent global warming trends that have caused the sea ice to melt, we can conclude that the purported decline in deer populations is the result of the deer's being unable to follow their age-old migration patterns across the frozen sea."

Instructions: "Write a response in which you discuss what specific evidence is needed to evaluate the argument and explain how the evidence would weaken or strengthen the argument."

In the above article the writer talks about the depleting number of deer that migrate across the frozen sea. He has provided some report from the local hunters, observed the global warming to be the reason for the melting sea ice, and concludes that the reason for this decline in number is due to the incapability of deer to migrate the way they used to previously. The author seems to overlook some of the points that can challenge his assertions.

Firstly, the writer makes his assumptions on the basis of some report from the local hunters. The conjecture made here seems vague as there are no proper statistics the writer refers to. Instead of a general mention of the reports, the argument could have been supported with a proper survey presenting the actual number of arctic Deer that were found each year. Only on the basis of the data collected, can an assertion be made about the depleting number of the deer. The writer also mentions that this report has been collected from the local hunters. These local hunters can only give a vague count of the animals. These observations given by the hunters will be strictly restricted to the population of the deer observed while hunting. Hence, this data cannot be fully dependable.

In the second point, the writer allures to be an assumed relation established between the recent global warming trends and the sea ice melting, concluding that decline in deer populations is the result of the deer's being unable to follow their age-old migration patterns across the frozen sea. Here, the point is agreeable that the global warming has definitely increased over the past few years, but it cannot be assumed that the rate of global warming has been equivalent to that required to melt the ice sheets. The glaciers are melting in the Arctic but the writer clearing mentions that these deer dwell in Canada. The effect of global warning cannot be the same for every place.

Another point worth considering here is that the decline of the deer can have other valid reasons too. The writer himself mentions the presence of hunters in these areas. One cannot deny the possibility of the deer being hunted in large numbers. There may be other reasons as well. It has

Page 10: Y9 Argument Topics

been mentioned that suitable temperatures are crucial for these deer to sustain. The global warming has caused erratic changes in the weather too. The cold temperatures in Canada could have risen so as to prevent the growth of plants on which these deer feed eventually leading to the death due to lack of nutrition. Therefore, although the rising temperatures may be one of the reasons for the melting of ice and eventually leading to attenuate the migration rate for these deer to some extent, there are other potent reasons too that might exist on proper investigation for the declining population of these deer.

It is evident that the writer fails to convince the readers with his assertions which seem to be purely based on assumptions and generalities.

Argument No.7

"We recommend that Monarch Books open a café in its store. Monarch, having been in business at the same location for more than twenty years, has a large customer base because it is known for its wide selection of books on all subjects. Clearly, opening the café would attract more customers. Space could be made for the café by discontinuing the children's book section, which will probably become less popular given that the most recent national census indicated a significant decline in the percentage of the population under age ten. Opening a café will allow Monarch to attract more customers and better compete with Regal Books, which recently opened its own café."

Write a response in which you discuss what questions would need to be answered in order to decide whether the recommendation is likely to have the predicted result. Be sure to explain how the answers to these questions would help to evaluate the recommendation.

The author recommends that oopening a cafe will in Monarch book store will help in attracting customers and also suggests to discontinue the children's section and open cafe in that space. To support this, the author cites the fact that there is decrease in percentage of population under age 10 in recent national census.

To begin with, the author assumes that discontinuing the children's section and opening a cafe in that space would attract customers. This may not necessarily be the case. The number of customers may decrease by few numbers as the customers who come to the store only to get the childrens' books may not show up after removing this section.

The author suggests to remove the children's section assuming that it would beome less popular and provides a census report. The report states that there is decrease in popuplation under age 10 nation wide and not in the particular place where the Monarch Books is located. So there might be very small decrease in the place where the shop is located. So removing children's section may result in losing regular customers.

Additionally, even if we assume that there is a decrease in population, there is no information regarding the customers who get books from children's section. We cannot tell for sure that only children below 10 years of age use the children's section. Parents or primary school teachers may use the section aswell. So removing the children's section may result in Monarch losing its customers to its competitors.

The author also assumes that opening a cafe will attract more customers. Author has also stated that this will help Monarch to better compete with its competitor Regal Books which has opened

Page 11: Y9 Argument Topics

a cafe recently. The author has not mentioned if there was increase in number of customers in Regal after opening a cafe and if that increase is only due to opening a cafe and not due to any other reason.

Hence the author provides no evidence for the assumptions made and hence the arguement is not convincing enough.

Argument No.8

The following appeared in a memo from the director of student housing at Buckingham College.

"To serve the housing needs of our students, Buckingham College should build a number of new dormitories. Buckingham's enrollment is growing and, based on current trends, will double over the next 50 years, thus making existing dormitory space inadequate. Moreover, the average rent for an apartment in our town has risen in recent years. Consequently, students will find it increasingly difficult to afford off-campus housing. Finally, attractive new dormitories would make prospective students more likely to enroll at Buckingham."

Write a response in which you discuss what specific evidence is needed to evaluate the argument and explain how the evidence would weaken or strengthen the argument.

The Director has stated that a number of new dormitories has to be built as enrollment is growing at Buckingham college and it will be double over the next 50 years. The Director could have given a report of increase in number of students enrolling each year and the total strength of college students that the dormitory accommodates. Even if the college strength doubles, it may not be necessary that all those students would be staying in college dormitories. Students staying with their parents would be coming from their houses. So if the Director had provided a report of how many students out of total college strength stay in college dormitory, it would have helped strengthen the argument.

Secondly, the director has mentioned that the rent of apartments in town has increased recently and that the students would find it difficult to afford to stay there. But director has provided no figures regarding how much the rent has increased and how much a student pays per year for staying in college dormitories. If there is only a little difference, two or three students can stay together in an apartment and share the rent. Hence this evidence lacks, which would have again helped in strengthening the argument.

Additionally, Director has mentioned that attractive new dormitories would make prospective students enroll at this college. To strengthen this statement, director could have approached the new students individually and prepared a survey of how many students enroll in a college based on how big and confortable the dormitory is.

Finally, providing a survey of how many students are currently stating in the college dormitories and how many new students can be accommodated in the following year could have illustrated the need for building new dormitories. Hence lack of all these evidences weakens the argument and does not make it look like a serious issue to be considered immediately.

Argument No.9

Page 12: Y9 Argument Topics

"Nature's Way, a chain of stores selling health food and other health-related products, is opening its next franchise in the town of Plainsville. The store should prove to be very successful: Nature's Way franchises tend to be most profitable in areas where residents lead healthy lives, and clearly Plainsville is such an area. Plainsville merchants report that sales of running shoes and exercise clothing are at all-time highs. The local health club has more members than ever, and the weight training and aerobics classes are always full. Finally, Plainsville's schoolchildren represent a new generation of potential customers: these schoolchildren are required to participate in a fitness-for-life program, which emphasizes the benefits of regular exercise at an early age.”

Write a response in which you examine the stated and/or unstated assumptions of the argument. Be sure to explain how the argument depends on these assumptions and what the implications are for the argument if the assumptions prove unwarranted."

The author concludes that Nature’s way will be successful when it opens its store in Plainsville. He bases his conclusion on the premise that Nature’s Way tends to be successful in towns where people live healthy lives. The author bases his argument on several unsubstantiated assumptions.

The first assumption that the author makes is that there are no other stores like Nature’s Way in Plainsville. Hence based on this assumption he claims that the store will be popular in Plainsville. However, there might be many stores like Nature’s Way in town, and therefore, because of the stiff market competition, Nature’s Way might not prove popular. If there are many stores like Nature’s way in the town, the authors claim that Nature’s way will be successful will be flawed.

Another assumption made is that the increase in sales of running shoes and exercise clothing means that the people in town lead healthy lives. However, this rise in sales may be due to other reasons such as exercise clothing and running shoes being in fashion. If this assumption proves to be wrong , the claim by the author that people in town lead healthy lives will be weakened.

Finally, the author assumes that schoolchildren attending the fitness for life program, which underscores the benefits of regular exercise will lead healthier lives. However, knowing the benefits of exercise does not mean that the children will start leading healthier lives. If the assumption that school children will lead healthier lives is proven unjustifiable, the store may not have school children as customers.

To conclude in order to make his argument persuasive, the author needs to address the assumptions made in the essay.

Argument No.10

"Twenty years ago, Dr. Field, a noted anthropologist, visited the island of Tertia. Using an observation-centered approach to studying Tertian culture, he concluded from his observations that children in Tertia were reared by an entire village rather than by their own biological parents. Recently another anthropologist, Dr. Karp, visited the group of islands that includes Tertia and used the interview-centered method to study child-rearing practices. In the interviews that Dr. Karp conducted with children living in this group of islands, the children spent much more time talking about their biological parents than about other adults in the village. Dr. Karp decided that Dr. Field's conclusion about Tertian village culture must be invalid. Some

Page 13: Y9 Argument Topics

anthropologists recommend that to obtain accurate information on Tertian child-rearing practices, future research on the subject should be conducted via the interview-centered method."

Instruction: "Write a response in which you discuss what questions would need to be answered in order to decide whether the recommendation and the argument on which it is based are reasonable. Be sure to explain how the answers to these questions would help to evaluate the recommendation."

In the statement, some anthropologists recommend that future research on the subject of Tertian child-rearing practices should be conducted through the interview-oriented method because this means of studying is more accurate. To substantiate this recommendation, the author points out that outcomes of the study conducted 20 years ago which used the observation-centered method conflict with the result of the more current researcnh. At first glance, the recommendation advocated by some anthropologists seems to be compelling. Actually it, however, is rife with flaws that should be addressed by answering some important questions.

First, the author claims that the different outcomes of the two researches result from the invalidation of the former study based on one assumption: that during the 20 years the Tertian culture did not change. In other words, Tertian child-rearing has been consistent all the time. This assumption, however, is not supported by any evidence in the argument. So to strengthen the statement, the author should answer the question whether Tertian culture changed during the 20 years by using factual evidence. If there are changes in the child-rearing culture, the claim that Dr. Field’s research is invalid seems untenable, because it is very possible that children were reared by the entire village 20 years ago. What conflict are the facts of different times, not the study results.

Second, Dr. Karp asserts that Dr. Field’s research is inaccurate and invalid based on the confidence that his research must be accurate and scientific. However, there is no solid evidence in the argument support this confidence. To enhance the argument, the author should answer some questions, such as whether Dr. Karp selected his interviewees randomly, whether the number of interviewees is large enough, and whether the content of the interview is objective and neutral without any guiding information biasing the results. If each answers is “yes," the conclusion that A can belie B is more persuasive.

Third, when Dr. Karp gets conclusion from his interview material, he assumes that a correlation exists between child-rearing culture and relationship of children and parents. In other words, he assume that a close relation between children and parents can prove parents foster children by their own strength, and estranged relations certify the conclusion that entire village raise children together. This assumption stands on untenable ground. The author should answer whether there is one possibility that children cultivated by entire village will still have closer emotional relation with parents. If children exactly have intimate relation with parents no matter who rear these kids, then the Dr. Karp’s will be wrong.

If any flaws discussed above are not resolved, the recommendation that to obtain accurate information on Tertian child-rearing practices, future research on the subject should be conducted via the interview-centered method will be irrational.

Argument No. 11

Page 14: Y9 Argument Topics

“The council of Maple County, concerned about the county's becoming overdeveloped, is debating a proposed measure that would prevent the development of existing farmland in the county. But the council is also concerned that such a restriction, by limiting the supply of new housing, could lead to significant increases in the price of housing in the county. Proponents of the measure note that Chestnut County established a similar measure ten years ago, and its housing prices have increased only modestly since. However, opponents of the measure note that Pine County adopted restrictions on the development of new residential housing fifteen years ago, and its housing prices have since more than doubled. The council currently predicts that the proposed measure, if passed, will result in a significant increase in housing prices in Maple County.”

Write a response in which you discuss what questions would need to be answered in order to decide whether the prediction and the argument on which it is based are reasonable. Be sure to explain how the answers to these questions would help to evaluate the prediction.

The council of Maple County has predicted that establishing a measure that would limit the supply of new housing will result in a significant increase in housing prices. This prediction was made based on the fact that Pine County adopted restrictions on the development of new residential housing fifteen years ago and its housing prices have doubled. The proponents of this measure have stated that ten years ago, Chestnut County established a similar measure and there was only a little increse in its housing prices. However, there are few questions that need to be answered before we can conclude that the proposed measure will result in increase in housing prises in Mapple County.

The oponents of the measure have cited the fact about increase in housing prices in Pine County since adopting a similar measure. Was there a high demand for houses in Pine County, soon after this measure was adopted, fifteen years ago? We need to know if new industries were opened in and around Pine County after this measure and if this was the reason for high demand of houses there. People would want to stay in places near their office for known advantages like reduced travelling time from office to home and back home. If there were no such industries developed near Pine County, this would confirm that the increase in housing prices is only due to restrictions on the development of new residential housing.

The argument states that restricting new residential housing development resulted in housing prices increasing to double the previous price in Pine County and only a small increase in housing prices in Chestnut County. To go or not go with the conclusion that adopting the measure will result in increased housing prices, we need the answer to two questions: How many houses are there in Maple County, Chestnut County and Pine County? How much demand does each of these places have for houses? Here is why: the housing prices in Pine County may have doubled because there were less number of houses compared to the increasing demand when the measure was adopted, and the housing prices in the Chestnut County may not have increased much on adopting the measure because there was no high demand for houses in in that area and there were more than enough houses for that much demand. Similarly we need to know the number of houses in Maple County at present and the demand from people for the houses in Maple County in order to conclude if adopting this measure would increase housing prices.

Page 15: Y9 Argument Topics

Hence, knowing answers to these questions would help us in deciding whether the proposed measure, if passed, would result in a significant increase in housing prices in Maple County.

Argument No.12

Prompt: “Fifteen years ago, Omega University implemented a new procedure that encouraged students to evaluate the teaching effectiveness of all their professors. Since that time, Omega professors have begun to assign higher grades in their classes, and overall student grade averages at Omega have risen by 30 percent. Potential employers, looking at this dramatic rise in grades, believe that grades at Omega are inflated and do not accurately reflect student achievement; as a result, Omega graduates have not been as successful at getting jobs as have graduates from nearby Alpha University. To enable its graduates to secure better jobs, Omega University should terminate student evaluation of professors.”

Write a response in which you discuss what specific evidence is needed to evaluate the argument and explain how the evidence would weaken or strengthen the argument.”

The author concludes that Omega University should terminate student evaluations of professors, because it was observed that after the evaluation had been introduced, professors were assigning higher grades to the students. Employers saw this increase in scores as inflated and considered them not accurate reflections of the students' potential. Although, this seems like a logical argument at first, certain evidence needs to be provided to justify its validity.

Firstly, the argument implies that the professors gave a higher grade to students so as to get a better evaluation. The increase in grades could have been due to increased efforts from the professors' side, to increase the possibility of receiving a good evaluation. Evidence that the students did not actually receive a higher grade due to increased efforts of professors and evidence that the increase was only due to professors grading higher, should be obtained. Furthermore, it should be found out whether a professor grading a student higher would actually lead to the student giving a better evaluation to the professor. Some students who score high anyway, may find it unfair to inaccurately grade everyone high. Hence, such students may give a bad review to the professors.

Secondly, from the argument, it seems as though employers look only at the grades of the students. Evidence supporting this fact is needed. Employers usually have a more rigorous selection procedure that is inclusive of grades in addition with many other criteria. There is a likelihood that the students failed to meet the other criteria such as a written test or interview. This could be the reason for the lesser success of Omega University than the Alpha University in securing good jobs.

Thirdly, there is an equal chance that Alpha university also has professor evaluation by students. Evidence regarding the same should be obtained, and if found true, further analysis should be done as to whether the students grades increased after the introduction of the evaluation of professors. If an increase was observed, whether the increase actually affected the placement of students in companies.

Lastly, questions like whether terminating the evaluation of professors would be beneficial need to be answered. This could only lead to more variation in the marks of students that may have a worse impact on the students performance and consequently on their placements.

Page 16: Y9 Argument Topics

Only after obtaining evidence on the above will the college be able to make a decision regarding whether or not terminating the evaluation of the professors would lead to better jobs for the graduates. This would need in depth analysis, but it would ensure that the future of the students would not be at stake.

Argument No.13

Prompt: “In an attempt to improve highway safety, Prunty County last year lowered its speed limit from 55 to 45 miles per hour on all county highways. But this effort has failed: the number of accidents has not decreased, and, based on reports by the highway patrol, many drivers are exceeding the speed limit. Prunty County should instead undertake the same kind of road improvement project that Butler County completed five years ago: increasing lane widths, resurfacing rough highways, and improving visibility at dangerous intersections. Today, major Butler County roads still have a 55 mph speed limit, yet there were 25 percent fewer reported accidents in Butler County this past year than there were five years ago.”

Write a response in which you discuss what specific evidence is needed to evaluate the argument and explain how the evidence would weaken or strengthen the argument.

The author of this argument has mentioned the failure in implementing the reduced speed limits in highways in Prunty county as many drivers exceed the speed limit. Hence the author of this argument suggests that Prunty County undertake same kind of project as Butler County undertook 5 years ago. To support his suggestion, the author cites the fact that there is a 25% decrease in Butler County in the past 5 years even though the speed limit in Butler County is the same as that in Prunty county. We need evidence to rule out other possibilities of reduced accidents in Butler county before we could agree with the arguer and take up the project.

Even after decreasing the speed limits in the highways the number of accidents in the Prunty County has not decreased and reports from highway patrol states that many drivers are exceeding the speed limits. Do these accidents happen only because of increased speed limits? The author fails to consider various other reasons like road conditions and the driving skills of driver. Author should provide evidence about how well the drivers drive in the highways and the road conditions in highways etc. If the roads are smooth and drivers are very good at driving and drive well in all weather conditions, this could strengthen the argument that these accidents happen majorly due to exceeding the speed limits.

Next, the author of the argument suggests to increase the lane width, and resurface the rough highways etc in Prunty County as was done in Butler County. The author should have provided the current lane width, road conditions and visibility at dangerous intersections in both counties and also how these measures were in Butler County before the project was undertaken. This comparison report would have strengthened the argument if from this report we could see that current lane width is less, and road conditions and visibility are worse in Prunty County than in Butler county and hence undertaking this project would improve those conditions of highways in Prunty County.

Finally, the arguer states that undertaking such highway improvement project in Butler County has decreased accidents by 20% in past 5 years. This statement does not have any strong

Page 17: Y9 Argument Topics

evidence to support it. There may be various reasons why there is a decrease in Butler county over the past 5 years. It is given in the argument that Butler County’s current speed limit is 55 mph, whereas there is no information on whether it was the same before the project was undertaken or is the speed limit was higher before and is decreased to 55 mph now. Also, Butler county could have enforced strict rules on wearing helmets and wearing seat belts and following speed limits and heavy fines for people who don't follow these rules. If the decrease in number of accidents is due to one of the above reasons, then this would weaken the argument. Alternatively, if there is no such rules imposed on drivers and the speed limit was not decreased after undertaking the project, this could mean that the number of accidents decreased because of this project only and this would strengthen the argument.

Argument No. 14

Prompt: The following appeared as part of an article in a business magazine. "A recent study rating 300 male and female Mentian advertising executives according to the average number of hours they sleep per night showed an association between the amount of sleep the executives need and the success of their firms. Of the advertising firms studied, those whose executives reported needing no more than 6 hours of sleep per night had higher profit margins and faster growth. These results suggest that if a business wants to prosper, it should hire only people who need less than 6 hours of sleep per night."

Write a response in which you examine the stated and/or unstated assumptions of the argument. Be sure to explain how the argument depends on these assumptions and what the implications are for the argument if the assumptions prove unwarranted.

The author of this article states that the company would prosper only when its executive have little sleep and suggests that companies should hire people requiring six hours of sleep or less. The author has concluded from studies that the firms in which executives have no more than 6 hours of sleep had higher profit margins and faster growth. There may be plenty of other reasons why these companies have faster growth and higher profit margins. Even before hiring such executives who sleep for 6 hours or less at nights, these firms might have been a fast growing state and have had good reputation. Besides, executives being awake for a long time is nowhere related to a firm's success. Being awake late in the night or waking up early in the morning does not mean that they work for the firm late night and in the morning. These executives may be watching a movie or playing games and hence the unstated assumption that a firm's executives who do not sleep for more than 6 hours work for the firm's success is not true.

The author has not mentioned the number of advertising firms studied and how many among them are successful and growing fast as mentioned in the article. Also by knowing the number of executives in these firms who sleep for 6 hours less in each of these firms, we would know if the firms that are growing faster have many executives who has little sleep and the other firms have only very less executives who sleep for 6 hours or less. If the previous statement is not true, then the assumption that a firm grows only when its executives sleep for only 6 hours or less will be proved unwarranted and hence the conclusion that a company has to hire only people who have less sleep becomes incorrect.

Argument No.15

Prompt: “The following memorandum is from the business manager of Happy Pancake House restaurants. ‘Recently, butter has been replaced by margarine in Happy

Page 18: Y9 Argument Topics

Pancake House restaurants throughout the southwestern United States. This change, however, has had little impact on our customers. In fact, only about 2 percent of customers have complained, indicating that an average of 98 people out of 100 are happy with the change. Furthermore, many servers have reported that a number of customers who ask for butter do not complain when they are given margarine instead. Clearly, either these customers do not distinguish butter from margarine or they use the term 'butter' to refer to either butter or margarine.’

Write a response in which you discuss one or more alternative explanations that could rival the proposed explanation and explain how your explanation(s) can plausibly account for the facts presented in the argument.”

In the argument the author concludes that customer do not distinguish butter from given margarine or they use the term ‘butter’ to refer to either butter or margarine. To support the assertion, the author first points out that only about 2 percent of customers have complained, indicating 98 percent of customers are happy with the change. In addition, the author claims that many customers who ask for butter do not complain when they are given margarine instead. However, the conclusion is flawed.

First of all, in the argument, the author points out that only 2 percent of customers have complained, but the author did not specify whether it is 2 percent of customers who purchased products with margarine replacement or customers who purchased other products that are not margarine related, and complained for various reasons. For instance, if two customers complain about margarine out of the total of 100 customers who complain for various reasons, but there are only 10 customers who purchase margarine made products, then, the 2 percent of customers complain about margarine out of 10 customers who actually purchase margarine can be significant, because the real percentage for customers who complain about margarine should be 20 percent instead of 2 percent.

Further, the author assumes that an average of 98 percent of customers are happy with the change based on that evidence that only 2 percent of customers who have complained about margarine replacement. In addition, one evidence mentions that many customers who ask for butter do not complain when they are given margarine instead. This does not support the conclusion effectively because the fact that customers do not complain about margarine does not necessarily mean customers like margarine. It is very possible that customers do not like margarine products, but they think it is too troublesome to complain. Moreover, it is possible that some customers who do not like margarine, and do not bother to complain, will not come back again.

Finally, the author jumps to the conclusion that customers do not distinguish butter from margarine or they use the term ‘butter’ for ‘butter margarine” . There is a leap between presented evidences and conclusion because given evidences are questionable as discussed previously. To bolster the author’s point, more evidences, which directly address customers like or do not mind having margarine or margarine made products, are needed.

Argument No. 16

Prompt: "In surveys Mason City residents rank water sports (swimming, boating, and fishing) among their favorite recreational activities. The Mason River flowing through the city is rarely used for these pursuits, however, and the city park department devotes little of its budget to maintaining riverside recreational facilities. For years

Page 19: Y9 Argument Topics

there have been complaints from residents about the quality of the river's water and the river's smell. In response, the state has recently announced plans to clean up Mason River. Use of the river for water sports is, therefore, sure to increase. The city government should for that reason devote more money in this year's budget to riverside recreational facilities. “

Write a response in which you examine the stated and/or unstated assumptions of the argument. Be sure to explain how the argument depends on these assumptions and what the implications are if the assumptions prove unwarranted.

The author assumes that more money in the budget will increase use of riverside recreational facilities by Mason city residents. This may not be true. It would be necessary to consider what other recreational activities outranked the water sports and if these results were statistically significant. Issues of the age groups that were surveyed, the different wealth groups and even education level and even marital status as all these factors may affect the use of any recreational facilities including water sports.

For example it may be that the Mason City residents may not engage in these activities because they lack skills to swim or row boats. Though they desire to do these activities they are not able to engage in water sports and thus engage in other activities.

It also may be that water sports are expensive. One must own a boat or hire a boat in order to boat or raft. One will need to pay an entrance fee to go swimming, buy a costume and floatation device to swim. If these weighed against sports that are not costly e.g. running or cycling. A bicycle is much cheaper than a boat and so one may choose to cycle than boat and they may choose to run because there is no entrance fee.

Lastly it may be that people truly prefer other sports to water sports. In addition there may be inherent problem with the river that funding on its own may not be able to correct. For example, the river may be too rapid or have wild animals such as crocodiles that may harm people. If there is a history of a crocodile attack – then in it may be difficult to woo residents to go back. The river may be too rapid have other animals e.g. crocodiles that scare people away. The smell that the residents complain about may not be due to dirt but by the rivers water composition. High concentrations of chemicals such as sulphur may give the water a smell and are also toxic to the human population and the water animals.

On the other hand it may be true that devoting more money in this year's budget to riverside recreational facilities may actually stimulate an interest in the water report. If the smell is due to dirt and this is cleaned away, people will be drawn to the fresh clean waters, the fish will breed again and people can take up boating and fishing. If the recreational facilities also involve swimming coaches and lifeguards then even people who are not skilled or are amateurs can venture into the water sports.

That said the best way forward would be to do a situation analysis. The local authority should conduct a cross-sectional survey which further explores why the residents do not use the river, establish if they use other rivers and why and further explore what would make them use Mason River for water sports. These suggestions should then be weighed against the economic viability of the project.

Argument No. 17

Page 20: Y9 Argument Topics

Prompt: The following appeared in a memorandum from the manager of WWAC radio station.

“To reverse a decline in listener numbers, our owners have decided that WWAC must change from its current rock-music format. The decline has occurred despite population growth in our listening area, but that growth has resulted mainly from people moving here after their retirement. We must make listeners of these new residents. We could switch to a music format tailored to their tastes, but a continuing decline in local sales of recorded music suggests limited interest in music. Instead we should change to a news and talk format, a form of radio that is increasingly popular in our area.”

Write a response in which you discuss what specific evidence is needed to evaluate the argument and explain how the evidence would weaken or strengthen the argument.

The manager of WWAC radio station claims that the number of listeners have got reduced despite the increase in population because of broadcasting in rock-music format. The manager has also mentioned that changing this to music format would not work as well as there is a continuing decline in local sales of recorded music. Finally the WWAC manager concludes that broadcast format should be news and talk format as that is increasingly popular in that area. The manager has presented points in such a way that one would agree with the conclusion in the first glance of the memorandum, however there is no strong evidence to make such a conclusion. Presence of the following evidence in the memorandum would help strengthen or weaken the decision that the news station would be a success by broadcasting in news and talk format.

First, the manager has mentioned that there is a decline in number of listeners in the area even when there is population growth due to people moving in after retirement. But if we could have the number of listeners over a period of time, we would know if there is really a decline in number of listeners. For instance, we do not know how the decline was measured. If it is measured by the ratio of number of people in the area to the number of people who listen to WWAC channel, then may be the number of listeners remain the same and only the new people moving to this area are not listening to the channel. So if the format was changed, there is a possibility of losing regular listeners to he channel. If the number of listeners over a period of time has been presented in the memorandum, it would have strengthened the argument if there is really a decline in number of listeners to the radio channel. If there is not much decline in the number of listeners, then this would have weakened the argument.

Secondly, the manager has reported that there is a decline in local sales of recorded music and hence concludes that music format would also not be popular in this area. But if we are provided the number of stores in the local area selling recorded music and the cost of music would help us in evaluating the claim that people in the area have lost interest in music. If there are only limited stores in the area, given that increasing population is only due to people settling in the area after retirement, they would not want to drive to these distant shops to get recorded music. So if we have evidence that there are many shops all over the area, selling recorded music, then this would strengthen the claim that people have lost interest in music and ,as I have mentioned, if these shops are scarce then this would weaken the claim. There is another important factor to be considered regarding the sales of recorded music-price. If there was an increase in cost of recorded music in recent days, then people would have gradually stopped buying them and this doesn't mean that they are not interested in music. If there was no increase in cost, this would

Page 21: Y9 Argument Topics

help us in agreeing with manager's conclusion and if there was an increase in cost then this would weaken the claim.

Finally, if the format was changed to news and talk, we do not know how many of the present listeners would be happy with this change. For this, we need the number of present listeners to this channel and if they would be happy with this change. Also we need to know why other people in the area do not listen to this channel. If the non-listeners reason for not listening as the rock-music format, and if they say that it is likely that they would listen to news and talk format, then it will strengthen the argument that the changing to this format would help in increasing the number of listeners to this channel, otherwise it would weaken the conclusion. Also if the current listeners enjoy rock-music format and if they state that they would not be happy with the change, then changing to news and talk format would make regular listeners upset and never tune to this channel again. If the regular listeners also like this change, then this would help strengthen the conclusion that the number of listeners would increase after this change. If they are not fine about this change, then it would cause the channel to lose regular listeners to the channel and hence this evidence would weaken the conclusion of the memorandum.

Argument No.18

"Over the past year, our late-night news program has devoted increasingly more time to covering national news and less time to covering weather and local news. During the same time period, most of the complaints we received from viewers were concerned with the station's coverage of weather and local news. In addition, several local businesses that used to run advertisements during our late-night news program have just cancelled their advertising contracts with us. Therefore, in order to attract more viewers to our news programs and to avoid losing any further advertising revenues, we should expand the coverage of weather and local news on all our news programs."

Write a response in which you examine the stated and/or unstated assumptions of the argument. Be sure to explain how the argument depends on these assumptions and what the implications are if the assumptions prove unwarranted.

The business manager of WLSS television station is of the view that they should include more weather and local news on all their news programs. This suggestion comes from the fact that since they have devoted their late night news program to covering more national news instead of weather and local news, there have been many complaints from viewers. Moreover, their advertisers have cancelled the contracts. Thus, the business manager feels that the change should be made to attract more viewers and also avoid losing more advertising revenues. However, the business manager does not venture into finding out other possible reasons of complaints and cancellation of advertising contracts. Therefore, this argument presented by him can be called narrowly perceived.

The business manager mentions that the complaints of the viewers were regarding the station's coverage of weather and local news. However, there is no mention of what the viewers want or what the nature of their complaint is. It cannot be established by the argument if the viewers actually want more coverage of local and weather news. It is possible that their complaints are regarding wrongly telecast news or concentration over limited area of local or weather news. It is also possible that their complaints are not regarding late-night news programs that have started covering more national news. It is likely that the viewers have complaints regarding the weather and local news covered in general by the television station.

Page 22: Y9 Argument Topics

Further, the writer says that the local businesses had cancelled their advertising contracts of the late night news programs. However, the writer does not site in vivid terms what the reason of their cancellation of contract is. It is possible that those who had cancelled their contracts were is a financial crisis and could not pay for their advertisements. It is also not known if the businesses had withdrawn their advertising only from the late night news programs or from all the programs of the television station. It is possible that they are not happy with the result of advertising on the television station and want to withdraw their support to the station altogether. Therefore, this may not be due to the late night news programs telecasting more national news. What the writer suggests holds true only if these businesses had withdrawn their advertising of the late night news programs, but the argument fails to give details of the cancellation. This makes the reader believe that there could be other reasons for this cancellation.

Last but not the least, the suggestion made by the writer is based on the conclusion drawn by the writer that late night news programs are not popular with viewers and businesses. However, this conclusion is extended to all the news programs of the television station, which is taking it too far. The writer says that in order to attract viewers and avoid losing any further advertising revenues they should expand the coverage of weather and local news in all their news programs. In the argument, however, he only speaks of the late night programs losing viewership and advertisement contracts. Therefore, this suggestion is doubtful and farfetched.

On the whole, the argument loses its convincing power when viewed critically. It fails to establish the exact reason behind the complaints of the viewers and cancellation of advertising contracts. Hence, the writer should think logically and come to a better conclusion after going through the above given discussion.

Argument No. 19

“Two years ago, radio station WCQP in Rockville decided to increase the number of call-in advice programs that it broadcast; since that time, its share of the radio audience in the Rockville listening area has increased significantly. Given WCQP's recent success with call-in advice programming, and citing a nationwide survey indicating that many radio listeners are quite interested in such programs, the station manager of KICK in Medway recommends that KICK include more call-in advice programs in an attempt to gain a larger audience share in its listening area.

Write a response in which you discuss what questions would need to be answered in order to decide whether the recommendation and the argument on which it is based are reasonable. Be sure to explain how the answers to these questions would help to evaluate the recommendation.

The station manager of KICK radio station in Medway recommends to include more voice-in advice programs as this increased the number of listeners of WCQP radio station in Rockville. The author of this argument has cited the fact that the nationwide survey indicates that many radio listeners are interested in voice-in advice programs. However, a few questions need to be answered before arriving at the conclusion of including more voice-in advice programs in KICK radio station.

First, the author has stated that there was an increase in number of listeners in Rockville area to this radio station since the time of including more voice in advice programs. However, author has not stated any information regarding other radio channels in the Rockville listening area. We would want to know if any other popular radio channel in the Rockville area had shut down their

Page 23: Y9 Argument Topics

station recently due to some problem, which has turned the listeners to other channels which lead to increase in radio audience for this channel. If there was a shut down of some other radio channel in the area, then we cannot conclude that the increase in number of listeners is because of including voice-in advice programs. If there was no such radio channel shut down, then the increase in number of listeners may be due to people liking call-in advice programs.

Secondly, author has cited the results from a nationwide survey indicating that many radio listeners are interested in call-in advice programs. We need figures for the number of radio listeners in the nation and how many of them like such programs. We cannot conclude from the word "many" the ratio of number of listeners who like such programs to total number of radio listeners in the nation. Besides, the survey has been taken nationwide and we do not know how many listeners like such programs in different parts of the nation. May be there are many listeners in Rockville, or other parts of the nation, who like call-in advice programs and may be there are very few radio listeners who like such programs in Medway. If there are not many listeners in Medway who like call-in advice programs, then the conclusion that there will be an increase in listeners to KICK channel after including more of such programs would be wrong. On the other hand, if there are many listeners in Medway area who like such programs, then including more call-in advice programs is worth an attempt to increase the audience of KICK channel.

Including more call-in advice programs to KICK channel seems a reasonable recommendation to increase the audience of KICK channel at the first glance of the argument as the author has cited the survey report and shown Rockville as an example. But above questions need to be answered to conclude that including such programs has a chance of increasing the number of listeners to this channel.

Argument No.20

Read Argument no.18

Argument No. 22

Prompt: "According to a recent report, cheating among college and university students is on the rise. However, Groveton College has successfully reduced student cheating by adopting an honor code, which calls for students to agree not to cheat in their academic endeavors and to notify a faculty member if they suspect that others have cheated. Groveton's honor code replaced a system in which teachers closely monitored students; under that system, teachers reported an average of thirty cases of cheating per year. In the first year the honor code was in place, students reported twenty-one cases of cheating; five years later, this figure had dropped to fourteen. Moreover, in a recent survey, a majority of Groveton students said that they would be less likely to cheat with an honor code in place than without. Thus, all colleges and universities should adopt honor codes similar to Groveton's in order to decrease cheating among students. “

Write a response in which you discuss what questions would need to be answered in order to decide whether the recommendation and the argument on which it is based are reasonable. Be sure to explain how the answers to these questions would help to evaluate the recommendation.

Page 24: Y9 Argument Topics

In the argument, the author suggests that all colleges should adopt honor codes similar to Gorveton’s in order to stop cheating among students, while failing to address several important issues that can potentially affect the validity of the conclusion.

First of all, in the case of Groveton College, the honor code system seems to help reduce the number of cheating cases, however, it is not known whether this is a typical case just for Groveton College or not. The author need to provide more successful examples of colleges across the nation, which adopted the honor code system, in order to make the conclusion that the honor code system is applicable nationwide. On the other hand, if there is evidence showing that the honor code system fails in several other colleges, then the effectiveness of honor code system would need to be re-evaluated.

In addition, the author assumes the figure of cheating cases has dropped after Groveton College implemented the honor code system, without specifying whether the drop of cheating cases is directly due to the effectiveness of honor code system or not. It is possible that students decide not to report cheating after a while, because this might potentially affect their social status. The author needs evidence to support that there is a causality or direct relationship between adopting the honor code system and the drop of cheating.

Furthermore, the author does not compare the effectiveness of the honor code system with other solutions such as counseling session. Nor does the author address whether the honor code system is the most effective one among all others or not. It is possible that the honor code system can help reduce student cheating, but counseling session works even more effectively, and students prefer counseling session more. Without comparisons, the conclusion is very vulnerable.

Finally, the author mentions that according to the survey, students said that they would be less likely to cheat with an honor code in place than without. However, the author did not specify characteristics of students who participated in the survey. If the survey was conducted among students who are proponents of the honor code system, then, the survey is less reliable and biased.

Overall, in order to bolster the conclusion, the author needs to address that more than one case show that the honor code system works, and the system can directly and effectively reduce the number of cheating. Compared to other solution, the honor code system is the best choice.

Argument No.24

“A recently issued twenty-year study on headaches suffered by the residents of Mentia investigated the possible therapeutic effect of consuming salicylates.

Salicylates are members of the same chemical family as aspirin, a medicine used to treat headaches. Although many foods are naturally rich in salicylates, food-processing companies also add salicylates to foods as preservatives. The twenty-year study found a correlation between the rise in the commercial use of salicylates and a steady decline in the average number of headaches reported by study participants. At the time when the study concluded, food-processing companies had just discovered that salicylates can also be used as flavor additives for foods, and, as a result, many

Page 25: Y9 Argument Topics

companies plan to do so. Based on these study results, some health experts predict that residents of Mentia will suffer even fewer headaches in the future. “

Write a response in which you discuss what questions would need to be answered in order to decide whether the prediction and the argument on which it is based are reasonable. Be sure to explain how the answers to these questions would help to evaluate the prediction.

In the argument, the author mentions that some health experts predict that residents of Mentia will suffer fewer headaches, because many food-processing companies plan to use salicylates as flavor additives for foods. However, in order to evaluate the prediction, several questions need to be addressed.

First all of, the prediction is built on that assumption that salicylates can help relieve or reduce headaches. Therefore, the essential question the argument needs to answer is that whether salicylates can indeed treat or relieve headaches. Although it is mentioned the argument that saylicylates are member of the same chimical family as aspirin, which is a medicine used to treat headaches, this does not mean salicylates can be used to treat headaches. Chemical members in the same family can have quite different chemical properties. In addition, only one study provided in the argument is not convincing enough. If many other studies find out that salicylates does not help relieve or cure headaches, then, the prediction would be invalid because salicylates might not contribute to relieving headaches. If this is the case, then, no matter how much more salicylates are added, it would not warrantee that residents of Mentia will suffer fewer headaches in the future.

Then, another question the author needs to address in order to bolster the predication is that if the more salicylates people consume, the fewer people suffer from headaches, and if it is possible that there are some side-effects of overusing salicylates that can make headaches worse. If studies find out that it is true that people would suffer fewer from headaches by increasing the amount of salicylates people consume, then, the prediction would be reasonable. However, if we eventually find out that once the dosage of salicylates a person taking exceeds a certain thresould, then salicylates would have adverse effect on the person, such as noxious, sleep, or even more severe headaches, then, it would not be reasonable to introduce more salicylates to people’s diet.

Overall, the prediction is only reasonable if the author can address that salicylate can actually relieve headaches, and the more salicylate a person consume, the less likely that person will suffer from headache.

Argument No.25

Prompt: The following was written as a part of an application for a small-business loan by a group of developers in the city of Monroe.

“A jazz music club in Monroe would be a tremendously profitable enterprise. Currently, the nearest jazz club is 65 miles away; thus, the proposed new jazz club in Monroe, the C-Note, would have the local market all to itself. Plus, jazz is extremely popular in Monroe: over 100,000 people attended Monroe's annual jazz festival last summer; several well-known jazz musicians live in Monroe; and the highest-rated radio program in Monroe is 'Jazz Nightly,' which airs every weeknight at 7 P.M. Finally,

Page 26: Y9 Argument Topics

a nationwide study indicates that the typical jazz fan spends close to $1,000 per year on jazz entertainment.'”

Write a response in which you discuss what specific evidence is needed to evaluate the argument and explain how the evidence would weaken or strengthen the argument.

In the argument, the author suggests that a jazz music club in Monroe would be a tremendously profitable enterprise. However, additional evidence is needed to validate the conclusion.

First of all, the author needs the statistics to show how many residents are attempting to join the jazz music club in Monroe. The a jazz music club can only be profitable if people attempt to join in the club. If a survey conducted among residents in Monroe shows that residents prefer to stay at home and listen to jazz music on radio, or attend jazz festival, regardless of the fact that there are many jazz fans in the city, and this is the only jazz club, the jazz club would not be profitable. The conclusion would be invalid unless there is evidence showing that there is a large number of jazz lovers in Monroe who are inclined to join the jazz club.

Then, the author need evidence to show that the jazz club can make more money than it costs to run in order to show a jazz music club is a tremendously profitable enterprise. If the author can provide evidence demonstrating that jazz music clubs make significantly more money than they cost to run, then, the conclusion would be convincing. For instance, if there are some statistics of typical revenues of other similar jazz clubs showing that jazz clubs are indeed profitable.

Finally, another evidence needed to evaluate the conclusion is that on average, how much money residents in Monroe county would like to spend on jazz entertainment, and how much they would want to spend on a jazz music club. The nationwide survey may not be applicable for Mornoe’s case, because residents in Mornoe can deviate from national norms. If statistics show that residents in Mornoe are willing to spend thousands of dollars on a local jazz music club, then, this can strengthen the conclusion. Otherwise, the conclusion is not robust.

Overall, in order to conclude that a jazz music club in Mornoe would be a tremendously profitable enterprise, substantial evidence is needed including, people in Monroe are willing to join local jazz club, and are inclined spend money there. Further, a jazz club typically can make significantly more money than it costs to run.

Argument No.27

Prompt: “The following appeared in a letter to the editor of a local newspaper. ‘Commuters complain that increased rush-hour traffic on Blue Highway between the suburbs and the city center has doubled their commuting time. The favored proposal of the motorists' lobby is to widen the highway, adding an additional lane of traffic. But last year's addition of a lane to the nearby Green Highway was followed by a worsening of traffic jams on it. A better alternative is to add a bicycle lane to Blue Highway. Many area residents are keen bicyclists. A bicycle lane would encourage them to use bicycles to commute, and so would reduce rush-hour traffic rather than fostering an increase.’ Write a response in which you discuss what specific evidence is needed to evaluate the argument and explain how the evidence would weaken or strengthen the argument.”

Page 27: Y9 Argument Topics

In this argument, the author suggests that the rush-hour traffic can be solved by adding a bicycle lane to Blue Highway instead widening the current land. However, the author fails to address some details that can affect the validity of the suggestion.

First of all, the author implies that addition of a motor lane to the current highway is not a good solution, because this proposal resulted in a worsening of traffic jams on Green Highway. However, it is fundamentally true that the result of one case does not always predict the result of another case. In this case, we need to know what caused the worsening of traffic jams after widening Green highway, and whether Blue Highway has the same problems as Green Highway does or not. If the worsening of traffic on Green highway is because drivers tend to drive faster after addition of a lane, then, governments could still consider adding a motor lane with a more stringent speed limit. If the worsening of traffic jams on Green Highway is coincidental, or completely irrelevant to adding a new lane, for instance most of these accidents are due to drunk driving, then, Blue Highway can still consider adding a motor lane, because the worsening of traffic has nothing to do with adding a new motor lane.

Moreover, the author claims that there are many keen cyclists, and people will use bicycles to commute after adding a bicycle lane, thus adding a bicycle lane could solve rush-hour traffic problem. Although there are many bicyclists, we need to know how many people among these keen cyclists are willing to use bicycles to commute. It is possible that many of these keen cyclists prefer driving over biking to work, then, it is conceivable that only a small number of people would use the bicycle lane, while motor lanes are heavily used, and this would not resolve the rush-hour traffic on Blue Highway. The alternative possibility can weaken the author’s suggestion.

Overall, in order to make the author’s suggestion more convincing, the author needs to show that there is a significant number of residents willing to bike to work if a new bicycle lane is added, and Blue Highway has the same potential problems as Green Highway which can result in a worsening of traffic after adding a motor lane.

Argument No.28,26

The following appeared in the summary of a study on headaches suffered by the residents of Mentia.

"Salicylates are members of the same chemical family as aspirin, a medicine used to treat headaches. Although many foods are naturally rich in salicylates, for the past several decades, food-processing companies have also been adding salicylates to foods as preservatives. This rise in the commercial use of salicylates has been found to correlate with a steady decline in the average number of headaches reported by participants in our twenty-year study. Recently, food-processing companies have found that salicylates can also be used as flavor additives for foods. With this new use for salicylates, we can expect a continued steady decline in the number of headaches suffered by the average citizen of Mentia."

Write a response in which you discuss what specific evidence is needed to evaluate the argument and explain how the evidence would weaken or strengthen the argument.

The given summary of the study on headaches suffered by the residents of Mentia, tends to propose the use of Salicylates as flavor additives for foods. This conclusion is based on the belief that the use will lead to decrease in the number of headaches encountered in the city of Mentia.

Page 28: Y9 Argument Topics

It relies on the assumption that, the same increase in commercial use of salicylates in food processing companies, will be encountered in the new use of salicylates too. There is a lot of evidence needed in order to effectively evaluate this argument. Without a mention of the required evidence, the argument will prove to be fallacious.

First of all, Salicylates are quoted to be the members of the chemical family as aspirin. But how much of the property does Salicylates have, to treat headaches must be ascertained. Just because, Saliclylates is a member of aspirin family, cannot guarantee that it will possess the same properties.

The foods which are naturally rich in salicylates must be further elaborated and whether they are being purchased by the people must also be made clear. Furthermore, the author quotes that, during the past 'several' decades, food processing companies have started to use salicylates as preservatives. The number of decades must be clearly presented, to solve the conflict, which is to follow in the argument. The author quotes that the commercial use of salicylates is rising on par with the steady decline in the average number of headaches. This, is said to be a result of the study. But the number of participants taken for the study, their age group etc... must be mentioned to get a clear view of the study. Moreover, since the study was conducted for twenty years, we cannot exactly predict whether the decrease in headaches is due to the salicylates added to foods as preservatives, or those salicylates which naturally occur in foods.

Moreover, the author commits an error of correlation-causation. The rise in commercial use of salicylates happened the same time around, when the decrease in average number of headaches were reported. However, the decline in headaches, can be due to a number of other reasons such as: the people becoming aware of their health, the incidence of headaches having been decreased in the past few years etc... Also, evidence has to be provided, whether all the people who have headaches, resort to foods containing salicylates. Because, there can be multiple ways to tackle headaches which have to be weighed against the use of salicylates.

The summary ends up in a hasty conclusion, basing its decision, on the recent findings of food-processing companies. These companies have found that salicylates can be used as flavor additives for foods. But even, if they are used, will those foods be readily accepted by the people? Also, the average number of headaches occuring are already reported to have dwindled. Then, there arises a doubt whether, the demand for foods containing salicylates in any form, will continue. If the demand decreases, then the recommendation will not hold good for implementation. When the link between headaches and foods containing salicylates, is not proved in the first hand, it will be too early to conclude that, introducing salicylates as flavor additives, will increase the sales and reduce the number of headaches.

Thus, in order to evaluate the given argument, lots of evidence needs to be furnished. Some of them include: Salicylates having the property of reducing the occurrence of headaches, elaborate details of the study conducted, details regarding the persons interviewed for the study, relation between the use of salicylates in foods and decrease in the occurrence of headaches, the effectiveness of the decision to use salicylates as flavor additives. These details must be provided, in order to evaluate the argument without any skepticism. Unless the argument is backed up with substantial evidence, it will remain fallible and unsuitable to be taken into consideration.

Argument No.29

Page 29: Y9 Argument Topics

Prompt: “The following appeared in a letter to the editor of a local newspaper. ‘Commuters complain that increased rush-hour traffic on Blue Highway between the suburbs and the city center has doubled their commuting time. The favored proposal of the motorists' lobby is to widen the highway, adding an additional lane of traffic. But last year's addition of a lane to the nearby Green Highway was followed by a worsening of traffic jams on it. A better alternative is to add a bicycle lane to Blue Highway. Many area residents are keen bicyclists. A bicycle lane would encourage them to use bicycles to commute, and so would reduce rush-hour traffic rather than fostering an increase.’ Write a response in which you discuss what specific evidence is needed to evaluate the argument and explain how the evidence would weaken or strengthen the argument.”

In this argument, the author suggests that the rush-hour traffic can be solved by adding a bicycle lane to Blue Highway instead widening the current land. However, the author fails to address some details that can affect the validity of the suggestion.

First of all, the author implies that addition of a motor lane to the current highway is not a good solution, because this proposal resulted in a worsening of traffic jams on Green Highway. However, it is fundamentally true that the result of one case does not always predict the result of another case. In this case, we need to know what caused the worsening of traffic jams after widening Green highway, and whether Blue Highway has the same problems as Green Highway does or not. If the worsening of traffic on Green highway is because drivers tend to drive faster after addition of a lane, then, governments could still consider adding a motor lane with a more stringent speed limit. If the worsening of traffic jams on Green Highway is coincidental, or completely irrelevant to adding a new lane, for instance most of these accidents are due to drunk driving, then, Blue Highway can still consider adding a motor lane, because the worsening of traffic has nothing to do with adding a new motor lane.

Moveover, the author claims that there are many keen cyclists, and people will use bicycles to commute after adding a bicycle lane, thus adding a bicycle lane could solve rush-hour traffic problem. Although there are many bicyclists, we need to know how many people among these keen cyclists are willing to use bicycles to commute. It is possible that many of these keen cyclists prefer driving over biking to work, then, it is conceivable that only a small number of people would use the bicycle lane, while motor lanes are heavily used, and this would not resolve the rush-hour traffic on Blue Highway. The alternative possibility can weaken the author’s suggestion.

Overall, in order to make the author’s suggestion more convincing, the author needs to show that there is a signicant number of residents willing to bike to work if a new bicycle lane is added, and Blue Highway has the same potential problems as Green Highway which can result in a worsening of traffic after adding a motor lane.

Argument No.30

The following appeared as a recommendation by a committee planning a ten-year budget for the city of Calatrava.

"The birthrate in our city is declining: in fact, last year's birthrate was only one-half that of five years ago. Thus the number of students enrolled in our public schools will soon decrease dramatically, and we can safely reduce the funds budgeted for education during the next decade. At the same time, we can reduce funding for

Page 30: Y9 Argument Topics

athletic playing fields and other recreational facilities. As a result, we will have sufficient money to fund city facilities and programs used primarily by adults, since we can expect the adult population of the city to increase."

Write a response in which you discuss what specific evidence is needed to evaluate the argument and explain how the evidence would weaken or strengthen the argument.

A first look at the recommendation above seems sound. However, the underlying assumptions and conclusion drawn by the writer is flawed because the claims are not and the associations drawn by the writer are not supported by cogent proofs.

The writers draws a weak correlation between lower birthrates and enrollment rates. To validate this recommendation, the writer has to show that the current trend would hold in the following years. Also, the writer has to show that holding other factors constant lower birthrates would lead to lower enrollment rates. Some factors the writer has to consider include the number of people that move into the city and the family structure, or adoption rates in the city. For instance if residents adopt kids rather than birth kids, the writer’s argument is weakened.

The writer further draws an spurious association between lower enrollment rates and financing public schools. Even if enrollment rates are reduced factors such as inflation and new educational technology may increase education spending. The failure to show how lower enrollment rates reduces education spending also weakens the writer’s conclusions.

Furthermore, the writer assumes that only children use athletic and recreational facilities. This premise is flawed because both adults and children use such facilities and adults may not use recreational facilities provided by the city. In addition, the writer fails to consider “new adults” that would relocate to other cities, To validate this claim, the writer can show that currently only children utilize these facilities and show future trends that the current adult and expected adults would continue to reside in the city and require these facilities.

In conclusion, the premises and recommendation stated by the writer is unconvincing and faulty. The writer needs to show that lower birth rate is the only factor responsible for lower enrollment rates. The call for reduced spending is invalid because even if there are lower enrollment rates, costs of education may increase thus weakening his claim. Likewise, the assumption that only children use recreational facilities is unsubstantiated.

Argument No.31

The following appeared in a letter to the editor of Parson City's local newspaper.

"In our region of Trillura, the majority of money spent on the schools that most students attend—the city-run public schools—comes from taxes that each city government collects. The region's cities differ, however, in the budgetary priority they give to public education. For example, both as a proportion of its overall tax revenues and in absolute terms, Parson City has recently spent almost twice as much per year as Blue City has for its public schools—even though both cities have about the same number of residents. Clearly, Parson City residents place a higher value on providing a good education in public schools than Blue City residents do."

Write a response in which you discuss what specific evidence is needed to evaluate the argument and explain how the evidence would weaken or strengthen the argument.

Page 31: Y9 Argument Topics

This argument is well presented but far-fetched. It lays a claim that Parson City residents place a higher value on providing a good education in public schools than Blue City residents do. The argument is in effect definitely impractical due to several flaws after a close scrutiny, albeit it may appear plausible at a cursory glance.

First off, a threshold problem comes into being in this argument that the author clearly assumes larger proportion of revenues on education with same number of people means bigger concentration on education. However, this contention is open to a number of interpretations. We would never know if the same amount of residents would not necessarily result in proportional number of students. In this light, Blue city may have spent more on average since the number of students in Parson City might be, say, more than twice than that in Blue City. Thus, without accounting for and ruling out other likely scenarios, by no means could the author conclude that the students in Parson City receive more revenues in on average.

Moreover, even though the author might be able to provide evidence for us to deduce a solution to the problem presented above afterwards, the argument still maintains ill-conceived owing to another problem. It's totally possible that the gross revenue of Parson City is more than 2 times as those of Blue City. In this case, the amount of money in Blue City certainly occupies larger proportion. To corroborate his point, the author should pay a close heed to as well as cope with the representative alternatives, such as the amount of revenue both cities earn every year, at least. Only then could he bolster the conclusion.

Finally, even if the foregoing problems might turn out to be solved by ensuing evidence, a crucial problem remains that is the more money invested in education meaning the higher value on providing education? It's reasonable to cast doubts upon the author's presumption which I reject as inadequate. For instance, the author omits to inform us about the attitude of the residents in both cities. Perhaps they regard money as a means of policy but not a really effective way to improve the quality of education. Pursuing this line of reasoning, it proves to be the author's responsibility to mull over his provisos so as to pave the way for a more tenable argument.

In retrospect, the author seems precipitous to jump to the conclusion based on a series of problematic assumptions pertaining to the residents' attitude, the amount of revenues in both cities, as well as the proportion of students among residents.

To dismiss the spectre of implausibility in this argument, the author ought to come to grips with the problems mentioned above: (1) the proportion of students are the same in both cities; (2) the amounts of revenues are congruent in both cities; (3) and the residents 'attitudes are like the governments, which regard the amount of money as a criterion of the emphasis on education. Only by grasping the gist of sound assumptions could the author draw a convincible conclusion. After all, feckless attempts with a fallible method could be nothing but a fool's errand.

Argument No.32

The following appeared in a memo from a vice president of Quiot Manufacturing.

"During the past year, Quiot Manufacturing had 30 percent more on-the-job accidents than at the nearby Panoply Industries plant, where the work shifts are one hour shorter than ours. Experts say that significant contributing factors in many on-the-job accidents are fatigue and sleep deprivation among workers. Therefore, to reduce the number of on-the-job accidents at Quiot and thereby increase productivity, we should

Page 32: Y9 Argument Topics

shorten each of our three work shifts by one hour so that employees will get adequate amounts of sleep."

Write a response in which you examine the stated and/or unstated assumptions of the argument. Be sure to explain how the argument depends on these assumptions and what the implications are for the argument if the assumptions prove unwarranted

The correlation the vice president draws between productivity and on-the-job accidents is plausible and reasonable. However, the writer weakens his assumptions by failing to identify other factors that may be responsible for on –the- job accidents and showing that Panoply and Quiot plants are comparable.

The vice president assumes that Panoply and Quiot are comparable and this may not be the case. To strengthen his conclusion the writer has to show that both companies are similar in staff strength and processes. For instance if Panoply has more staff than Quiot, the percentages of accidents may be lesser. Relately, the use of percentages may not actually show the true situation. The claim of the vice president would be strengthened if uses actual numbers, that is if Quiot and Panoply are comparable in terms of staff strength.

Also, the writer assumes that productivity would be increased if hours are shortened. This claim is unwarranted because he fails to state why Panoply shortened work hours. Was it for cost cutting rather than productivity reasons? If the company reduced work hours for cost reasons, doing same at Quiot manufacturing may be detrimental. Therefore to strengthen his argument, the vice president has to show that Panoply reduced work hours for productivity reasons.

The writer’s final and over arching claim is that fatigue and sleep deprivation is the cause of on -the- job accidents at Quiot. This claim is flawed because the correlation he draws is weak and unsupported. Other factors may be responsible for on-the-job accidents such as lack of adequate safety measures, lack of training on how to handle equipments. Also, on-the-job accidents could have increased if the company employed new staff that were unfamiliar with the processes. To bolster is recommendation the writer should show that majority of the accidents that occured on the plant were a result of sleep deprivation.

The failure of the writer to provide cogent proofs for his claims makes his conclusion unwarranted. If sleep deprivation and fatigue are not the causes of on-the-job accidents, then the organization does not have to reduce shift hours.

In conclusion, the vice president has to show that holding other factors (similarity between the companies, reason for reduced work hours in Panoply and exact causes of accidents at Quiot) sleep deprivation and fatigue are the causes on the job-accidents.

Argument No.33

Prompt: “The following appeared in a memorandum from the planning department of an electric power company. ‘Several recent surveys indicate that home owners are increasingly eager to conserve energy. At the same time, manufacturers are now marketing many home appliances, such as refrigerators and air conditioners, that are almost twice as energy efficient as those sold a decade ago. Also, new technologies for better home insulation and passive solar heating are readily available to reduce the energy needed for home heating. Therefore, the total demand for electricity in our area will not increase—and may decline slightly. Since our three electric

Page 33: Y9 Argument Topics

generating plants in operation for the past twenty years have always met our needs, construction of new generating plants will not be necessary.’ Write a response in which you examine the stated and/or unstated assumptions of the argument. Be sure to explain how the argument depends on these assumptions and what the implications are for the argument if the assumptions prove unwarranted.”

In the argument, the author suggests that construction of new generating plants is unnecessary because the total demand of electricity may decrease slightly in this area. The conclusion is based on several assumptions that are not addressed.

First of all, the author claims that the demand of electricity will decrease slightly, because several recent surveys indicate that homeowners are increasingly eager to conserve energy. However, this argument is only valid based on the assumption that people will indeed use less electricity, and do what they say. It is possible that people are willing to conserve energy, but many of them may not actually do what they say. If people do not do anything pragmatic that indeed contributes to energy conservation, even though they are willing to do so, then the results of these surveys would not predict the demand of electricity, and this can weaken the author’s argument.

Furthermore, the author argues that people will use less electricity, because energy efficient appliances are available on the market. However, this argument is only reasonable based on assumption that majority of people will use these energy efficient appliances. If this assumption is unwarranted, for instance, people may not purchase these energy efficient appliances, because most of these appliances are usually more expensive than regular ones, then, this could significantly weaken the author’s claim.

In addition, the author also assumes that there is no other product that can potentially increase the demand of electricity. However, it is possible that people start to used electric cars or other appliances that used to powered by other resources such as gasoline, but now using electricity instead. This can increase the demand of electricity, and invalidate the author’s claim.

Finally, even if the demand of electricity indeed decreases in this area, which may not be true, construction of new generating plants may still be necessary. The author assume that the electricity generated by new plants only supplies the demand of this specific area, but if this assumption is unwarranted, for example, it is possible that the additional electricity generated by new plants can supply other surrounding areas. In this case, if the assumption is unwarranted, then it would weaken the author’s claim.

To sum up, the author’s conclusion is based on several unspecified assumptions and if these assumptions are unwarranted, the author’s conclusion would not be convincing.

Argument No.34

Prompt: ‘The vice president of human resources at Climpson Industries sent the following recommendation to the company's president. “In an effort to improve our employees' productivity, we should implement electronic monitoring of employees' Internet use from their workstations. Employees who use the Internet from their workstations need to be identified and punished if we are to reduce the number of work hours spent on personal or recreational activities, such as shopping or playing games. By installing software to detect employees' Internet use on company

Page 34: Y9 Argument Topics

computers, we can prevent employees from wasting time, foster a better work ethic at Climpson, and improve our overall profits."

Write a response in which you examine the stated and/or unstated assumptions of the argument. Be sure to explain how the argument depends on these assumptions and what the implications are for the argument if the assumptions prove unwarranted.’

The vice president of Climpson Industries recommends that in order to improve employees’ productivity, the company should implement electronic monitoring of employees’ internet used from their workstations. However, the recommendation is flawed. First of all, the vice president assumes that if employees are restricted on personal activities, they would then spend more time on their work. However, this assumption is very questionable. Preventing employees from spending less time on internet, does not predict that employees will spend more time on work. For instance, if a employee in not in a working mood, then, this monitoring system may prevent her from doing recreational things online, but it can prevent her from playing games on phones or chatting with others. These alternative possibilities can weaken the recommendation.

Moreover, the recommendation is only valid based on the assumption that the longer hours an employee works, the more productive he is. In some situations, this assumption is unwarranted, because productivity is not a matter of how long a person works, but how efficiently an employee works. Is the worker who spends 8 hours on his work but does not finish the assignment more productive than the other worker who spends 7 hours on his work with an hour break, but finishes the same assignment? This alternative possibility can weaken the vice president argument who assumes that the more time employees spend on their work, the more productive they would be. Even though, it is logical in some sense, but the vice president ignores that sometimes recreational activities can refresh people’s mind, and help them relax, thus improving employees’ productivity.

Further, the vice president assumes that the monitoring systems would force employees to work, thus allowing Climpson to foster a better work ethic. However, this assumption may be unwarranted; it is possible that employees would not like this monitoring system, and feel they are not trusted, then, this can have a negative effect on work ethic at the company, because employees are not happy, and feel the company does not trust them.

Overall, the recommendation is based on several assumptions that are not clearly addressed in the argument. When these assumptions are unwarranted, the recommendation is not convincing.

Argument No.35

Prompt: "The following appeared in a letter from the owner of the Sunnyside Towers apartment complex to its manager. 'One month ago, all the showerheads in the first three buildings of the Sunnyside Towers complex were modified to restrict maximum water flow to one-third of what it used to be. Although actual readings of water usage before and after the adjustment are not yet available, the change will obviously result in a considerable savings for Sunnyside Corporation, since the corporation must pay for water each month. Except for a few complaints about low water pressure, no problems with showers have been reported since the adjustment. I predict that modifying showerheads to restrict water flow throughout all twelve buildings in the Sunnyside Towers complex will increase our profits even more dramatically.' Write a response in which you discuss what questions would need to be answered in order to decide whether the prediction and the argument on which it is based are reasonable.

Page 35: Y9 Argument Topics

Be sure to explain how the answers to these questions would help to evaluate the prediction."

The argument contains several unaddressed details that can potentially affect the validity of the author’s argument. First of all, there is no evidence available to show that the water consumption indeed has decreased after implementing modified showerheads. Then, there is no solid evidence to show that predicted outcome is warranted. The author predicts that the consumption of water will drop by restricting the maximum water flow of all the showerheads. However, the owner ignores an important fact that it is possible when the maximum water flow is reduced to one-third of what is used to be, it takes longer for residents to shower. Consequently, the longer shower time would compensate for the reduced flow rate. After all, this would not contribute to reducing water consumption. If this is the case, the owner’s predication is invalid. Therefore, due to the other possibility, it is questionable if the owner’s suggestion can help reduce water consumption, and further save money on the water bill.

Moreover, the letter implies that residents are not unhappy about modified showerheads, because there is not complaints about showerheads. This is flawed. Apparently, some people notice that there is something wrong with their shower water, and it is possible that they do not realize that the low shower-water flow may not be due to low water pressure but instead to showerhead modification. It is very likely that it has nothing to do with water pressure at all. If the real cause of these complaints about low water pressure is actually showerhead modification, then, the the owner’s suggestion is not applicable.

In addition, even if the manager does not receive complaints about showerheads or water pressure, this does not mean that people are satisfied. It is possible they are unhappy with the showerhead modification, but they think it is too troublesome to complain. It is possible that residents will stop renting Sunnyside Tower complex because of low water flow for all showerheads, but they never complain anything.

Overall, the current argument represented in the letter is not cogent, because there are several unaddressed problems that can invalidate the owner’s suggestion.

Argument No.36

'The following report appeared in the newsletter of the West Meria Public Health Council. "An innovative treatment has come to our attention that promises to significantly reduce absenteeism in our schools and workplaces. A study reports that in nearby East Meria, where fish consumption is very high, people visit the doctor only once or twice per year for the treatment of colds. Clearly, eating a substantial amount of fish can prevent colds. Since colds represent the most frequently given reason for absences from school and work, we recommend the daily use of Ichthaid—a nutritional supplement derived from fish oil—as a good way to prevent colds and lower absenteeism." Write a response in which you discuss what specific evidence is needed to evaluate the argument and explain how the evidence would weaken or strengthen the argument.'

Page 36: Y9 Argument Topics

The author gives a “slippery slope” recommendation based on several causes and effects, which are flawed. Thus, the author’s recommendation is not very convincing. First of all, the author implies that fewer people get colds in East Meria, because people rarely visit the doctor for the treatment of colds. However, based on the common sense, not all people who get colds need to go to see the doctors. Thus, we need evidence such as statistics to show that more people get colds in East Meria, instead of showing there are fewer people who visit the doctor for the treatment of colds, which does not accurately reflect the actual number of people who get colds.

Moveover, the author argues that eating substantial amount of fish can prevent colds, because a study shows that in nearby East Meria, where fish consumption is very high, people rarely visit the doctor for the treatment of colds. Even though it seems like that there is a correlation between eating fish and preventing colds, it is possible that this correlation between preventing colds and fish consumption is coincidental. Is it possible that alcohol regulation in East Meria contributes to fewer colds? Thus, the author needs to give evidence to rule out other possibilities that can potentially contribute to fewer colds, such as regulation of alcohol consumption or banning smoking. Alternatively, if the author can give evidence that directly indicates eating more fish can indeed prevent colds, then, the author’s argument would be more cogent.

Further, the author claims that colds are the most frequently given reason for absences from school and work, and eating fish can prevent colds, which may not be true, thus, eating fish can lower absenteeism. Although colds represent the most common reason for absences, it is possible colds may not be the real reason for these absences. For instance, a student, who wants to stay at home playing computer games, uses getting a cold as an excuse. Therefore, the author needs evidence to show that getting a cold is the real reason for the majority of absences, or how many people fail to go to work or schools indeed get colds? If we find out getting colds is just an excuse for most absences, and getting colds has nothing to do with these absences, then, eating fish would not help lower absenteeism.

Overall, since the recommendation is based on a series of causes and effects, thus, the author has to give direct evidence to valid the causality. Otherwise, the recommendation is not valid.

Argument No.37

The following appeared in a recommendation from the planning department of the city of Transopolis. "Ten years ago, as part of a comprehensive urban renewal program, the city of Transopolis adapted for industrial use a large area of severely substandard housing near the freeway. Subsequently, several factories were constructed there, crime rates in the area declined, and property tax revenues for the entire city increased. To further revitalize the city, we should now take similar action in a declining residential area on the opposite side of the city. Since some houses and apartments in existing nearby neighborhoods are currently unoccupied, alternate housing for those displaced by this action will be readily available."

Page 37: Y9 Argument Topics

The arguments presented by the arguer in support of his stand are vague and baseless. The arguer talks of a comprehensive urban renewal program that was adopted ten years ago and after the lapse of a decade how can the same plan work for the other area of the city? As there must have been remarkable changes in the ways of getting development in a city implementing the same program could not yield the same fruitful results as it did ten years ago.

The arguer talks about the declined crime rate and the increased property tax revenue for the entire city. This assumption about how declined crime is related to several factories being constructed is very vague. The fall in crime could have been due to vigilant staff or more imprisonment of the criminals. Additionally the residents must have become more cautious with time or more security personnel must have been placed in the area.

The arguer has mentioned the increase in the property tax for the entire city. The arguer has not mentioned the separate taxes of the different regions of the city. How can the arguer assume that the increase in property tax was due to the contribution of taxes from this area of the city? The arguer has also not mentioned anywhere that increase in property taxes was due to the newly set up factories.

The arguer talks about construction of industries to bring development in the other part of the city but the arguer is forgetting the further consequences of constructing so many industries in the same city. The arguer thinks industrial development is the only way for development but other measures can also be adopted by government for development. If the land on that side of the city is fertile then that could be taken under cultivation or schools or colleges can be constructed. The set up of industries bring in so much of pollution which could be fatal for the people living in the city.

The arguer fails to make his recommendation acceptable. The arguer should present more relevant arguments in support of his stand and must present some new measures as compared to a plan which was adopted ten years ago.

Argument No.38

Prompt: 'The following appeared in a memo from the new vice president of Sartorian, a company that manufactures men's clothing. "Five years ago, at a time when we had difficulties in obtaining reliable supplies of high quality wool fabric, we discontinued production of our alpaca overcoat. Now that we have a new fabric supplier, we should resume production. This coat should sell very well: since we have not offered an alpaca overcoat for five years and since our major competitor no longer makes an alpaca overcoat, there will be pent-up customer demand.

Page 38: Y9 Argument Topics

Also, since the price of most types of clothing has increased in each of the past five years, customers should be willing to pay significantly higher prices for alpaca overcoats than they did five years ago, and our company profits will increase." Write a response in which you discuss what specific evidence is needed to evaluate the argument and explain how the evidence would weaken or strengthen the argument.

In the argument, the author’s recommendation contains several unaddressed issues that can potentially invalidate author’s conclusion. First of all, the author implies a link between Sartorian's and the other major competitor's having stopped making alpaca overcoats for five years, and the current demand for alpaca overcoats. In order to verify this suggested link, we need evidence about the reason that made the other competitor stop making alpaca overcoats. Is it because the other competitor does not have reliable supplies of high quality wool fabric? If it is because that the other competitor is capable of manufacturing alpaca overcoats, but they find out customers do not like wearing alpaca overcoats anymore, or alpaca overcoats are out of fashion, then, this would not predict a large demand for alpaca overcoats. If the latter possibility is proven to be true, then it would significantly weaken the author’s argument.

In addition, the author claims that people will pay more for alpaca overcoats because the price of most types of clothing has increased. To test the veracity of this claim, we need evidence to show that, nowadays, people are willing to spend more money on coats, or on alpaca overcoats specifically. It is possible that the increasing of clothing price is due to inflation instead of people being willing to pay more on their clothing. If this is the case, people might not want to buy alpaca overcoats at all since alpaca overcoats are relatively expensive. The alternative possibility can weaken the author’s claim.

Finally, the author suggests that selling alpaca overcoats will be more profitable, because the retail price for alpaca overcoats is high. In order to make the argument more persuasive, we need evidence about how much it costs to manufacture alpaca overcoats. If the price of fabric has increased significantly, and compensates for the higher retail price, then, selling alpaca overcoats would not be very profitable as predicated.

Overall, the author’s conclusion is based on several details that the author fails to give substantial evidence to validate. Thus, in order to make the recommendation more sound, the author should provide more concrete evidence.

Argument No.39

A recent sales study indicated that consumption of seafood dishes in Bay City restaurants has increased by 30 percent over the past five years. Yet there are no currently operating city restaurants that specialize in seafood. Moreover, the majority of families in Bay City are two-income families, and a nationwide study has shown that such families eat significantly fewer home-cooked meals than they did a decade ago but at the same time express more concern about eating healthily. Therefore, a new Bay City restaurant specializing in seafood will be quite popular and profitable.

The arguer has drawn the conclusion that establishing a new Bay City restaurant that specializes in seafood would be both profitable as well as popular. The argument relies on two facts as evidence in support of the claim made. The first is that the consumption of seafood dishes in Bay City restaurants have increased over the past five years, but there are no restaurants in Bay City that specialize in seafood. The second fact states that the majority of the families in Bay City are two-income families and the results of a nationwide survey have indicated that two-income

Page 39: Y9 Argument Topics

families are increasingly concerned about healthy eating habits and they are eating fewer home-cooked meals. However, the facts presented by the arguer are insufficient to prove that a new Bay City restaurant specializing in seafood would be popular and profitable.

The fact that there are no seafood restaurants in Bay City, despite the increase in the consumption of seafood, does not necessarily mean that a restaurant specializing in seafood would be popular and profitable. Firstly, the increase in consumption by thirty percent is insignificant in view of the fact that this has happened over a period of five years. Moreover, if seafood was indeed so popular among the residents of Bay City, then restaurants specializing in seafood would have come up by now. Moreover, the restaurants that serve seafood dishes would have diversified into separate restaurants for serving seafood dishes only, if there was a huge demand for the same. The very fact that these restaurants are continuing to serve seafood dishes in addition to other dishes prove that these restaurants are not making much profit from the consumption of seafood.

Additionally, it is likely that the increase in consumption is not only for seafood, but also for the other dishes being served by popular restaurants. People may be going to these restaurants because they like their preparation or the ambience. It is highly unlikely that they will stop going to their favorite restaurants to dine at a new restaurant that serves only seafood. Therefore, the new restaurant will have to compete with the other restaurants that are already popular in Bay City and which offer other dishes in addition to seafood dishes. The new restaurant may not be able to make profit unless it is able to build up a sizeable clientage, which seems unlikely in view of the above mentioned aspects related to dining in Bay City restaurants.

The arguer fails to provide substantial evidence to prove the relevance of the nationwide survey in the context of the two-income families residing in Bay City. There is no mention of the percentage of Bay City families who were a part of this survey. It is likely that the survey included families residing in big cities and metros and the type of lifestyle that they follow permits them to eat out regularly. It is quite possible that the families in Bay City are not too keen on eating out as opposed to their counterparts in bigger cities. Moreover, if the families are health conscious, then they would not like to eat in a new restaurant as they would doubt the quality of food being served and they would rather eat at the restaurants where they have been dining in the past. Therefore, the arguer fails to provide a link between the nationwide survey and the dining habits of the two-income families of Bay City in order to substantiate his recommendation.

The argument could have been strengthened if both the surveys had provided information related to the willingness of the Bay City residents to eat seafood. Moreover, the profits earned by a restaurant depend on the initial investments along with the expenses incurred in the maintenance of the restaurant. The arguer does not provide information related to such data. Therefore, the argument fails to convince the reader that a new Bay City restaurant specializing in seafood will be profitable and popular.

Argument No.40

Milk and dairy products are rich in vitamin D and calcium—substances essential for building and maintaining bones. Many people therefore say that a diet rich in dairy products can help prevent osteoporosis, a disease that is linked to both environmental and genetic factors and that causes the bones to weaken significantly with age. But a long-term study of a large number of people found that those who consistently consumed dairy products throughout the years of the study have a

Page 40: Y9 Argument Topics

higher rate of bone fractures than any other participants in the study. Since bone fractures are symptomatic of osteoporosis, this study result shows that a diet rich in dairy products may actually increase, rather than decrease, the risk of osteoporosis.

Write a response in which you discuss what specific evidence is needed to evaluate the argument and explain how the evidence would weaken or strengthen the argument.

The group of people who have consistently consumed dairy products throughout the years may have had fractures more than other people, but the author of the article makes number of unsubstantiated assumptions about the risk of disease. Based on these assumptions, the author boldly claims that the consumption of the dairy products actually increases the risk of osteoporosis (OS).Though the argument or study is made for the betterment of the society, it may fail to achieve the goal because it is flawed.

The author’s first mistake is to assume that many people think that the disease OS can be prevented by consumption of dairy products and that the disease is associated with number of environmental and genetic factors. Although people may think that the dairy products may be good for their bones on the whole, the author has no basis to assert that people think that the disease is prevented by consuming such products. It is not stated that how many people are of this opinion. May be only the grandma living next door to the author thinks so or may be a very large population is of this view.

Secondly, the author states that the study conducted led him to the conclusion that the people who consume dairy products throughout their lives have a higher rate of bone fracture. The author never states that how many people were included in the study and for how long were they studied. Also it is not stated that what kind of people were included in the group and what were their environmental and genetic condition. May the people that were under study were football players or may be hockey players and there were no computer nerd included in the study group. Obviously, the people involved in games tend to have a higher rate of injury and may be fracture. Also it does not make sense that to consider the consumption of dairy products, the cause of this disease when author himself states that the disease is caused by many environmental and genetic factors.

Lastly, the author states that bone fractures are symptoms of OS, therefore the consumption of dairy products increase the risk of OS. The author states this on the above flawed assumptions and also wrongly assumes that any kind of fracture is an indication of OS. It may be true that OS is accompanied with weak bones but with someone having a fracture because of a car accident is not likely the case of OS. Also it is not stated that in which conditions these fractures occurred. For instance , if the person injured got a fracture by only a gentle kick of his best buddy, may be a sign of poor bone condition. Neither is the body structure or the genetic makeup is stated.

The author’s argument can be very powerful and persuasive if the author were to include certain assumptions. For example if the author were to include the type of study that was carried and by whom, the people that participated in it, their daily activities and genetic make-up and also the detail of why where and how these fractures occurred , can make the authors argument very powerful. With these kind of assertions the authors call might be more justifiable.

Page 41: Y9 Argument Topics

Argument No.41

The following appeared in a health newsletter.

"A ten-year nationwide study of the effectiveness of wearing a helmet while bicycling indicates that ten years ago, approximately 35 percent of all bicyclists reported wearing helmets, whereas today that number is nearly 80 percent. Another study, however, suggests that during the same ten-year period, the number of bicycle-related accidents has increased 200 percent. These results demonstrate that bicyclists feel safer because they are wearing helmets, and they take more risks as a result. Thus, to reduce the number of serious injuries from bicycle accidents, the government should concentrate more on educating people about bicycle safety and less on encouraging or requiring bicyclists to wear helmets."

Write a response in which you examine the stated and/or unstated assumptions of the argument. Be sure to explain how the argument depends on these assumptions and what the implications are for the argument if the assumptions prove unwarranted.

The argument with conclusion that people take more risks as a result of wearing helmets, and recommendation that the government should concentrate more on educating people about bicycle safety, less on wearing helmets is weak. It is based on two studies. However, there are still many missed assumptions, as explained in the following paragraphs.

First, the assumption that the quality of the roads is the same as, or even better than that quality of the roads was ten years ago. If the two qualities are the same, then the comparison will possibly have some sense. On the other hand, providing the quality of the roads is bad, mush rougher than it of ten years ago, perhaps for bad enhancement of the ineffective government, it is more likely to be the reason of more accidents, other than wearing helmets.

Moreover, it should be included the assumption that the traffic condition is still the same as it was. The situation of traffics often changes over time. If, as in some case, the condition happen to be the same, or be better by the improvement done by the government, the conclusion will be strengthened. However, if the circumstance is even worse, resulting from the new industrial areas or popularity of cars with high speed limit, then it may be the element of high accidents rate, which weaken the conclusion.

What's more, the assumption that people will take risks with the feeling of safety. It might be usually the case as in teenagers as well as some reckless adults. However, it is also likely that the sense of safety makes people feel more confidence and relieved such that they can take their own time, riding at ease and slowly. With each decision done with confidence and the resulted discernment, the accidents will likely to decrease, which makes wearing helmets anything but the reason.

In the end, the argument is rather weak. However, if the above three arguments are included, it will be greatly reinforced. Therefore, it is hard to be accepted. The reason for the increasing rate of accidents will probably not be wearing helmet, and the recommendation is useless, ridiculous in fact.

Page 42: Y9 Argument Topics

Argument No.42

Prompt: 'The following is a letter to the head of the tourism bureau on the island of Tria. "Erosion of beach sand along the shores of Tria Island is a serious threat to our island and our tourist industry. In order to stop the erosion, we should charge people for using the beaches. Although this solution may annoy a few tourists in the short term, it will raise money for replenishing the sand. Replenishing the sand, as was done to protect buildings on the nearby island of Batia, will help protect buildings along our shores, thereby reducing these buildings' risk of additional damage from severe storms. And since beaches and buildings in the area will be preserved, Tria's tourist industry will improve over the long term." Write a response in which you discuss what specific evidence is needed to evaluate the argument and explain how the evidence would weaken or strengthen the argument.'

First of all, the author suggests that the tourism bureau on the island of Tria should charge people for using the beaches to raise money for replenishing the sand, and to protect the beaches. To test the validity of this suggestion, we need evidence to show whether charging people for using the beaches will significantly reduce the number of tourists. It is possible that many tourists use the beaches because they are free. If the tourism bureau starts to charge tourists, then, it is possible that the number of tourists would decline significantly. Further, if the number of tourists drops significantly, the tourism bureau on the island of Tria would not be able to raise a lot of funds for replenishing the sand. For this reason, the suggestion will be proved to be invalid, and weaken the argument.

Secondly, the author implies that replenishing the sand can help prevent the erosion of beach sand along the shores. To test the veracity of this claim, we need evidence about whether the project succeed in Batia or not, in other words, whether replenishing the sand can effectively stop the erosion or not. Since no substantial evidence is provided to show that replenishing the sand can effectively prevent erosion. It is possible that replenishing the sand has failed in Batia, and it cannot effectively prevent the erosion of beach, then, there would be no grounds for the author’s suggestion, and the author’s claim is weakened.

Moreover, even granted that replenishing the sand can effectively stop the erosion, the author also suggests that copying the project of replenishing the sand from Batia can help prevent the erosion of beach sand in Tria. To verify the validity of this suggestion, we need evidence about whether this project is suitable for Tria’s case. It is possible that beaches of two islands are facing different situations. For instance, nearby buildings along beaches of Tria are well- constructed, or there are less storms in Tria area, or the erosion in Tria is not as serious as that in Batia. If these alternative possibilities are proven to be true, then, it seems that replenishing the sand is not the appropriate solution, because, replenishing the sand does not solve the specific problem for Tria’s case.

To sum up, in order to evaluate the argument, we need evidence about whether charging tourists for using the beaches will result in the decline in the number of tourists, whether

Page 43: Y9 Argument Topics

replenishing the sand is an effective solution to prevent erosion, and whether it is suitable for Tria’s case or not.

Argument No.43

Prompt: 'The following appeared in a memorandum written by the chairperson of the West Egg Town Council. "Two years ago, consultants predicted that West Egg's landfill, which is used for garbage disposal, would be completely filled within five years. During the past two years, however, the town's residents have been recycling twice as much material as they did in previous years. Next month the amount of recycled material—which includes paper, plastic, and metal—should further increase, since charges for pickup of other household garbage will double. Furthermore, over 90 percent of the respondents to a recent survey said that they would do more recycling in the future. Because of our town's strong commitment to recycling, the available space in our landfill should last for considerably longer than predicted."

Write a response in which you discuss what specific evidence is needed to evaluate the argument and explain how the evidence would weaken or strengthen the argument.

The chairperson of the West Egg Town Council has concluded that the available space in the landfill would last longer than predicted, but has not provided strong evidence from which such a conclusion can be arrived at. First, the assumption that the amount of recycled material would increase further does not have any supporting evidence. Just because the people are charged double for picking up garbage that cannot be recycled doesn't mean that people would use only materials that can be recycled. There are people who can afford to pay even triple the cost for picking up garbage and even other people may try and pay double charge for collecting garbage as they may be sluggish to sort out the recycled materials and other materials and use the recycled materials. The evidence that the people are actually interested in using recycled material is not provided here. One evidence could be the number of people from town who attended the public meeting where usage of recycled material was discussed. Such evidence would have helped in coming to a conclusion on how many people are really interested in keeping their town clean. This would help strengthen the argument if people are interested in recycling and hence the conclusion that amount of recycled material would increase.

The next statement without evidence is about the percentage of respondents who said that they would do more recycling in future. There is no figures of number people who responded or number of residents of the town. If the number of people who responded is more than half of the people who reside in the town and if out of those respondents, 90 percent said that they would do more recycling in future, then this would strengthen the argument. If the number of respondents itself is very low, then positive response from 90 percent of them cannot be taken into consideration for concluding that people would do more recycling in future and this would weaken the argument.

Page 44: Y9 Argument Topics

Finally, the chairperson has stated that people of the town have strong commitment to recycling and concluded that available state in the landfill would last longer than predicted. Again, there is no evidence that people in the town have commitment to recycling. One possible evidence that could confirm this assumption true may be a survey taken throughout the town to see how many people are committed to recycling the garbage. This could strengthen the argument if the number of people showing commitment in the town is more. Otherwise this evidence would weaken the argument. Hence without these supporting evidences the conclusion at which the chairperson has arrived is not convincing.

Argument No.44

The following appeared in a letter to the editor of a journal on environmental issues.

"Over the past year, the Crust Copper Company (CCC) has purchased over 10,000 square miles of land in the tropical nation of West Fredonia. Mining copper on this land will inevitably result in pollution and, since West Fredonia is the home of several endangered animal species, in environmental disaster. But such disasters can be prevented if consumers simply refuse to purchase products that are made with CCC's copper unless the company abandons its mining plans."

Write a response in which you examine the stated and/or unstated assumptions of the argument. Be sure to explain how the argument depends on these assumptions and what the implications are for the argument if the assumptions prove unwarranted.

The author claims that customer's boycott could forbid CCC carrying out their mining plan could protect the African tropical rainforests on the West Fredonia. However, there are no sufficient persuasive evidence to support the advice though it seems appealing at the first glance.

The benefit of customers would be cared firstly. Refusing CCC's copper productions maybe could affect their works or lives, notwithstanding they can refuse CCC brand, which is forged on the direct-productions, how can they realize those indirect productions such as the wire or integrated devices? Once CCC provides their copper to other factory as wire corn, but customers cannot tear the plastically surface down to copper corn to identify if it comes from CCC. So completely refusing the CCC's productions is unpractical.

Moreover, if the CCC gives up their mining plans, other companies will invest to that land for their industrial projects too. Not for the copper mining, but cleaning the forests for cattle grazing, logging, automobile factory or building highways or dams, the local ecosystem would be destroyed too. So to prevent one kind of investment is not a property step, because we cannot refuse all the daily consumptions.

To protect the tropical rainforest, not only West Fredonia but also all rainforest nations should prohibit all kinds of extractive industry. Because the rainforest ecosystem is a delicate balanced web, the species depend on each other, one disappears, another extinct with it, which in turns destroy the entire web. So the sole plan of preventing the CCC’S mining plan is meaningless.

Page 45: Y9 Argument Topics

People would not believe the boycott to the CCC's products will entirely protect the West Fredonia's rainforests unless more information should be provided, such as the nature environmental information about the West Fredonia, the CCC's production list and the analysis of the plan's possibility.

Argument No.45,49

The following is part of a memorandum from the president of Humana University.

"Last year the number of students who enrolled in online degree programs offered by nearby Omni University increased by 50 percent. During the same year, Omni showed a significant decrease from prior years in expenditures for dormitory and classroom space, most likely because instruction in the online programs takes place via the Internet. In contrast, over the past three years, enrollment at Humana University has failed to grow, and the cost of maintaining buildings has increased along with our budget deficit. To address these problems, Humana University will begin immediately to create and actively promote online degree programs like those at Omni. We predict that instituting these online degree programs will help Humana both increase its total enrollment and solve its budget problems."

This argument is well presented yet far-fetched. It lays a claim that the increasing number of students and the decreasing expenditures for dormitory and classroom in Omni University is because of the new degree program - online program. Nevertheless, the argument is in effect definitely unreasonable due to several flaws after a closer scrutiny, though it may appear plausible at the first glance.

To begin with, the president of Humana University imputes the decreasing number of enrollments to the lack of the online degree program, like the nearby Omni University. Maybe the true cause of the Omni University to implement the program is because it is inconvenient traffic condition. It might locate at the barren area that impedes their students to go to school and forces them to take the online degree program. And the Humana University fails to increase the number of students is due to its diminishing quality of teaching or its outmoded facilities.

Secondly, the author states that the decreasing expenditures for dormitory and classroom in the Omni University is due to the online program which enable the students to take the courses online at home or other places and don't need the dormitory or classroom. Maybe the dormitory and classroom in the Omni University are much fresher than those in the Humana University which cost the school to spend even more money to maintain.

Page 46: Y9 Argument Topics

Thirdly, the president of the Humana University insists that in order to attract more and more students to enroll, they should immediately implement a new program just like Omni University. However, the president fails to consider the opinion of the students. By replacing the traditional method with the online program, the students who enjoy hanging around with classmates and the lifestyle in the campus will refuse to take the courses and leave. Finally, the Humana University will not be able to solve its budget problems, even worsen.

In retrospect, it seems precipitous for the author to jump to the conclusion based on a series of problematic premises. To bolster it the president has to provide clear evidence that the online degree program contributed to the increasing number of enrollment in the Omni University and the program will attain the same effects in the Humana University. After all, feckless attempts with a fallible method could be nothing but a foolish errand. Thus, only by grasping the key point of this argument could the author draw a convincible conclusion.

Argument No.46

The following appeared in a health magazine published in Corpora.

"Medical experts say that only one-quarter of Corpora's citizens meet the current standards for adequate physical fitness, even though twenty years ago, one-half of all of Corpora's citizens met the standards as then defined. But these experts are mistaken when they suggest that spending too much time using computers has caused a decline in fitness. Since overall fitness levels are highest in regions of Corpora where levels of computer ownership are also highest, it is clear that using computers has not made citizens less physically fit. Instead, as shown by this year's unusually low expenditures on fitness-related products and services, the recent decline in the economy is most likely the cause, and fitness levels will improve when the economy does."

Write a response in which you examine the stated and/or unstated assumptions of the argument. Be sure to explain how the argument depends on these assumptions and what the implications are for the argument if the assumptions prove unwarranted.

The argument with conclusion that fitness levels will improve when the economy does is rather weak. It is based on the premises that only one-quarter of Corpora's citizens meet the current standards for fitness, even though one-half met the standards as then defined twenty years ago, that overall fitness levels are highest where levels of computer ownership are also highest, resulting in that using computers has not made citizens less physically fit, and that this year's unusually low expenditures on fitness-related products and services. However, there are many hidden assumptions needed to be supposed, as explained in the following paragraphs.

First, the assumption that the proportion of citizens meet the current standards is lower than the proportion of citizens twenty years ago met the standards now has to be verified, for the standards must be the same in order to make the comparison meaningful. If the assumption is

Page 47: Y9 Argument Topics

demonstrated, the total discussion will have sensible base, making the argument more strong. However, the lack worse the validity of the argument.

Moreover, the assumption that the relation between computer and fitness is unrelated needs to be testified. It is claimed in the argument, without any proof. The fact that overall fitness levels are highest in regions of Corpora where levels of computer ownership are also highest is simply not enough. It is the fact that the people fond of using computer have the some condition of fitness as those who have no preference in computer that makes sense. This kind of detailed description needed to be mentioned in the statement that confirmed the resumption, strengthening the argument.

Furthermore, the assumption that trend which people tend not to purchase fitness-related products and services is a result of tighter budget should be certified. There is a tendency that people just stop buying these goods, but the reason is not stated. Although economy is one answer, without evidences such as polls of predisposition of using fitness-related products and services and possible cause of leaving them, the assertion is just point of view of the author, not verified. The conjecture ought to be proved, so as to fortify the logic of the argument.

In the end, the argument is rather weak. It is based on shaky premises and flawed logic. However, if the above assumptions are explained in the statement, it will be greatly reinforced. Therefore, the conclusion is not acceptable, more facts needed to be presented.

Argument No.47

The following appeared in a memorandum from the owner of Movies Galore, a chain of movie-rental stores.

"In order to stop the recent decline in our profits, we must reduce operating expenses at Movies Galore's ten movie-rental stores. Since we are famous for our special bargains, raising our rental prices is not a viable way to improve profits. Last month our store in downtown Marston significantly decreased its operating expenses by closing at 6:00 P.M. rather than 9:00 P.M. and by reducing its stock by eliminating all movies released more than five years ago. By implementing similar changes in our other stores, Movies Galore can increase profits without jeopardizing our reputation for offering great movies at low prices."

Write a response in which you examine the stated and/or unstated assumptions of the argument. Be sure to explain how the argument depends on these assumptions and what the implications are for the argument if the assumptions prove unwarranted.

The author hastily puts forward two solutions to increase profit at Movies Galore rental store. Firstly, operating expenses should be cut down. Secondly, old stock of movies should be removed. At first glance, the authors argument seems somewhat convincing but further reflection reveals that the statement omits some substantial concerns that should be addressed and thus not cogent enough to substantiate the argument. At least, this argument suffers from three main logical flaws.

To start with, the author’s assumption that reduction in operating expenses can stop the recent decline in profit is unwarranted. If according to the stated elucidation, the store activities are seized three hours earlier, people wishing to relax after office hours can be deprived of

Page 48: Y9 Argument Topics

entertainment. They can look for other stores that can provide them with some good relaxing movies. If this be the case, the income of rental movie store will automatically decrease. Operating expenses can be decreased by limiting the glamor of the store or the workforce. The author needs to have a comprehensive study of expense sheet and can think of other ways to control expenditures.

In addition, the author also proposes to have the stock of movies released not older than five years. The author has failed to consider that super hit classic movies of all times are evergreen. People find pleasure in watching old movies and if they will not be available at Movies Galore rental store, the customers will definitely change their choice to some other movie store that can fulfill their demands.

Finally, the author assumed that implementing the same measures as done in the store in downtown Marston will be beneficial in raising profit. However, it is not a well-thought-out assumption. It may be possible that people residing in downtown Marston are not very fond of movies which may not be the case at other rental stores. The author has failed to discuss other factors that can help raise profit with same working hours. May be more customers can be attracted by effective advertising or entertaining customers with special offers on movies rent.

To sum up, the author fails to substantiate his claim that the profit can be boosted by lessening the working hours or reducing the stock, because the proofs referred in the analysis do not lend strong support to what the author maintains. Without additional evidences, however, we should be very circumspect about accepting the truth of the author’s conclusion.

Argument No.48

The following appeared in a magazine article about planning for retirement.

"Clearview should be a top choice for anyone seeking a place to retire, because it has spectacular natural beauty and a consistent climate. Another advantage is that housing costs in Clearview have fallen significantly during the past year, and taxes remain lower than those in neighboring towns. Moreover, Clearview's mayor promises many new programs to improve schools, streets, and public services. And best of all, retirees in Clearview can also expect excellent health care as they grow older, since the number of physicians in the area is far greater than the national average."

The argument that encouraged people to choose a quiet place to retire for the benefits of economy and health may seem logical at first glance. The author makes a valid argument, one that would be correct if its premises were true. However, his conclusion relies on assumptions for which there is no clear evidence, and it uses terms that lack definition.

First, the writer assumes that a clearview place represents natural beauty and suitable environment. In fact, there is not a concrete connection among them. For example, a natural beautiful place may include a beautiful lake, a spectacular mountain, and a wilderness area where elderly can enjoy fresh air and quiet while a clearview place has nothing around it. Furthermore, the author assumes that people want to live in quiet place after they retire. Indeed, many of whom prefer to live in big cities where the public transportations, restaurants, and other services are very convenient, and their families and their friends live near them.

Page 49: Y9 Argument Topics

Additionally, the writer provides no evidence that housing costs in clearview place have decreased. In order to convince customers, he should give the specific statistic of how many percent of housing costs were reduced, and the rate of the decrease to compare with others places. Moreover, which programs will benefit to elderly should be showed from many new programs that clearview’s mayor promises. In some cases, improving school systems and public services advantage to young people than old people because retirees often spent a lot of time at home, and they pay more attention for health care systems and their pensions. The assumption is that if the number of doctors in this place is greater than others, there is not anybody can trust that the quality of health care systems in clearview place is better than others without some evidences.

In conclusion, the writer would not necessarily be wrong to assert that clearview place brings many advantage for retirees. But to support his conclusion, the writer must first define the clearview place more clearly and submit more conclusive evidence that housing costs, health care systems, and public services are suitable for retirees.

Argument No.50

An ancient, traditional remedy for insomnia—the scent of lavender flowers—has now been proved effective. In a recent study, 30 volunteers with chronic insomnia slept each night for three weeks on lavender-scented pillows in a controlled room where their sleep was monitored electronically. During the first week, volunteers continued to take their usual sleeping medication. They slept soundly but wakened feeling tired. At the beginning of the second week, the volunteers discontinued their sleeping medication. During that week, they slept less soundly than the previous week and felt even more tired. During the third week, the volunteers slept longer and more soundly than in the previous two weeks. Therefore, the study proves that lavender cures insomnia within a short period of time.

Write a response in which you discuss what specific evidence is needed to evaluate the argument and explain how the evidence would weaken or strengthen the argument.

The argument with conclusion that the scent of lavender flowers is an effective remedy for insomnia is rather weak. It is based on the study in the statement. However, there is still many missed evidence needed to be explained, as stated in the following paragraphs.

First, the evidence that volunteers really have chronic insomnia. The volunteers have to prove, through some demonstration from the doctors or something, that they really suffer from this disease, so as to make the experiment reasonable. On the contrary, if they are fake, just longing

Page 50: Y9 Argument Topics

to sleep on fancy, lavender-scented pillows, the whole study will fail. As a result, the evidence have to be explained, for the sake of the strength of the result.

Moreover, even if they all suffer form the sickness, the evidence that the deviation of the result in the study is rather low should be mentioned. The variation from all the volunteers ought to be rare in order to fortify the result. It is the number of volunteers that is crucial, that is, just 30. In almost all the study, the number of tests is very low, and the deviation, as a consequence, may rise to be huge, overshadowing the reliability of the study. In order to overcome all the suspicions, the evidence should be provided.

In addition, with certification of the above two evidence, there is still one needed to be testified, that is, the evidence that the control in the study is so firm that no volunteers took their medicine secretly. The discontinuity of taking drugs is vary essential, to such an extent that if such a restrict is flawed, all the study will be indecisive. The confirmation of a strict control should be stated, in order to make the argument meaningful.

In the end, although many is provided, there is still a plenty of evidence needed to be explained, leaving the conclusion even weaker. However, if the above three is justified in the argument, it will be greatly reinforced. Therefore, there is no way to simply take the claimed remedy to be true. More tests needed to be conducted.

Argument No.51

The following memorandum is from the business manager of Happy Pancake House restaurants.

"Recently, butter has been replaced by margarine in Happy Pancake House restaurants throughout the southwestern United States. This change, however, has had little impact on our customers. In fact, only about 2 percent of customers have complained, indicating that an average of 98 people out of 100 are happy with the change. Furthermore, many servers have reported that a number of customers who ask for butter do not complain when they are given margarine instead. Clearly, either these customers do not distinguish butter from margarine or they use the term 'butter' to refer to either butter or margarine."

Author has mentioned a memorandum from the business manager of Happy Pancake House restaurant. It is talking about the replacement of the butter with margarine. There are main two points are discussed here. One is effect of replacing the butter with margarine to the customer and another one is customers power of distinguishing the butter and margarine.

Here it is stated that two percent of the customers have complained and other 98 percent of the customer are satisfied. But one thing is that it is not necessary that all the customer who are not

Page 51: Y9 Argument Topics

satisfied with the change make the complain. Some customer may be unhappy with the change but they avoided making complain.

Some of the customer coming first time may think that the quality of the butter in Happy Pancake House restaurant is of such a bad only so deciding not to visit the restaurant again they left the restaurant. So such people may be out of count causing the result wrong that 2 percent customer unsatisfied.

Then author is adding the comment from servers that a number of customer who ask for butter do not complain when they are given margarine instead butter. But it doesn’t mean that customer can not distinguish the different between butter and margarine. As people are using butter very often so at least some of the customer would surely distinguish the difference between both. It might that customer don’t like arguing and making issue so they might not have complained.

In the last paragraph author is criticizing the distinguishing power of customer. Author us saying that either they could not distinguish both or they refer margarine and butter as the same think. To decide in better way one has taken review from each of the customer and analyzing the reason could have stated that whether they are satisfied with the change or not.

Reporting from the server may be false and they might have said such a comment for their personal benefit or due to some other issue. So as it states, author arguments are not compelling. To strengthen the argument manager needs to take the review and can also put feedback form for their experience in the restaurant. Such a result can let manager to decide whether customer are satisfied or not. So based on the arguments given in the main paragraph one cannot say any thing about customer distinguishing power or their satisfaction regarding the replacement of butter with margarine.

Argument No.52

The following appeared in a letter from the owner of the Sunnyside Towers apartment complex to its manager.

"One month ago, all the showerheads in the first three buildings of the Sunnyside Towers complex were modified to restrict maximum water flow to one-third of what it used to be. Although actual readings of water usage before and after the adjustment are not yet available, the change will obviously result in a considerable savings for Sunnyside Corporation, since the corporation must pay for water each month. Except for a few complaints about low water pressure, no problems with showers have been reported since the adjustment. I predict that modifying showerheads to restrict water flow throughout all twelve buildings in the Sunnyside Towers complex will increase our profits even more dramatically."

Page 52: Y9 Argument Topics

Write a response in which you discuss what questions would need to be answered in order to decide whether the prediction and the argument on which it is based are reasonable. Be sure to explain how the answers to these questions would help to evaluate the prediction.

The owner of the Sunnyside towers apartment complex predicts that their profits will increase by modifying the showerheads of all the twelve towers in the complex. However, the assumptions and the predictions of the owner raises a lot of questions that need to be answered before the argument seems plausible.

The first question that needs to be addressed is that, what is the basis of his assumption that the water use in the complex will be reduced just by modifying the showerheads? It is not necessary that maximum water is used by people under the shower. It is also not necessary that people take bath under a shower. It can happen that people might use buckets and tumblers while using water and the showers are lying idle. Also, the prediction that water bills will reduce and profits will rise before the results of water usage is available is questionable.

Another question that can be raised is: what is the contribution of water bills in the overall expenses of the complex? If the water bills contribute to just a fraction of the overall expenses of the complex, then this assumption fails to gain any favor. This is because if efforts are made to reduce costs in that part of the budget which already contribute minimally in the expenses, it is highly unlikely that the profits will rise dramatically. Thus, the owner may want to try and find other amenities that are likely to rise their profits if used economically.

Also, it can be asked whether any polls were being conducted before taking the decision of modifying the showerheads? These polls would have asked the residents about their water usage such as whether they use showers too much or whether their use of water is for other purposes where showers are not used such as cooking, gardening etc. If this step was taken, it could've given a better idea to the management about the steps to be taken to save water rather than just doing trials like these showerheads modification.

Thus, the argument put forward by the owner of Sunnyside Towers apartment complex about modifying showerheads stands a lot of scrutiny and answering. If those questions discussed above are not answered, the assumptions and predictions of the owner are absolutely implausible.

Argument No.53

Prompt: ‘The following appeared in a health magazine. "The citizens of Forsythe have adopted more healthful lifestyles. Their responses to a recent survey show that in their eating habits they conform more closely to government nutritional recommendations than they did ten years ago. Furthermore, there has been a fourfold increase in sales of food products containing kiran, a substance that a scientific study has shown reduces cholesterol. This trend is also evident in reduced sales of sulia, a food that few of the most healthy citizens regularly eat." Write a response in which you discuss what specific evidence is needed to evaluate the argument and explain how the evidence would weaken or strengthen the argument.’

Page 53: Y9 Argument Topics

First of all, the author suggests that the citizens of Forsythe have adopted more healthful lifestyles since the survey shows that their eating habits are more nutritious. To evaluate this claim, we need evidence about if the survey is representative. If the sampling population of that survey is not large enough, or the participants they chose were biased, then, the result of the survey would not reflect eating habits of the majority citizens, and the result would not be conclusive. For this reason, the author’s argument would be weakened because the given supporting evidence is problematic.

Moreover, the author also implies the link between the increase in sales of food products containing kiran and citizens having adopted healthful lifestyles. To test the veracity of that implied link, we need evidence to show whether citizens consume more food products containing kiran because they want to be healthy, and thus they purchase more food products containing kiran on purpose, or whether food manufacturers added kiran into various food products while consumers are not even aware of that. If the increase in sales of food products containing kiran is a coincidence, and people buy more food products containing kiran for other reasons instead of realizing kiran can help reducing cholesterol, then, the author’s claim would be weakened.

Furthermore, the author also mentions the link between sales of sulia and citizens having adopted healthy lifestyles. In order to evaluate this claim, we need evidence about the nutritional content of sulia, and whether people choose not to buy products containing sulia solely due to their health concerns. The argument would be weakened if the evidence show that sulia is actually a healthful substance; but even if sulia is not good for the health, there can be a plethora of reasons (such as bad taste, expense) other than health concerns that prevent people from purchasing food products containing sulia.

To sum up, in order to evaluate the author’s conclusion, we need evidence to show whether the survey is reliable, and whether people choose to buy or not buy some food products due to their health concerns.

Argument No.54

Humans arrived in the Kaliko Islands about 7,000 years ago, and within 3,000 years most of the large mammal species that had lived in the forests of the Kaliko Islands had become extinct. Yet humans cannot have been a factor in the species' extinctions, because there is no evidence that the humans had any significant contact with the mammals. Further, archaeologists have discovered numerous sites where the bones of fish had been discarded, but they found no such areas containing the bones of large mammals, so the humans cannot have hunted the mammals.

Page 54: Y9 Argument Topics

Therefore, some climate change or other environmental factor must have caused the species' extinctions.

In this argument, the author concludes that humans were not a factor in the extinction of large mammal species found in the Kaliko islands 7,000 years ago. The author has tried to justify his statements by pointing out that there is no evidence that the humans had any significant contact with the mammals. According to him, while archeologists have found bones of discarded fish in the islands, they have not found any discarded mammal bones there. However, the author has not considered the possibility that humans might have exported the parts of mammals especially bones during that period. The author has completely overlooked this possibility. Without considering this explanation, he has jumped to the conclusion that humans were not a factor in their extinction from the islands. There is also a possibility that the humans hunted the large mammals for food, and disposed the bones as well so that none was left behind as evidence. There are many cultures that destroy the bones and all other parts of mammals. Hence, this could be a reason that there was simply nothing left of the mammals to be found by the archaeologists. Hence, this point taken by the author is not valid.

This argument depends upon the author's assumption that without significant contact with these mammal species, humans could not have been a factor in their extinction. It might be that the humans had not done anything significant to these animals, but intruded their territory and natural habitat. As a result, these animals might have left their living areas. There is also a possibility of the humans destroying the food sources of the mammal species knowingly or unknowingly. Perhaps the humans consumed the plants and animals on which these species relied for their existence. These days many species are facing extinction due not to the animals being killed by humans, but by the elimination of their food sources and habitats. The humans are encroaching upon their living spaces to accommodate their increasing population. The author has failed to address these possibilities. Hence, it critically weakens the argument.

Lastly, the author has assumed that the bones of fish that archeologists have found discarded on the island were discarded by human beings, and not by some other large animal. However, the speaker has provided no evidence to support this assumption. The same thing applies to the mammals also. Hence, this evidence lends little credibility to the author's theory about the extinction of large species of mammals.

The evidence given by the author in his support is very unconvincing. To strengthen his argument, the speaker should have ruled out the possibility that humans exported the bones of these other species or that some other animal has not eaten these species. The author must have collected information about the food habits of the humans and these mammal species during that time. That would have helped to evaluate the actual reason for the extinction of these species. Without the right information, this argument is pure speculation and cannot be justified.

Argument No.55

The following appeared in an editorial in a business magazine.

Page 55: Y9 Argument Topics

"Although the sales of Whirlwind video games have declined over the past two years, a recent survey of video-game players suggests that this sales trend is about to be reversed. The survey asked video-game players what features they thought were most important in a video game. According to the survey, players prefer games that provide lifelike graphics, which require the most up-to-date computers. Whirlwind has just introduced several such games with an extensive advertising campaign directed at people ten to twenty-five years old, the age-group most likely to play video games. It follows, then, that the sales of Whirlwind video games are likely to increase dramatically in the next few months."

Write a response in which you examine the stated and/or unstated assumptions of the argument. Be sure to explain how the argument depends on these assumptions and what the implications are for the argument if the assumptions prove unwarranted.

Whirlwind video games have experimented a sales decline over the past two years, in order to increase the sales they have introduced several games which are really likely to attract customers according with a survey. However, the conclusion article relies in some assumptions which are not proved at all or for which there is no clear evidence. Hence, the argument is unconvincing and has several flaws.

First of all, although the argument stated that Whirlwind video games had experimented a sales decline, any figure to support that is presented. Reading the argument is impossible to know if the sales decline is important or not. A sales decrease could represent a 1% of sales or a 90% and, therefore, without further information, is impossible to asses how important this decline is. However, the article seems to assume that the decline is really important. If the decrease of sales is proved not to be important, any decision to fix it should be taken and the article at all becomes unnecessary.

Secondly, the main causes of this decline are not addresses in the paper. The selection of video games in the shop is not the only factor to take into account, as the author readily assumes. For example, the prirate download or the opening of shopping centre near the shop could also affect the sales. In case that there exists any other possible reason to explain the decline, the solution of changing the videogames selection could not fix the problem at all.

Finally, the decision of what videogames introduce to attract new customers is based on a survey which scope is not stated in the article. The author trust on the survey results while the is not enough information. How many people had been interviewed? What ages-groups were they? Further information is needed in order to decide how reliable the survey is.

Summarizing, the argument is flawed because of the above mentioned causes and is therefore unconvincing. Moreover, without further information that proves if the assumptions right or wrong, the argument remains unsubstantiated.

Argument No.56,57

Page 56: Y9 Argument Topics

The following appeared in a memo from the vice president of marketing at Dura-Socks, Inc.

"A recent study of Dura-Socks customers suggests that our company is wasting the money it spends on its patented Endure manufacturing process, which ensures that our socks are strong enough to last for two years. We have always advertised our use of the Endure process, but the new study shows that despite the socks' durability, our customers, on average, actually purchase new Dura-Socks every three months. Furthermore, customers surveyed in our largest market—northeastern United States cities—say that they most value Dura-Socks' stylish appearance and availability in many colors. These findings suggest that we can increase our profits by discontinuing use of the Endure manufacturing process."

The argument put forward by the vice president of marketing at Dura-Socks Inc., in order to discontinue the current Endure manufacturing process owing to their less durability is cast with serious assumptions. The recommendation further assumes that on discontinuing the Endure manufacturing process used to manufacture socks, we can increase the profit. But how this actually happens is of doubt and several questions need to be answered before arriving at a conclusion based on the recommendation by the vice president.

First of all, the time during which the study was conducted is not clearly mentioned. If the study had been conducted years before, then it is least assured that the same effects will continue till now. Therefore, the exact time during which the study was conducted must be ascertained in order to proceed further with the argument. Also, the group of customers who suggest that the company is wasting money on Endure manufacturing process must be thoroughly scrutinized and they must be provided with a questionnaire so that they can ascertain their side and prove their point.

Secondly, the Endure manufacturing process is quoted to be patented; then how come, will there be any doubt of following the process? This question must be analyzed in great detail because unless a process is of vital importance and providing great benefits, it will least likely be patented. Then how come a patented process provide negative effects? Moreover, the Endure process is said to be advertised to the people and the people are found to purchase the socks once in every three months. But how come these two processes are interlinked with the notion of "durability" must be clearly explained. If, the people buy socks once in three months, does it necessarily imply that the durability of the socks has gone worst? It can be due to numerous other reasons like the need of more number of same kind of socks due to high demand, increase in population leading to the increase in number of socks etc...

The author mentions about a survey conducted in the popular market of Dura Socks. But questions like: How many customers were examined? What form of questionnaire was used to elicit information from them? Does preference for stylish appearance imply that there can be increase in the sales of socks? Moreover, there might be a case that the profits incurred are solely due to the usability of the product. In this case, the color and aesthetic nature of the socks becomes a second nature and of least importance. Hence these factors must be examined before deciding that color and variety of socks will surely lead to increase in profits.

The Endure manufacturing process is said to have been patented and passed a large number of rigorous testing stages in order to be eligible for use. It also ensures that the socks will remain good for two years. Hence reverting to a new process altogether will do more harm than good. What effects will the discontinuing of process have on the sales of Dura Socks? Will there be

Page 57: Y9 Argument Topics

additional production methods available which outweigh the benefits of Endure manufacturing process? These points need to be considered before discontinuing the Endure manufacturing process.

Hence the argument put forward in the memo from the Vice president of Dura Socks Inc., leads to several questions like: the time during which the study is performed, validity of the study, the nature of patented process, durability playing a role in purchasing socks frequently, nature of survey conducted to examine the preference of customers, existence of other processes similar in nature and advantageous than Endure manufacturing process. These questions must be answered properly. Otherwise, it will go a long way in impeding the progress of the company and taking the company towards negative growth. Therefore, the words of the Vice President must be analyzed properly in order to decide whether they have to be considered for implementation.

Argument No.58

The vice president of human resources at Climpson Industries sent the following recommendation to the company's president.

"In an effort to improve our employees' productivity, we should implement electronic monitoring of employees' Internet use from their workstations. Employees who use the Internet from their workstations need to be identified and punished if we are to reduce the number of work hours spent on personal or recreational activities, such as shopping or playing games. By installing software to detect employees' Internet use on company computers, we can prevent employees from wasting time, foster a better work ethic at Climpson, and improve our overall profits."

Write a response in which you examine the stated and/or unstated assumptions of the argument. Be sure to explain how the argument depends on these assumptions and what the implications are for the argument if the assumptions prove unwarranted.

The argument with conclusion that the company should implement electronic monitoring of employees' Internet use is weak. It is based on two premises that employees using the Internet need to be identified and punished to reduce the number of work hours spent on personal or recreational activities, and that the company can prevent employees from wasting time and perform better with the aid. However, there are many missed assumptions needed to be stated, as explaining in the following paragraphs.

First, the assumption that there are so many employees using Internet that the business is affected. There are no statement about the quality of these "abuse" is pretty bad, while it may really exist. In fact, there is no guarantee that many workers use Internet during their work time such that they are not concentrating on their work. It deserves some explanation about how bad the problem is.

In addition, it should also be mentioned in the argument that such an "abuse" is extremely affected that result in the decrease of the productivity. It seems really that bad that one of the leaders in the company want to take some action. However, what if he (or she) just see part of the truth? Maybe some of them did the "wicked" doing while most of the workers are diligent in their own work. Even more, the employees who did things seemed similar to it may just search for ideas if the jobs happily require some creativity. It all should be included in the argument in order to make it stronger.

Page 58: Y9 Argument Topics

Furthermore, the crucial point that the company should install the software is that workers will not waste their time on using Internet. However, there lies in a hidden assumption that employees have no other ways to waste their time. Watching TV, reading newspaper, or simply gossiping are all wasting time and, of course, will affect the performance of the company. If it is the case, then strict regulation and management preventing them from doing them should be mentioned.

In the end, the argument is rather weak. However, if assumptions above are explained in the argument, it will be greatly reinforced. Hence, the recommendation from the vice president should not be accepted as truth. To restrict the use of Internet, there ought to be more confirmations.

Argument No.59

The following appeared in a memo from the president of Bower Builders, a company that constructs new homes.

"A nationwide survey reveals that the two most-desired home features are a large family room and a large, well-appointed kitchen. A number of homes in our area built by our competitor Domus Construction have such features and have sold much faster and at significantly higher prices than the national average. To boost sales and profits, we should increase the size of the family rooms and kitchens in all the homes we build and should make state-of-the-art kitchens a standard feature. Moreover, our larger family rooms and kitchens can come at the expense of the dining room, since many of our recent buyers say they do not need a separate dining room for family meals."

Write a response in which you examine the stated and/or unstated assumptions of the argument. Be sure to explain how the argument depends on these assumptions and what the implications are for the argument if the assumptions prove unwarranted.

The speaker argues that UltraClean should be supplied at all hand washing stations throughout the hospital system to prevent serious patient infections. To bolster the conclusion, the speaker suggests a laboratory study of liquid antibacterial hand soaps which has a solution of UltraClean. The speaker also shows a subsequent test of UltraClean at one hospital in Workby, where gave rise to the effect of the solution. It seems so plausible at first glance; nevertheless, there are several weak and porous analyses which lead the argument to the flawed one.

In the first place, the speaker overlooks other variable which could affect the laboratory study. What if, for example, people assigned to the experimental group used the antibacterial hand soaps more frequently that other people who used just normal soaps? In addition, depending on how well each group of people was controlled or treated toward the infection, the result could be different. In order for the argument to be accepted, the speaker should show that the experiment was well controlled enough to rule out the effects of other variables.

In the second place, the argument assumes that a test of UltraClean in a hospital in Workby could represent the whole conditions of other hospitals. However, there is no comparison of other manipulations or controlled conditions between the hospitals in Workby with other areas. It is quite possible that other medical treatment to protect infection was better performed in

Page 59: Y9 Argument Topics

Workby or the environmental conditions such as temperature or moisture was better in Workby during the test period. Without nullifying such a myriad of possibilities which could affect the infection results, the argument cannot be justified.

In the third place, the argument assumes that the whole patients could have the same conditions and the same immune systems toward any infection. However, such assumption is flawed because each individual has different immune system and the treatment for each of them should be differently designed. In addition, the argument overlooks the possibility that other antibacterial soaps or procedure could better work for patients. Since there is no additional analysis about other procedure or chemical products, the argument cannot be accepted.

In conclusion, the argument is biased and not reasonable. To strengthen the argument, the speaker should show the lab study was well conducted enough to reject other variables. In addition, there should be a critical data that the condition of Workby quite well represents that of other hospital in different areas. Moreover, in order for me to accept the argument, the speaker should present more reasonable evidence that the soaps of UltraClean can work for every patient, and there is no better way than using the soaps.

Argument No.59

The following appeared in a memo from the president of Bower Builders, a company that constructs new homes.

"A nationwide survey reveals that the two most desired home features are a bathroom with a whirlpool tub and a large kitchen. Homes in a nearby development built by our competitor, Domus Construction, have whirlpool tubs and have sold much faster and at significantly higher prices than the average. To increase our sales and profits, we should include whirlpool tubs and larger kitchens as standard features in all our new homes. Since our recent buyers have voiced no complaints about small yards, we can also increase profits by decreasing the size of our yards."

In his memo, the president of Bower Builders suggests that they should include whirlpool tubs and large kitchens in all their new homes in order to increase sales and profits. His suggestion comes from the homes built by Domus Construction, which have these features and have sold faster and at prices higher than average. Further he suggests that they could decrease the size of yards since there have been no complaints from buyers about having small yards. The president, however, is mistaken in believing so and should look at the issue using logic and reasoning.

Firstly, the president refers to a nationwide survey that people prefer whirlpool tubs and large kitchens. However, since the scope of the survey is too large, Bower Builders must refer to a more specific survey. They should consider the demand of buyers in the area where they want to build homes. It is likely that in some places, people do not prefer whirlpool tubs and large kitchens. People in some parts of the nation could have a fancy for simpler and old styled homes without these tubs and large kitchens. It is likely that due to the fast paced life of people of a certain area, they do not spend too much time cooking in their kitchens, due to which they would prefer smaller kitchens. The president should keep in mind that even if the general nationwide survey indicates a particular trend, there could be variation in the choice of buyers from place to place.

Page 60: Y9 Argument Topics

The president should not base his suggestion on the experience of another construction company. It could be true that the houses of Domus Construction have sold at higher prices and faster than average. However, it is possible that this is not because they have included whirlpool tubs and large kitchens in their homes. The prices of homes also depend upon the locations. If the homes constructed by Domus Construction are located at a prime location, it is evident that they will be sold fast and at a higher price. Moreover, Domus Construction could be targeting rich people who need luxuries like the whirlpool tubs in their homes. However, this may not be the case with all buyers. The president should think logically about the location of their homes and the category of buyers that would prefer those homes. Accordingly, it is possible that where they are constructing homes, people do not want to invest in large kitchens and whirlpool tubs.

Further, it can be said that selecting one design for all the homes is not a wise decision to be taken by a construction company. Bower Builders should offer variety in the homes that they build with different combinations of features in different houses. As discussed above, the choice of buyers can vary. Some buyers may desire large kitchens but may not want whirlpool tubs, while some may like the opposite.

Lastly, there could be a large number of people who would not like to compromise on the yard space for larger kitchens. In this age where there is hardly any open space, people would desire to have open yards in their homes. If recent customers have voiced no complaints about small yards, it is possible that the builders misinterpret their demand and offer very small yards. Since there is no mention of any specification in terms of area of the yard, it is possible that the builders build smaller yards than those preferred by people. Therefore, the president should not rely on the arbitrary data as is given by him in the argument.

It is evident from the discussion given above that the president does not consider many key issues that need consideration before making such suggestions. The argument given by the president proves to be unreasonable and illogical since it does not take into consideration these important issues.

Argument No.60

The following appeared in a letter from a firm providing investment advice to a client.

"Homes in the northeastern United States, where winters are typically cold, have traditionally used oil as their major fuel for heating. Last year that region experienced 90 days with below-average temperatures, and climate forecasters at Waymarsh University predict that this weather pattern will continue for several more years. Furthermore, many new homes have been built in this region during the past year. Because these developments will certainly result in an increased demand for heating oil, we recommend investment in Consolidated Industries, one of whose major business operations is the retail sale of home heating oil."

Write a response in which you discuss what questions would need to be answered in order to decide whether the recommendation and the argument on which it is based are reasonable. Be sure to explain how the answers to these questions would help to evaluate the recommendation.

The letter from a investment consulting company suggests that given the fact that the period of winter seems longer and people build more houses in the region, investors need to invest on

Page 61: Y9 Argument Topics

Consolidated industries since the retail of home heating oil is involved, which is the necessaries for the residents in northern United States. The argument seems to be convincing and riveting in a cursory reading, but an in-depth analysis reveals several logical fallacies and loopholes that have rendered it untenable.

First, the author, on the basis of the information provided by climate forecasters, argue that the similar weather pattern will repeat in the following few years. However, oftentimes, the report from forecaster is inaccurate and particularly for the long-term prediction. The author need to provide the information whether the report is conducted for several forecasters or the large group discussion. As far as technology concerned, two or three climate forecasters cannot presage accurate reports due to the subjective opinion, which usually ignore inconsequential but essential details. So solely reply on the single forecaster report will result in unpredictable consequence.

Second, the argument assumes that many new homes have been built is equivalent to more people will demand the heating service, which will result in the increase on the consumption of heating oil. Yet author does not provide any convincing evidence that sales of heating oil will be at the rate of upswing as a result of more homes in the region. The author should conduct a survey on those people living in the newly built house to see whether they intend to use the heating. It is likely that people who lived in the new house do not need heating or they are not at home most of time.

Third, Consolidated Industries is a large sector, but home heating oil is just small business, which is part of the industry. It is too impetuous to invest in the market before more warranted information and evidence present. The author need to do more research on the explicit relation between the industry and the home heating oil. Assuming that the forecast is accurate, if the relation was extricate, investors would not have any and significant profits, which invalidate the argument.

In conclusion, the author's inference solely rely on the unproven assumptions and unconvincing evidence, which could be interpreted on the other way around and weaken the argument. To make the recommendation more reliable, it is categorical that the argument is at presence of justified evidence and information other than unwarranted assumptions.

Argument No.61

The following appeared in a memo from a budget planner for the City of Grandview.

"To avoid a budget deficit next year, the City of Grandview must eliminate its funding for the Grandview Symphony. Our citizens are well aware of the fact that while the Grandview Symphony Orchestra was struggling to succeed, our city government promised annual funding to help support its programs. Last year, however, private contributions to the Symphony increased by 200 percent, and attendance at the Symphony's concerts-in-the-park series doubled. The Symphony has also announced an increase in ticket prices for next year. Such developments indicate that the Symphony can now succeed without funding from city government and we can eliminate that expense from next year's budget. This action will surely prevent a budget deficit."

The budget planner for the city of Grandview argues that the government should eliminate its funding for the Grandview Symphony Orchestra in order to evade a budget deficit in the coming

Page 62: Y9 Argument Topics

year. He supports the argument by citing three facts. The first is that the private contributions to Symphony have increased by 200 percent in the past year. The second fact is that the attendance in the Symphony's concerts-in-the-park series had doubled in the past year. The last fact presented by the arguer in support of his recommendation is that Symphony has announced an increase in the ticket prices for next year. The arguer utilizes these three facts to conclude that Symphony does not require funding from the government for its success and therefore, the arguer recommends that the government can stop funding Symphony for avoiding a budget deficit in the coming year. A close scrutiny of the argument reveals that the facts presented do little towards supporting the recommendation made.

The first fact presented by the arguer is largely unsubstantiated as it fails to prove that Symphony is now enjoying unquestionable success. The increase in the private contributions in the past year may be for a variety of reasons which may not occur again in the coming future. There may have been some celebrities or prominent figures that were approached by Symphony for donations and these people might have made some contributions to help Symphony establish itself. There is no guarantee that such private contributions will be made in the future also. Moreover, it is likely that the funding being done by the government is encouraging people to make private contributions in order to assist Symphony in achieving success. Therefore, it is likely that once the government withdraws its financial support, the private contributions also stop as the people will get the indication that Symphony has achieved the desired level of success.

Even if we assume that the private contributions will continue after the government withdraws its support, it is not necessary that the contributions being made will be sufficient for Symphony to sustain itself. Furthermore, the arguer does not mention the amount that has been contributed and the difference that it would make to the functioning and success of Symphony. It is likely that the private contributions being made were negligible in the past and the increase by 200 percent in the past year makes little or no difference to the financial support needed by Symphony for its success. Therefore, the first fact provided by the arguer does nothing to substantiate the recommendation made by the arguer.

The second fact is related to the attendance for Symphony's concerts-in-the-park series. It is likely that this series was conducted in a festive season or was conducted in conjunction with some other popular artists. It is likely that there were some celebrities who were attending or performing in these series and that might explain the increase in attendance. There is no mention of the duration of this series of concerts. It is likely that this series is conducted once in a year and so a high attendance in these concerts may not be able to make a significant difference to the total income earned by Symphony.

The third fact counters the recommendation made, rather than supporting it. If Symphony is planning to increase its ticket prices, then it indicates that Symphony has not been able to sustain itself at the present ticket prices. There is no information pertaining to how the people may react to the increase in ticket prices. If there is a dip in the sale of tickets due to the hike in prices, then the overall effect will be that Symphony will not be able to muster enough financial support for achieving success especially if the government also withdraws its funding.

Even if we assume that the government should withdraw its funding as Symphony is now successful enough, there is no information related to the difference it would make to the budget. It is likely that the amount being provided by the government to Symphony is insignificant when compared with the funds allocated to the city as its annual budget. Therefore, stopping the

Page 63: Y9 Argument Topics

funding of Symphony may not have the desired effect on the budget deficit. Hence, lack of sufficient evidence has rendered the given argument indefensible in several respects.

Argument No.62

The following appeared in a memo from the director of a large group of hospitals.

"In a laboratory study of liquid antibacterial hand soaps, a concentrated solution of UltraClean produced a 40 percent greater reduction in the bacteria population than did the liquid hand soaps currently used in our hospitals. During a subsequent test of UltraClean at our hospital in Workby, that hospital reported significantly fewer cases of patient infection than did any of the other hospitals in our group. Therefore, to prevent serious patient infections, we should supply UltraClean at all hand-washing stations throughout our hospital system."

Write a response in which you examine the stated and/or unstated assumptions of the argument. Be sure to explain how the argument depends on these assumptions and what the implications are for the argument if the assumptions prove unwarranted.

The argument with recommendation that they should supply UltraClean at all hand-washing stations throughout our hospital system is rather weak. It is based on two premises that UltraClean produced a 40 percent greater reduction in the bacteria population than did the liquid hand soaps currently used in a laboratory study, and that hospital in Workby reported significantly fewer cases of patient infection than did any of the other hospitals in our group during a subsequent test of UltraClean. However, there are many missed assumptions, as explained in the following paragraphs.

First, the assumption that the study is trustworthy is needed to be certified. There are a plenty of laboratory study, many in them are concrete and worth credits, while still many are simply fake, produced for confusing the authority and the public. The claimed study must be testified in order to confirm the result of the study, or all the memo will be invalid. In order to strengthen the argument, this assumption should be explained.

In addition, the assumption that, during the claimed subsequent test of UltraClean in Workby, no other hospitals, as comparisons, adopt the policy of using the UltraClean must also be certified. There must be some differece between the tested hospital and the hospitals for comparison, that is the use of UltraClean must be special, say an eccentric doing. If the assumption is further verified, then the result may be trustworthy. If, on the contrary, there are some of the hospitals standing for comparison using it, the reason of the dramatic report must be reevaluated.

Moreover, even the above assumption is confirmed, the assumption that the conditions of all the hospitals, including one in Workby, are the same must be corroborated, as well. The deviation of every situation may result in very different conclusion. For example, if, even without UltraClean, the hospital in Workby is much cleaner than others, say the concentration of bacteria just at least ten percents lower than the others, the conclusion of the report may just be proved to be wrong. So as to fortify the argument, the assumption must be explained in the statement.

In the end, the argument is rather weak. There are three assumption stated above needed to be explained, in order to reinforce the conclusion. Therefore, without the additional aid, there is no way to simply accept it. Leave the memo until further confirmation.

Argument No.63

Page 64: Y9 Argument Topics

The following appeared in a letter to the editor of the Parkville Daily Newspaper.

"Throughout the country last year, as more and more children below the age of nine participated in youth-league softball and soccer, over 40,000 of these young players suffered injuries. When interviewed for a recent study, youth-league softball players in several major cities also reported psychological pressure from coaches and parents to win games. Furthermore, education experts say that long practice sessions for these sports take away time that could be used for academic activities. Since the disadvantages apparently outweigh any advantages, we in Parkville should discontinue organized athletic competition for children under nine."

In this argument, the author has discussed the problem of children under 9 years playing softball and soccer. According to the author, the people of Parkville should not organize athletic competition for children under nine. As per the author, throughout the country, there were over 40,000 of these players, who participated in youth-league softball and soccer last year and they suffered serious injuries. The author's argument looks justified in the first look. However, if we look at the data offered by him to support his argument, it does not seem convincing. A careful study and discussion can reveal that the letter is groundless and has several flaws.

Firstly, the author has given a number of 40,000 children being injured. However, this is incomplete information as nowhere has the author mentioned the number of children who actually played the game. The absolute data provided by the author does not reflect the real seriousness of young players who suffered injuries. If the author wants us to assess the actual situation of young players who suffered injuries, then he should have dished out the ratio of young players injured, not the mere number of young players.

The second evidence produced by the writer is based on a study done in some major cities where these young softball players are facing pressure from their coaches and parents. There is a possibility that these big cities' people are comparatively aggressive in competition, and therefore, they push their children more as compared to the parents from a small city. Hence, their kids are more likely to suffer from mental pressure. Moreover, the result acquired only through studying several major cities cannot represent all cities. Without accurate information, it is unjustified to propose to drop out athletic competitions in Parkville.

Another support sought by the author for his argument is that education experts say that long practice sessions for these sports take away time that could be used for academic activities. Here again the author has not provided any logical evidence. Such sports provide the children some break from their studies that helps them to have more progress in academic courses. Moreover, athletic activities help in the growth of children and hence, can enhance one person's physical condition. Therefore, the author's recommendation of giving up all the athletic competition is obviously groundless.

In conclusion, the letter's recommendation and its evidence are unconvincing. The author has taken the help of an unconvincing study, which cannot be representative of all the cities of the country. To strengthen the letter, the author should have provided some solid information as the ratio of the young players who were injured in Parkville to prove that athletic competition has more disadvantages than advantages.

Argument No.64

Page 65: Y9 Argument Topics

Collectors prize the ancient life-size clay statues of human figures made on Kali Island but have long wondered how Kalinese artists were able to depict bodies with such realistic precision. Since archaeologists have recently discovered molds of human heads and hands on Kali, we can now conclude that the ancient Kalinese artists used molds of actual bodies, not sculpting tools and techniques, to create these statues. This discovery explains why Kalinese miniature statues were abstract and entirely different in style: molds could be used only for life-size sculptures. It also explains why few ancient Kalinese sculpting tools have been found. In light of this discovery, collectors predict that the life-size sculptures will decrease in value while the miniatures increase in value.

Write a response in which you discuss what questions would need to be answered in order to decide whether the prediction and the argument on which it is based are reasonable. Be sure to explain how the answers to these questions would help to evaluate the prediction.

Collectors of ancient sculptures have mentioned that due recent discovery of molds near Kali island, it will have profound effect on prizes if life-sized sculptures. Also due to this, miniatures will increase in value, but this prediction is itself based on number of false assumptions.

Firstly, life-size human statues are found near Kali island by collectors and they have mentioned it as made by Kalinese artists. Questions arises here, How one can confirms same without any further detailed proof. It may be the case that sculptures were made somewhere at another place and after that they have shifted to Kali island. Eventually, if my first point is true then molds of human bodies, which has been detected at Kali may not be used for making statues. So assumption made by collectors doesn't hold validity in both the cases. Even though sculptures are made at Kali, there is no sure indication made by collectors that molds have same size, texture as that of sculptures.

Secondly, false conclusion has been drawn by collectors that, due discovery of molds, Kali artists had not used sculpting tools and technique. To prove this, collector will have to present some more detailed information. Also they have concluded that these molds, used by Kali artist, only meant for sculpture making without any pragmatic proof. Also it may be the case in which molds are made entirely at different era than that of sculptures creation. So collectors discovery in this case doesn't hold validity.

Thirdly, Due to recent discovery, value of life-sized sculpture will decrease and consequently value of miniatures will increase. My first objection to this assumption is, there is no direct relationship mentioned in between miniatures and life-sized sculptures, then how collector can predict this result out of this. Also my second objection is, though discovery of molds can tell us regarding technique used by Kali artist, but that doesn't mean to be worthless as compare to miniatures.

So in nutshell, based on very vague assumptions collectors has predicted their result out of recent discovery. To prove their point they have to come up with more strong point and has to do more research on Kali people and their history.

Argument No.65

Page 66: Y9 Argument Topics

When Stanley Park first opened, it was the largest, most heavily used public park in town. It is still the largest park, but it is no longer heavily used. Video cameras mounted in the park's parking lots last month revealed the park's drop in popularity: the recordings showed an average of only 50 cars per day. In contrast, tiny Carlton Park in the heart of the business district is visited by more than 150 people on a typical weekday. An obvious difference is that Carlton Park, unlike Stanley Park, provides ample seating. Thus, if Stanley Park is ever to be as popular with our citizens as Carlton Park, the town will obviously need to provide more benches, thereby converting some of the unused open areas into spaces suitable for socializing.

Write a response in which you examine the stated and/or unstated assumptions of the argument. Be sure to explain how the argument depends on these assumptions and what the implications are for the argument if the assumptions prove unwarranted.

The argument suggests that the popularity of a park is relevant to whether it has the open area socializing, and by comparing Stanley park to Carlton park, author recommends that the park with more space for socializing has higher popularity. A casual reading of the statement seems convincing, but a careful inspection brings out some loopholes in the claim. The argument is flawed in many places and cannot be accepted as reliable source of study.

First, the argument recommends that Stanley park's popularity, according the number of visiting cars recorded by the video camera, is dropping and by comparing to the number of visitors in the Caltron park is burgeoning. However, author assumes the number of visitors is equivalent to the number of cars and consequently concludes the number of people visiting Staneley park on the basis of the number of cars recorded, whereas it is not the case when it come to the Caltron park. So making a comparison between the two parks does not make any sense. If two hundred people visited Stanley car park, but only fw of them owned the cars, the evidence that author provided is all but nonsense. Furthermore, irrespective of whether there are just fifty people who owned the cars visited the Stanley car park, it is inaccurate solely dependent upon the Video camera to record the number of visitors since cameras are unable to cover every single corners, particularly Stanley car is the biggest the park in the town and so might not be possible for camera to entirely cover the it. So if camera missed recording other hundred visitors, it would invalidated the evidence author provided.

Second, the popularity of a park also depends upon its location. Author mentioned that Caltron park situates in the heart of business district, which logically contributes to the high popularity. On the other hand, the argument does not indicate whether the Stanley park locates in the similar areas. If it located in the outside the urban area, it would, on common sense, have lower popularity comparing to those in the cities. In addition to the location of the parks, author assumes that visitors in the park are for the socials without convincing evidence. But It is likely that people might just coincidently visit the park. In a consequence, these incomplete and unproven assumptions weaken the argument.

In conclusion, had the author provided more concrete evidence other than subjective assumptions, the argument would have been more convincing; Or alternatively, If author had subsumed the assumption with complete and valid evidence, the claim would have been less controversial.

Argument No.66

Page 67: Y9 Argument Topics

The following appeared in a memo from the president of a chain of cheese stores located throughout the United States.

"For many years all the stores in our chain have stocked a wide variety of both domestic and imported cheeses. Last year, however, the five best-selling cheeses at our newest store were all domestic cheddar cheeses from Wisconsin. Furthermore, a recent survey by Cheeses of the World magazine indicates an increasing preference for domestic cheeses among its subscribers. Since our company can reduce expenses by limiting inventory, the best way to improve profits in all of our stores is to discontinue stocking many of our varieties of imported cheese and concentrate primarily on domestic cheeses."

In the given memo, the president of a US based chain of cheese stores recommends that they should stock domestic cheese only and discontinue stocking imported cheese. The president claims that this would lead to reduction of expenses due to the limitation on inventory. The argument is supported by the fact that the five best-selling cheeses in the past year at their newest stores were domestic cheeses. Moreover, a recent survey conducted by Cheese of the World magazine has indicated an increasing trend of consumption of domestic cheeses among its subscribers. A closer analysis of the argument reveals that it suffers from several logical fallacies.

The fact that there has been an increase in sales for domestic cheeses in the newest stores in the past year does not necessarily mean that the remaining stores across the country are experiencing similar preferences from their customers. It is likely that the new stores have been set up in rural areas where the people are used to having domestic cheese. As the sales statistics refer to the past year only, it is likely that in the coming time the sales of imported cheese will also increase once the people get used to their taste. Moreover, it is likely that the locations of the new stores are such that their customers come from humble backgrounds and they cannot afford expensive imported cheeses. Therefore, the arguer unfairly makes a hasty recommendation that all the stores of the company should discontinue stocking many of their varieties of imported cheeses.

Another flaw in the argument is the assumption that the results of the survey conducted by Cheeses of World magazine will hold true for the customers of the company's cheese stores as well. There is no mention of the number of subscribers who are regular customers of the company. Moreover, there is no mention of the location where the survey was conducted. Was it conducted in the towns where the company has its cheese stores? The results of the survey may hold true for the people who were a part of the survey, but the arguer fails to establish a link between the subscribers who participated in the survey and the people who buy domestic cheeses from the company's cheese stores. Therefore, the preferences of the subscribers as reflected by the results of the survey may have little to do with the sales of the imported cheeses offered at the stores of the company across the country.

Even if we assume that there is indeed an increased preference for domestic cheeses among the customers of the company, the arguer needs to provide details of the sales figures which can prove that the action of discontinuing the stocking of imported cheeses would result in profits. It is likely that the sale of imported cheeses brings in much higher profit as compared to the sale of domestic cheeses. Therefore, discontinuing the stocking of imported cheeses may ultimately lead to losses for the company especially since the arguer has failed to prove that the customers of the company prefer domestic cheeses in all the stores throughout the country.

Page 68: Y9 Argument Topics

As the argument does not consider any other alternatives for increasing the profits of the company, the recommendation made by the arguer sounds unreasonable and unconvincing. The argument could have been bolstered with information that could have linked the results of the survey to the preferences of the customers of the company. Moreover, there is a need to provide information on the sales statistics related to imported cheeses and domestic cheeses at all the stores of the company.

Argument No.67

The following appeared as part of a business plan developed by the manager of the Rialto Theater.

"Despite its downtown location, the Rialto Movie Theater, a local institution for five decades, must make big changes or close its doors forever. It should follow the example of the new Apex Theater in the mall outside of town. When the Apex opened last year, it featured a video arcade, plush carpeting and seats, and a state-of-the-art sound system. Furthermore, in a recent survey, over 85 percent of respondents reported that the high price of newly released movies prevents them from going to the movies more than five times per year. Thus, if the Rialto intends to hold on to its share of a decreasing pool of moviegoers, it must offer the same features as Apex."

The manager of Rialto Movie Theater expresses a mandatory need to either make big changes in the theater or to shut it down forever, as a part of a business plan. In addition, he strongly recommends bringing changes in the theater as is done by the newly opened Apex Theater in the mall outside the town.

The argument however appears to be full of flaws from start to end. First of all, there is no substantial reason given by the manager for either making changes or closing the theater for good. The reason for giving such a statement itself is not clear. The measures suggested can be taken up in case the business of Rialto Theatre is incurring financial loss as compared to the previous years or in comparison to other theaters. However, no such point is mentioned by the manager.

Further, there are no details of feedback from the common people over the preference of theater. No dissatisfaction over the present setup of Rialto Theater is expressed by the moviegoers. Not only this, but also there is no favoritism shown towards Apex Theater for its grand interiors and a state-of-the-art sound system. This rejects any need to imitate Apex Theater in terms of its video arcade, plush carpeting and seats etc.

Moreover, such changes made will only add up to the expense of the owners and will most likely end up burdening the movie goers by hike in ticket prices. This will lead to a decrease in the number of people going to Rialto Theater. Subsequently, instead of increasing the returns, these changes may result in financial loss. By raising the price of tickets and emulating the newly built Apex Theater, there is also a risk of Rialto Theater losing its popularity, earned in five decades.

Reports from the recent survey conducted suggest that eighty five percent of respondents are prevented from going frequently to the theaters due to high price of tickets of newly released movies. This indicates a general decline in the number of moviegoers. Moreover, one can not really point out if Apex Theater is running more successfully, as there is no information about the theater drawing more crowds. Similarly, the likelihood of Rialto Theater being visited more frequently can not be ruled out.

Page 69: Y9 Argument Topics

More significantly, Rialto Theater has the advantage of its location and a sound history. It is situated in the city center for almost about five decades. There is a strong probability of Apex Theater being at a disadvantage, as it is situated in a mall outside the town. This may be the reason behind its lavish interior decors in order to attract more crowds. Making the most of the privilege of its suitable location, Rialto Theater should lookout for other modes to attract moviegoers, instead of aping the other theaters and spending unnecessarily.

Conclusively, the argument given by the manager to bring about drastic changes in Rialto Theater is not acceptable due to lack of any sound reasoning.

Argument No.68

The following appeared in the Sherwood Times newspaper.

"A recent study reported that pet owners have longer, healthier lives on average than do people who own no pets. Specifically, dog owners tend to have a lower incidence of heart disease. In light of these findings, Sherwood Hospital should form a partnership with Sherwood Animal Shelter to institute an 'adopt-a-dog' program. The program would encourage dog ownership for patients recovering from heart disease, which will help reduce medical costs by reducing the number of these patients needing ongoing treatment. In addition, the publicity about the program will encourage more people to adopt pets from the shelter, which will reduce the risk of heart disease in the general population."

The above article appeared in the Sherwood Times newspaper. The arguer in the above argument wants to recommend Sherwood Hospital to form a partnership with Sherwood Animal Shelter to institute 'adopt-a-dog 'program. The arguer gives various reasons in support of his argument. The first reason that the arguer gives in favor of his recommendation is a recent study which shows that pet owners have longer, healthier lives than people who do not have pets. The arguer further supports his argument by mentioning that the dog owners have a lower incidence of heart disease. The second argument presented by the arguer in that the program would encourage dog ownership for patients recovering from heart disease, this will help in reducing medical cost by reducing the number of these patients needing ongoing treatment. Further, the arguer states that the publicity from the program will encourage people to adopt pets from the shelter and hence it will reduce the risk of heart disease in the general public. Although the recommendation by the arguer has a noble cause but only by adopting an animal one can not be saved from the ill effects of the disease. The arguments by the arguer are not sufficient for his recommendation to be accepted.

The first argument by the arguer is the recent study which shows that pet owners are healthier people and specially dog owners who tend to have lower incidences of heart diseases. The arguer has not mentioned the exact number of people surveyed as it is very much possible that the number of people who were surveyed was very less and they had pets and did not have any heart problem. It could be possible that the surveyed people comprised more of younger people than older generation as older people often have these problems. The arguer has also not mentioned about any other health problem of the pet owners. It is possible that these people were not having heart problems but other health problems which could instigate heart problems like high blood pressure or high cholesterol. The arguer has also not mentioned whether other pet owners had heart problems or not.

Page 70: Y9 Argument Topics

The second argument presented by the arguer is that the program would encourage people to adopt more dogs and hence they would save on their otherwise ongoing treatment's medical cost. The arguer fails to bring into notice that it is not necessary that all patients can adopt a dog. There are a large number of people who are afraid of dogs or do not prefer pets. This program would not be effective for those patients. The arguer has also failed to mention whether or not any of the patients is allergic to dog fur or not. It is also possible that because of allergies some patients prefer not to adopt dogs.

Finally, the arguer mentions the publicity that the program will get and which would further assure general public adopting more dogs and hence it will reduce their chances of heart attack. Adopting a pet alone can not save anyone from heart diseases. It is regular exercise and good eating habits that matter a lot.

The recommendation by the arguer does not have relevant explanations therefore it should not be accepted.

Argument No.69,70

The following appeared in a memo from a vice president of a large, highly diversified company.

"Ten years ago our company had two new office buildings constructed as regional headquarters for two regions. The buildings were erected by different construction companies—Alpha and Zeta. Although the two buildings had identical floor plans, the building constructed by Zeta cost 30 percent more to build. However, that building'sepenses for maintenance last year were only half those of Alpha's. In addition, the energy consumption of the Zeta building has been lower than that of the Alpha building every year since its construction. Given these data, plus the fact that Zeta has a stable workforce with little employee turnover, we recommend using Zeta rather than Alpha for our new building project, even though Alpha's bid promises lower construction costs."

Write a response in which you discuss what questions would need to be answered in order to decide whether the recommendation and the argument on which it is based are reasonable. Be sure to explain how the answers to these questions would help to evaluate the recommendation.

The argument recommends that the floor that Zeta company build is usually cost-effective and it has stable workforce. A casual reading of recommendation seems convincing, but a careful inspection leads to some loopholes in the argument since the author ignores other interpretations of the assumptions and the evidence given.

First, the author claimed that Zeta company cost thirty percent more than Alpha's company for constructing similar building. However, the argument does not specify the general structure of the buildings which Zeta company and Alpha company construct. If two buildings structure were almost identical, it would be the Zeta company's strategy to lower the price for marketing. Furthermore, Although the construction of a building is attributable to the energy consumption of the building, the number of people working in the building and the efficiency of their work are also contributes to the energy consumption. Author need to be explicit about number of people working in the premise and how efficiency their work is. If there were more people working in the building constructed by Alpha's company, the energy consumption would be higher in the building. And also supposing number of people working in the premises were same, if people

Page 71: Y9 Argument Topics

working in the the building constructed by Zeta company were more aware of efficiently consuming energy, the general energy consumption would, indeed, be lower.

Second, the recommendation indicates the fact that Zeta company has a stable workforce comparing to Alpha's company. However, the argument does not provide any convincing evidence or specify the reason. Zeta company might have stable workforce temporarily. The author should focus on their long-term stability, as well as financial situation, which is the main reason that cause the instability. If Zeta's company were only stable for short-term, its workforce might be unstable during construction for any unforeseen circumstances. Furthermore, it is likely that Alpha's company has trouble in their tight budget that causes unstable workforce, but from long-term prospective, it might have the capability of dealing the situation quickly, which would be more advantageous during the construction.

In conclusion, the argument is mostly based on unproven assumptions and unconvincing evidence and consequently does not form a solid ground for recommendation.

Argument No. 71

The following is a letter to the editor of the Waymarsh Times.

"Traffic here in Waymarsh is becoming a problem. Although just three years ago a state traffic survey showed that the typical driving commuter took 20 minutes to get to work, the commute now takes closer to 40 minutes, according to the survey just completed. Members of the town council already have suggested more road building to address the problem, but as well as being expensive, the new construction will surely disrupt some of our residential neighborhoods. It would be better to follow the example of the nearby city of Garville. Last year Garville implemented a policy that rewards people who share rides to work, giving them coupons for free gas. Pollution levels in Garville have dropped since the policy was implemented, and people from Garville tell me that commuting times have fallen considerably. There is no reason why a policy like Garville's shouldn't work equally well in Waymarsh."

Write a response in which you discuss what specific evidence is needed to evaluate the argument and explain how the evidence would weaken or strengthen the argument.

The letter written to the editor of the Waymarsh Times suggests that a policy similar to that found in Garville can be implemented in Waymarsh to reduce the traffic flow. But author's argument rests upon unstated assumptions and it is filled with lack of evidence to support the quoted facts. Hence, the argument must be thoroughly analyzed before anyone happens to take a clear-cut decision based on the suggestions put forward in this letter to the editor.

First of all, to what extent is the Traffic in Waymarsh is becoming a problem must be established. If the traffic is noted only in the peak hours, then it is a common thing to be noted and hence can be ignored. On the other hand, if the traffic is severe throughout the day, then it must be given serious consideration. Hence the traffic problem must be clearly defined before the author moves on to further argument.

The traffic survey conducted three years ago is compared with the survey conducted recently. It is a faulty analogy because lots of changes might have taken place since the past three years. Therefore, to compare the current status with a status established three years ago is a fault and has to be substantiated with proper reasoning, as of why the survey is compared in this manner.

Page 72: Y9 Argument Topics

Rather than comparison, if extrapolation is being done, then we can interpret the results in a clear manner.

The members of the town council are found to have suggested some road building measures to address the problem. Evidence needs to be given as to whether the suggestions were taken for discussion. Unless, this suggestion is thoroughly discussed, we cannot come to know the pros and cons of it and hence to determine the plan as expensive and going further to state that it will disrupt the existing residential neighborhoods is a overkill. Even if this fact happens to be true, we need proof as to whether the government will be in a proper position to fund the road building and shift the residents to new area in order to create space for roads.

Above all, the author commits the fallacy of comparing two different places and attributing the benefits found in Garville to Waymarsh. In Garville, the people were told to share rides to the work - this might seem as a convincing argument for reduction in traffic. But the information that they were provided with coupons of free gas is a red herring; there is no reason for this information to be introduced in this situation and proper proof needs to be given for how the coupons contributed to reduction in traffic. Furthermore, to assume that reduced pollution levels having been obtained from the policy is not that satisfactory. There can be other reasons for the pollution to get reduced such as the planting of trees along the roads and concern among the people to save the environment etc...

Finally, the views of the people play an important role in determining the success or failure of a policy. Because, the policy of rewarding people to share rides to work went on well in Garville, it does not imply that the same results will be obtained in Waymarsh, Therefore, various factors need to be considered before implementing this policy and questionnaire can be given to people so as to elicit their responses and arrive at a proper conclusion.

All in all, the letter to the editor of the Waymarsh Times is flooded with lots of unverified assumptions lacking evidence like: validity of the survey conducted, traffic caused due to commuters who go to work, implementation of road building, the reward measure having effect on the commuters, success of the Waymarsh policy in Garville taken for granted. They must be thoroughly analyzed before arriving at a conclusion based on the suggestions put forward by the author.

Argument No.72

The following appeared as a letter to the editor of a national newspaper.

"Your recent article on corporate downsizing* in the United States is misleading. The article gives the mistaken impression that many competent workers who lost jobs as a result of downsizing face serious economic hardship, often for years, before finding other suitable employment. But this impression is contradicted by a recent report on the United States economy, which found that since 1992 far more jobs have been created than have been eliminated. The report also demonstrates that many of those who lost their jobs have found new employment. Two-thirds of the newly created jobs have been in industries that tend to pay above-average wages, and the vast majority of these jobs are full-time."

Page 73: Y9 Argument Topics

*Downsizing is the process in which corporations deliberately reduce the number of their employees.

In a letter to the editor of a national newspaper, the author claims to have been misled by an article of corporate downsizing in the United States. According to the letter, the article gives a wrong impression of competent workers facing economic hardship by losing their jobs. The letter finds support in a recent report on the United States economy which demonstrates that since 1992, more jobs have been created than eliminated. The report also suggests that majority of the newly created jobs are full- time and two-thirds of them are paying above average wages.

However, the claims made by the newspaper article and recent report on the United States economy, do not seem to be contradictory to each other. It is possible that the downsizing in United States led to people losing their jobs and the competent ones faced economic hardships. As far as the report on more jobs created is concerned, it is about the situation since 1992. There is every possibility of jobless employees facing a tough time, till 1992.

Maybe, 1992 onwards, more jobs were created by other newly launched companies. Due to growing technology and growing international business, it is possible that many more jobs were available after 1992, especially when compared to the jobs eliminated earlier. There are also chances that after the downsizing, workers had a tough time without jobs but later on they started drawing enough salaries in the new job. This could further be attributed to the improving economy of the nation or due to rise in the global economy standards.

Though, the report of the survey conducted states that those who had lost their jobs had found new employment and were drawing above average wages, no comparison in the financial status of the employees can be made. It is not mentioned whether the employees were drawing much more or less before the corporate downsizing. Moreover, in the new jobs created, wages are mentioned instead of salaries and it is not clear if the workers were drawing monthly salaries or daily wages earlier. Thus, it is likely that even after re-employment the competent workers faced economic hardships and this further nullifies any contradiction in the article on corporate downsizing and economic survey conducted in 1992.

Further, one cannot compare the nature of earlier jobs with the new ones. While the new jobs were majorly full time jobs, nothing is known about the old jobs. Even though in most of the industries above average wages are given, maybe the working hours are much more than the previous jobs.

Since, the detailed finding of the recent report on the United States' economy are not mentioned, one cannot assume that it points out at a good economic state of the workers. In the absence of any such conclusion, there seems to be no contradiction with the facts given in the article on corporate downsizing in United States.

In summary, the article on corporate downsizing can be misleading only if it is contradicted by the economic survey, as referred in the letter. Since, the letter has not provided enough details to arrive at any such conclusion; the argument given in the letter to the editor is not valid.

Argument No.73

"The Mozart School of Music should obviously be the first choice of any music student aware of its reputation. First of all, the Mozart School stresses intensive practice and

Page 74: Y9 Argument Topics

training, so that students typically begin their training at a very young age. Second, the school has ample facilities and up-to-date professional equipment, and its faculty includes some of the most distinguished music teachers in the world. Finally, many Mozart graduates have gone on to be the best known and most highly paid musicians in the nation."

The argument presented above is based on clearly mentioned reasons. It says that the Mozart School of Music has all the qualities of a good school of music because of which music students should choose it over other schools. It lays importance on training since timely and good training leads to good learning. The school not only boasts of having all the latest equipments and numerous facilities for students, it also has the best teachers that you can have. According to the argument, students graduating from this school turn out to be successful by being popular as well as highly paid musicians of the nation. However, the argument cannot be called completely true since there are many situations where it can lose its stand.

First of all, the argument does not consider the presence of other reputed music schools. It can be called a one-sided argument since no reference is made of any comparison with the features of other music schools. It is possible that apart from the Mozart School of Music there are other schools with additional features that are also popular amongst students. In that case, the Mozart School of Music is not the obvious first choice since it will have to face the competition of other reputed music schools.

The argument says that this school stresses on the importance of training and practice, which are very important to begin at a young age. However, it does not mention about the quality of practice and training being imparted. Although there are many distinguished music teachers, it cannot be said that the music education being imparted is of the utmost quality. This is because there is a lot that depends upon the attitude of the faculty as well as students. It is possible that not much is being done in the name of training. If the teachers are not serious about teaching the students, it reflects in the quality of learning and training.

Having ample facilities and latest professional equipment is surely important for any music school to be reputed but it is not the sole factor that can attract students. Equipment can be gotten with funds, but they need to be made the best use of in order to be useful for students. Some colleges are concerned about maintaining the equipment to such an extent that they do not allow the use of expensive and latest equipment by students to avoid damaging them. This type of equipment is of no use and it will not help in making students learn.

The argument says that many students graduating from the Mozart School of Music have turned out to be noted and highly paid musicians. However, this fact could be a mere co-incidence. It is likely that these students could have reached the high in their life by purely their own efforts. In such a case, the school could have taken the opportunity to take the credit of it in order to attract more students.

It can thus be seen that nearly all the statements presented in the argument can have a different side to them. Considering the other possibilities, which are equally possible, it can be said that the Mozart School of Music may not be the first choice of students. Hence, the argument loses its meaning.

Argument No.74

Page 75: Y9 Argument Topics

The president of Grove College has recommended that the college abandon its century-old tradition of all-female education and begin admitting men. Pointing to other all-female colleges that experienced an increase in applications after adopting coeducation, the president argues that coeducation would lead to a significant increase in applications and enrollment. However, the director of the alumnae association opposes the plan. Arguing that all-female education is essential to the very identity of the college, the director cites annual surveys of incoming students in which these students say that the school's all-female status was the primary reason they selected Grove. The director also points to a survey of Grove alumnae in which a majority of respondents strongly favored keeping the college all female.

Write a response in which you discuss what questions would need to be answered in order to decide whether the recommendation and the argument on which it is based are reasonable. Be sure to explain how the answers to these questions would help to evaluate the recommendation.

In this argument, to the future development of the university, the president of the school and the director of the alumnae association held opposite opinions. The president expects a revolution due to the university is all-female school at present and an increasing in applications if male students are allowed to apply too. However, the director assumes this action will stain the school's old image and offend the majority members of alumnae association. Before hastily following the ideas of either of the both, there are several questions needed to be answered.

By citing the examples of other all-female universities which implement the new policy of including male students and stimulate the number of applications, the president deems that enacting the same policy will benefit Grove College too. The question here is, however, whether Grove College can be compared with those universities. Because the author does not provide any information of those schools even Grove College's. Perhaps the same action should be ineffective on Grove College since their disparate background details such as the historical rate of applications and quality of education. For example, due to the high quality of education and advanced facilities of Grove College, its historical rate of application is rather high and staying stable, while other universities may confront financial problems because of their low applications and then a reformation is quite essential to them. If true, there is no need for Grove College to change its basic policy to follow others' trend which may does not fit itself. Therefore, before offering more convincing information about the similarities of those schools, the president cannot persuade his/her rival.

The president's rival-the director cites two surveys to undermine the president's ideas. According to the first survey of incoming students who say the all-female status is the initial reason why they choose GC, the director deems that the changing policy will impair the college's old tradition and very identity. At first glance, the assumption seems quite reasonable; however, the survey fails to provide the respondents' size whether they can be the representatives of whole students. Besides, whether those respondents' reply can be trusted is also suspect able because there are possibilities that the students did not express their very thoughts if they got their parents' companies around at that time, or that the students tried to impress the professors of the school to get accepted. Either scenario, if true, would serve to undermine the director's own claim.

Furthermore, based on the latter survey of Grove alumnae who strongly support keeping the school's old tradition. However, the survey's reliability is highly doubted due to the investigator is the director who is the leader of the Grove alumnae association. Those respondents who are

Page 76: Y9 Argument Topics

afraid to offend their leader may not offer their truthful opinions. In addition, it is quite possible that for supporting his/her own ideas, the director only select part of the results of the survey which may favor his/her ideas. Unless, the director can provide other objective and sounded survey, the opinion hold by him/her can not convince the president to drop the initial thoughts.

To sum up, there are questions for each of the person to answer due to neither of them provides reliable and persuasive evidence which can thoroughly illustrate the present condition of the college. Although their concerns about the school should be appreciated, other methods which can enhance the applications of school should be considered such as hiring famous professors and updating technological facilities. Only after well-rounded considering and unbiased discussing, the best policy will be appeared.

Argument No.75

The following appeared in a letter to the editor of a Batavia newspaper.

"The department of agriculture in Batavia reports that the number of dairy farms throughout the country is now 25 percent greater than it was 10 years ago. During this same time period, however, the price of milk at the local Excello Food Market has increased from $1.50 to over $3.00 per gallon. To prevent farmers from continuing to receive excessive profits on an apparently increased supply of milk, the Batavia government should begin to regulate retail milk prices. Such regulation is necessary to ensure both lower prices and an adequate supply of milk for consumers."

The given argument highlights the fact that the price of milk at the local Excello Food Market has increased from $1.50 to $3.00 per gallon despite the report given by the department of agriculture in Batavia that the number of dairy farms across the country has increased by 25% in the past 10 years. The arguer recommends that the Batavia government should regulate retail milk prices for three reasons. The reasons quoted by the arguer are that this regulation will prevent farmers from gaining excessive profits from an increased supply of milk, lower prices will be ensured and there will be an adequate supply of milk for the consumers. The argument stands unconvincing despite the evidence provided in its support as the evidence in itself is highly questionable.

A major stumbling block for the argument is the assumption that the milk prices of the local Excello Food Market are an indication of the milk prices throughout Batavia. The relationship between the milk prices throughout Batavia and those at the Excello Food Market has not been explicitly established by the argument. Hence, the contention that the Batavia government needs to regulate its retail milk prices does not sound convincing enough. Moreover, the arguer does not present any evidence that proves that the profits from the sale of milk are higher than those accrued from the sale of other products. It is likely that whatever profits that are being accrued by the farmers are being contradicted by the cost involved in producing and distributing the milk. Therefore, the assumption that the farmers are receiving excessive profits from the sale of milk is baseless. Hence, the reason quoted by the arguer for the regulation of milk prices does not stand on firm ground.

The assumption, that an increase in the number of dairy farms across the nation implies that the milk supply has also increased, fails to convince the reader. It is not necessary that a major percentage of the new dairy farms are located in Batavia. Even if there are a substantial number of new diary farms in Batavia, it is not necessary that this has led to an increase in the milk supply. This is because the new dairy farms may be equipped with machinery for the production

Page 77: Y9 Argument Topics

of other dairy products like cheese and milk cream. Hence, the supply of milk to the population of Batavia may not be affected by a great margin despite the increase in the number of dairy farms.

Finally, the arguer overlooks the fact that imposing a regulation on the milk prices would lead to the farmers trying out other profit making businesses. They would not like to run a business which gets them less profits. Therefore, the assumption that the regulation would ultimately result in an adequate milk supply for consumers does not sound convincing.

The argument could have been substantiated with the presence of clear evidence that proved the link between the Excello milk prices and an increase in the number of dairy farms across the country. Moreover, there is a requirement of strong evidence that proves beyond doubt that the proposed regulation would indeed result in an adequate milk supply for consumers. To sum up, one can say that the conclusion arrived at by the argument is not supported by adequate evidence and hence, it fails to convince the reader that the recommendation made by the arguer is justified.

Argument No.76

The following appeared in a newsletter offering advice to investors.

"Over 80 percent of the respondents to a recent survey indicated a desire to reduce their intake of foods containing fats and cholesterol, and today low-fat products abound in many food stores. Since many of the food products currently marketed by Old Dairy Industries are high in fat and cholesterol, the company's sales are likely to diminish greatly and their profits will no doubt decrease. We therefore advise Old Dairy stockholders to sell their shares and other investors not to purchase stock in this company."

The given argument advises the stockholders of Old Dairy Industries to sell their shares and it recommends that investors should not purchase shares of this company as it is likely to face a sharp dip in sales and profit. The arguer has based his recommendation on evidence that highlights the desire of 80 percent of the participants of a recent survey that they would like to reduce their intake of high fat foods. The arguer adds that most of the food products marketed by Old Dairy are high in their fat and cholesterol content and low-fat products are easily available in most of the other food stores. Using the above mentioned assumptions, the arguer concludes that the sales and subsequently the profits of Old Dairy are going to reduce. However, the argument lacks critical information, the absence of which has rendered the argument indefensible.

Firstly, the results of the survey clearly indicate that 80 percent of the participants have expressed a desire to reduce their intake of food products with high-fat content. There is a wide difference between expressing a desire to do something and actually executing it on ground. It is likely that these 80 percent are still purchasing food products that are high in fat and cholesterol. The results of the survey have not included information related to the percentage of people who have already restricted their intake of food products with high-fat content. Moreover, the results of the survey cannot describe the opinion of the entire population as there is no description of the section of the people who were a part of this survey. It is likely that the survey was conducted among the older generation of people and therefore, 80 percent of them expressed their concern for restricting their diet. It is likely that a majority of the younger generation of people still likes to consume products that are high in fat content and they have no intention of

Page 78: Y9 Argument Topics

switching to low-fat food products. Therefore, the findings of the survey fail to substantiate the recommendation made by the arguer.

The second evidence presented by the arguer refers to the abundant stock of low-fat products in food stores. It is not necessary that this stock indicates a requirement of low-fat products. On the contrary, it is likely that there are diminished sales of low-fat food products and therefore, their stocks are not depleting in the food stores.

The argument does not provide concrete evidence to support the contention that the company will face severe losses due to declining sales. Even if the company does face some form of decline in sales, it might continue to make profit. This is because the profits of a company are calculated on the basis of the investment made in the business and the cost of running the business coupled with its maintenance. It is likely that Old Dairy Industries is an old establishment and it has already recovered its initial investments. Additionally, the arguer does not make a mention of the profit margins of the company. Therefore, the assumption that people are keen on avoiding the consumption of high-fat products may not affect either the sales or the profits of Old Dairy at all.

The argument could have been strengthened with additional information that proved a link between the results of the survey and the opinion of the entire population and their subsequent action on ground. Moreover, the argument lacks information pertaining to the sale and profit statistics of Old Dairy. Hence, the advice being given by the arguer to the stockholders of Old Dairy and other investors should be completely ignored as it is grossly flawed.

Argument No.77

The following recommendation appeared in a memo from the mayor of the town of Hopewell.

"Two years ago, the nearby town of Ocean View built a new municipal golf course and resort hotel. During the past two years, tourism in Ocean View has increased, new businesses have opened there, and Ocean View's tax revenues have risen by 30 percent. Therefore, the best way to improve Hopewell's economy—and generate additional tax revenues—is to build a golf course and resort hotel similar to those in Ocean View."

Write a response in which you examine the stated and/or unstated assumptions of the argument. Be sure to explain how the argument depends on these assumptions and what the implications are for the argument if the assumptions prove unwarranted.

The argument with conclusion that Hopewell should build a golf course and resort hotel is weak. It is based on the premise that the nearby town of Ocean View built them two years ago, and that tourism in Ocean View has increased, new businesses have opened there, and its tax revenues have risen by 30 percent during the past two years. However, there are many missed assumptions needed to be stated, as explained in the following paragraphs.

First, the assumption that the environment of Hopewell suits to the golf course. As the requirement of building a golf course, the circumstances of Hopewell should have a wide-range of grassland, some smooth hills, and less trees. The success of a golf course relies greatly on these environmental conditions. If the assumption is proved to be true, then there is a high

Page 79: Y9 Argument Topics

hence the development will result in profit. However, if it fails to fall in the category, the recommendation will be nullified. Although how crucial it is, it is hidden in the argument.

Furthermore, in order to encourage tourists to travel there, there is still a missed assumption that people will take a vocation there because the entertainment of golf activities. On the contrary, people may not go there for playing golf. For example, there is no guarantee that the town called Hopewell is not well-known for a huge, magnificent shopping mall. If it is the case, which may make the assumption unwarranted, the tourists might go there simply for enjoying shopping. If it is happily the case, the conclusion will be wrong.

In addition, there still are many factors of the thrive of the business, say, the assumption that the traffic is good and may attract a lot of tourists. Providing that the traffic situation is so bad, say the roads are rough and tortuous, that results in the scanty of travelers as well as visitors. In this case, the improvement of facility such as a golf course and a resort hotel won't help increase the tourism, leaving the waste of budget and probably dilapidated building to come.

In the end, the argument is rather weak. However, if the assumptions mentioned above are certified, it will be greatly strengthened. Therefore, there is no way to accept the recommendation. Stop the action until further demonstration.

Argument No.78

The following appeared in a memo from the vice president of a food distribution company with food storage warehouses in several cities.

"Recently, we signed a contract with the Fly-Away Pest Control Company to provide pest control services at our fast-food warehouse in Palm City, but last month we discovered that over $20,000 worth of food there had been destroyed by pest damage. Meanwhile, the Buzzoff Pest Control Company, which we have used for many years, continued to service our warehouse in Wintervale, and last month only $10,000 worth of the food stored there had been destroyed by pest damage. Even though the price charged by Fly-Away is considerably lower, our best means of saving money is to return to Buzzoff for all our pest control services."

Write a response in which you discuss what specific evidence is needed to evaluate the argument and explain how the evidence would weaken or strengthen the argument.

The argument with recommendation that the company should return to Buzzoff for all pest control services is rather weak. It is based on the premises that the damage in warehouse in Palm City is over $20,000 at which the Fly-Away Pest Control Company provide services, that the damage in warehouse in Wintervale which is under control of the Buzzoff Pest Control Company is just $10,000,and that the price charged by Fly-Away is considerably lower. However, there is many missed evidences, as explained in the following paragraphs.

First, the evidence that there are no specific environmental situations that causes the damage in Palm City is needed to be verified. The special conditions may possibly be the result of higher cost in food damage, leaving the comparison in the statement about which company has better performance in doubt. For example, there are swarms of locusts in Palm City and the damage over $20,000 is the lowest in the proximate of the area, making the Fly-Away Pest Control Company free of blame. On the contrary, maybe there is happen some policy of the local

Page 80: Y9 Argument Topics

government in Winter vale that almost all the pest is killed with pesticide. Then the efficiency of the Buzzoff is worth suspicion.

Furthermore, the evidence that the rate of damage comparing to the whole volume of the storage is low in Winter vale should be certified, providing the previous evidence is proved. The evidence is aimed to evaluate the real performance of the two pest-control Companies. For example, if the total value in the warehouse in Palm City is one million dollars, the damage rate is just 2 percent, comparing to the rate of 100 percent if the storage in Winter vale happen is $10,000. As a result, the conclusion is shaky, leaving the motives of the vice president in question. The evidence is so crucial that needs to be testified.

Moreover, the evidence that the price comparing to the damage in Palm City is so high that the return to Buzzoff Pest Control Company saves money ought to be demonstrated. If it is the case, say the price of adopting Fly-Away Pest Control Company is just $5,000 lower with the damage $10,000 higher, then the recommendation might be sensible. On the other hand, if the result is contrary, that is, if the cost of adopting of Fly-Away Pest Control Company is $50,000 lower ,then the return is unreasonable leaving the recommendation invalid.

Argument No.79

The following is a memorandum issued by the publisher of a newsmagazine, Newsbeat, in the country of Dinn.

"Our poorest-selling magazine issues over the past three years were those that featured international news stories on their front covers. Over the same period, competing news-magazines have significantly decreased the number of cover stories that they devote to international news. Moreover, the cost of maintaining our foreign bureaus to report on international news is increasing. Therefore, we should decrease our emphasis on international news and refrain from displaying such stories on our magazine covers."

The memorandum issued by the publisher of Newsbeat suggests reducing emphasis on the publishing of international news in order increase its sale. This suggestion is based on the observation that the magazine featuring international news on its cover page has seen poor sales in the past three years. On the other hand, other competitive magazines have decreased reporting of international news on the cover page at the same period of time. Taking a clue from them, the publisher suggests cutting down on their international news. Moreover, it has become expensive to maintain foreign bureaus for reporting international news and it is best to decrease the importance given to international news and cut the extra cost.

This is, however, not a wisely thought over suggestion. First of all, the publisher has not thought logically over the reason behind the decrease in sale of the magazine in the last three years. In the argument it is said that the issues that feature international news on the front page have seen poorest-sales. However, no mention is made of the type of news that was published. The sale of a magazine depends upon the weight a piece of news carries. If there has not been any international activity worth the attention of the readers, it is foolish to put it on the cover page. This magazine will obviously see poor sales. On the other hand, if the international news is so big that it overshadows all other news stories, it needs to be on the covers. The memorandum, however, does not mention the nature of the news stories published in the magazines.

Page 81: Y9 Argument Topics

There is another way of looking at the reduced sale of these issues of the magazine. If there has been a national event that has gathered a lot of attention, the readers will like to read more about that event. In such a case, if a magazine ignores the event and focuses on an international story, it will surely not be sold. The observation made in the argument does not consider this aspect and thus the reason behind the reduced sale of international news cannot be confirmed.

The argument also mentions that over the same period of time other competitive magazines have decreased the number of international stories in their issues. Whether this is because they have also experienced a decrease in their sales or for some other reason is not mentioned in the argument. It is likely that other magazines are not able to bear the cost of supporting an international bureau or they have changed their strategy to focus on a particular class of readers. This decision cannot be equated to the decrease in sale of issues of Newsbeat. It is also not known if this change has benefited these magazines in terms of increase in sales or not. Moreover, what has proved good for one magazine may not prove good for another. If other magazines have gained from reducing their international news it may not be the same for Newsbeat as well.

Lastly, the memorandum says that there is an increase in the cost of maintaining foreign bureaus and it would be best to reduce the emphasis given to international news. This can be a way of reducing the cost of publishing the magazine, but it is not a solution of the decrease in sales. Instead of reducing the cost of production, it is better to focus on the quality of news reports being published by the magazine. After the above discussion, the memorandum presented by the publisher seems lacking in logical reasoning and surely does not present a sure shot solution of the problem.

Argument No.80

The following is taken from a memo from the advertising director of the Super Screen Movie Production Company.

"According to a recent report from our marketing department, during the past year, fewer people attended Super Screen-produced movies than in any other year. And yet the percentage of positive reviews by movie reviewers about specific Super Screen movies actually increased during the past year. Clearly, the contents of these reviews are not reaching enough of our prospective viewers. Thus, the problem lies not with the quality of our movies but with the public's lack of awareness that movies of good quality are available. Super Screen should therefore allocate a greater share of its budget next year to reaching the public through advertising."

Write a response in which you discuss what questions would need to be answered in order to decide whether the recommendation and the argument on which it is based are reasonable. Be sure to explain how the answers to these questions would help to evaluate the recommendation.

The memo enumerates that despite the increasing number of rave reviews by movie reviewers, the number of attendees of Super Screen produced movies are on the decline. The memo concludes that this is due to the lack of public awareness and suggests that increasing the budget for advertising can reverse the trend.

The argument though has a few flaws. Firstly it states that there is a decline of number of people attending the Super Screen produced movies. It does not, however, mention the other sources through which a viewer can watch movies. It does not note the effect of the modern filmology

Page 82: Y9 Argument Topics

where there are varied sources where people can watch a movie. Also it does not take into account the condition of the Super Screen theaters, which might have also been the reason for the declining trend.

Secondly, the memo elucidates that the increase in the number of positive reviews for the movies does not reach the attention of the viewers. It does not however take into account the genre of the movies being screened. The movies screened might be appreciate by a select few, however well made the movie is. These movies might not have met the tastes of the greater part of the population. It is plausible that viewers might not have found real interest in these movies.

Next the director does not think the timing of the movie releases can affect the number of viewers for movies. The movies might have been released at a time when most of them do not have the inclination or interest to do so. Often the blockbuster movies coincide with the holiday seasons or the festive seasons. Also the timing of the screening of the movies might have coincided with some calamity or the other. The director however makes no mention of it.

Finally the memo have suggested that the advertising budget be increased that would spread the movies to a larger audience and thus attract more viewers. The argument however does not necessary hold as advertising is not the only means by which viewers tend to know about the movies. There are at times such ads can actually drive away potential viewers. It makes no claim that other means of spread of information can help attract viewers.

The director could have strengthened the case by collecting feedback from the viewers. The director could also have talked in general about the genre of the movies that are released. As it stands, though, the memo has its share of pitfalls and argument made by the director does not hold completely.

Argument No.81

The following appeared in a business magazine.

"As a result of numerous complaints of dizziness and nausea on the part of consumers of Promofoods tuna, the company requested that eight million cans of its tuna be returned for testing. Promofoods concluded that the canned tuna did not, after all, pose a health risk. This conclusion is based on tests performed on samples of the recalled cans by chemists from Promofoods; the chemists found that of the eight food chemicals most commonly blamed for causing symptoms of dizziness and nausea, five were not found in any of the tested cans. The chemists did find small amounts of the three remaining suspected chemicals but pointed out that these occur naturally in all canned foods."

Write a response in which you discuss what questions would need to be addressed in order to decide whether the conclusion and the argument on which it is based are reasonable. Be sure to explain how the answers to the questions would help to evaluate the conclusion.

In this editorial, the author argues that the Promfoods which result in part of consumers of tuna feel dizziness and nausea did not pose a health risk after testing. To support the conclusion the writer points out that the conclusion is based on tests performed on samples of the recalled cans by chemists from Promofoods and only find small amounts of the three remaining suspected chemicals which could cause symptoms of dizziness and nausea and they occur naturally in all

Page 83: Y9 Argument Topics

canned foods. However the argument suffers from several flaws and is therefore unconvincing as it stands, albeit it may appear plausible at a cursory glance.

First, there are problems about the representativeness and the randomness of the survey’s sample. The author should provide the information about the sample, we don’t know where are the cans from and when do they product, perhaps the sample comes from nationwide or just only a local, the two statistics does not necessarily apply to each other, so the survey results are not reliable. Hence, without accounting for as well as ruling out other likely factors, the author could not assumes that the canned tuna did not pose a health risk.

Secondly, even if five of the eight common food chemicals which cause symptoms of dizziness and nausea were not found in any of the tested cans but it does not means there are only eight food chemicals can result in the symptoms of dizziness and nausea, perhaps the device of testing is not advanced enough to find out other potential food chemicals which may cause the same symptoms. So the company should pay attention as well as deal with testing the canned tuna if they want to substantiate that there is no health risk with the canned tuna. With the evidence of ruling out other likely factors could writer bolster his assumption.

Thirdly, the author cites that the chemists did find small amounts of the three remaining suspected chemicals but pointed out that these occur naturally in all canned foods. Small amounts does not necessarily indicate zero and the author did not say what is the degree about small amounts, we can regard 0.1 percent as small amounts and we also regard 0.01 percent as small amounts, so the point is how much can cause people sick, so is it open to a number of interpretations, without compelling evidence to support, the arguer’s conclusion is not worthy of consideration. Providing more information about the results of testing the writer could substantiate his assumption.

In retrospect, further investigation and analysis are needed, if the argument had included the given factors discussed above, it would have more thorough and logically acceptable.

Argument No.82

The following appeared in a memo from the vice president of marketing at Dura-Socks, Inc.

"A recent study of Dura-Socks customers suggests that our company is wasting the money it spends on its patented Endure manufacturing process, which ensures that our socks are strong enough to last for two years. We have always advertised our use of the Endure process, but the new study shows that despite the socks' durability, our customers, on average, actually purchase new Dura-Socks every three months. Furthermore, customers surveyed in our largest market—northeastern United States cities—say that they most value Dura-Socks' stylish appearance and availability in many colors. These findings suggest that we can increase our profits by discontinuing use of the Endure manufacturing process."

The argument put forward by the vice president of marketing at Dura-Socks Inc., in order to discontinue the current Endure manufacturing process owing to their less durability is cast with serious assumptions. The recommendation further assumes that on discontinuing the Endure manufacturing process used to manufacture socks, we can increase the profit. But how this

Page 84: Y9 Argument Topics

actually happens is of doubt and several questions need to be answered before arriving at a conclusion based on the recommendation by the vice president.

First of all, the time during which the study was conducted is not clearly mentioned. If the study had been conducted years before, then it is least assured that the same effects will continue till now. Therefore, the exact time during which the study was conducted must be ascertained in order to proceed further with the argument. Also, the group of customers who suggest that the company is wasting money on Endure manufacturing process must be thoroughly scrutinized and they must be provided with a questionnaire so that they can ascertain their side and prove their point.

Secondly, the Endure manufacturing process is quoted to be patented; then how come, will there be any doubt of following the process? This question must be analyzed in great detail because unless a process is of vital importance and providing great benefits, it will least likely be patented. Then how come a patented process provide negative effects? Moreover, the Endure process is said to be advertised to the people and the people are found to purchase the socks once in every three months. But how come these two processes are interlinked with the notion of "durability" must be clearly explained. If, the people buy socks once in three months, does it necessarily imply that the durability of the socks has gone worst? It can be due to numerous other reasons like the need of more number of same kind of socks due to high demand, increase in population leading to the increase in number of socks etc...

The author mentions about a survey conducted in the popular market of Dura Socks. But questions like: How many customers were examined? What form of questionnaire was used to elicit information from them? Does preference for stylish appearance imply that there can be increase in the sales of socks? Moreover, there might be a case that the profits incurred are solely due to the usability of the product. In this case, the color and aesthetic nature of the socks becomes a second nature and of least importance. Hence these factors must be examined before deciding that color and variety of socks will surely lead to increase in profits.

The Endure manufacturing process is said to have been patented and passed a large number of rigorous testing stages in order to be eligible for use. It also ensures that the socks will remain good for two years. Hence reverting to a new process altogether will do more harm than good. What effects will the discontinuing of process have on the sales of Dura Socks? Will there be additional production methods available which outweigh the benefits of Endure manufacturing process? These points need to be considered before discontinuing the Endure manufacturing process.

Hence the argument put forward in the memo from the Vice president of Dura Socks Inc., leads to several questions like: the time during which the study is performed, validity of the study, the nature of patented process, durability playing a role in purchasing socks frequently, nature of survey conducted to examine the preference of customers, existence of other processes similar in nature and advantageous than Endure manufacturing process. These questions must be answered properly. Otherwise, it will go a long way in impeding the progress of the company and taking the company towards negative growth. Therefore, the words of the Vice President must be analyzed properly in order to decide whether they have to be considered for implementation.

Argument No.85 (4 star)

In a study of the reading habits of Waymarsh citizens conducted by the University of Waymarsh, most respondents said that they preferred literary classics as reading

Page 85: Y9 Argument Topics

material. However, a second study conducted by the same researchers found that the type of book most frequently checked out of each of the public libraries in Waymarsh was the mystery novel. Therefore, it can be concluded that the respondents in the first study had misrepresented their reading habits.

Write a response in which you discuss what specific evidence is needed to evaluate the argument and explain how the evidence would weaken or strengthen the argument.

The author argues that, since the results of the two studies about reading preferences conducted in Waymarsh are contradictory, the respondents of the first study had misrepresented their reading habits. Though his claim may well have merit, the author presents a poorly reasoned argument, based on several questionable premises and assumptions.

Firstly, the author fails to provide details about the study conducted by the University of Waymarsh. The types of questions asked, whether choices were given for replies, or the respondent was required to write his own answer, the number of questions asked, the mode of survey whether oral, written or online and whether Waymarsh’s reading community was adequately represented are left to the reader’s imagination. The survey could have included specific answer choices, forcing the respondent to select one among them and thereby not effectively gathering data from the respondents. Likewise, a written survey is likely to have attracted only participants from an older age group since youngsters would such tasks unworthy of their time.

Furthermore, the author mentions that the second study indicated that the book most frequently checked out of the public libraries belonged to the mystery genre. It is entirely possible that the public libraries had a very limited collection of literary classics, thereby urging the readers to prefer mastery novels.

Additionally, high borrowing charges for the classics section, would have swayed the readers to the mystery section. The author would have done well to provide evidence to prove that the members of the public libraries preferred the mystery novels to literary classics as the first study indicated.

In conclusion, the author presents an unconvincing argument. Minutiae regarding the two studies conducted and the availability of books in the public libraries would help one to evaluate his argument.

Argument No.86,89

The following appeared in a memo at XYZ company.

When XYZ lays off employees, it pays Delany Personnel Firm to offer those employees assistance in creating résumés and developing interviewing skills, if they so desire. Laid-off employees have benefited greatly from Delany's services: last year those who used Delany found jobs much more quickly than did those who did not. Recently, it has been proposed that we use the less expensive Walsh Personnel Firm in place of Delany. This would be a mistake because eight years ago, when XYZ was using Walsh, only half of the workers we laid off at that time found jobs within a year. Moreover, Delany is clearly superior, as evidenced by its bigger staff and larger number of

Page 86: Y9 Argument Topics

branch offices. After all, last year Delany's clients took an average of six months to find jobs, whereas Walsh's clients took nine."

Write a response in which you examine the stated and/or unstated assumptions of the argument. Be sure to explain how the argument depends on these assumptions and what the implications are for the argument if the assumptions prove unwarranted.

Although it is very considerate of XYZ company to concern so much about its laid-off employees on their next job offer and cautiously choose the personnel firm to help them polish materials on job hunting, however, the author's recommendation is not persuasive due to several unwarranted assumption it made.

First, the author asserts that a personnel firm is necessary to help employees to find a job and only fabulous resumes and interviewing skills can attract the boss but fails to recognize that personal talents and uniqueness are decisive factors whether people can be enrolled. Sometimes, those personnel firms can not put extra energy or effort on certain people due to their big volume of work, so most of the application materials made by them seem alike. Employers may look through hundred of CV each day, then it is not hard to imagine what kind of materials may catch their eyes. Therefore, XYZ company may considerate arrange some extra consultant to offer some advice to those employees rather than just throw them to personnel firms.

Second, according to the recommendation, Delany Personnel Firm seems a better choice for XYZ company due to its efficiency on searching jobs last year. However, there is potential flaw here. It is only a one year record which can not strongly prove the company can offer the same high quality of service through other years unless the job marketing and personal factors of employees remain the same. The reason of its outstanding achievement this year may result from the high demand of workers in society or most of the laid-off employees have special talents can meet the vacancy. Unless the author rules out all these possible causes, DPF can be fairly substantiate a qualified and smart choice for them.

Finally, for further proving the DPF can offer a perfect service, the author unfairly compares it with another firm called Walsh Personnel Firm. Without any detail information of each company's staff, the author jumps into the conclusion that bigger size of staff and numerous branch offices can guarantee a better service. However, whether staff being qualified or experienced are not due to the firms sizes but personal talents and knowledge and WPF maybe a small-sized company with excellent employees who have more experiences on their field. In addition, the author put out another evidence to prove DPF's superiority that when XYZ hired WPF eight years ago, it found jobs for half of the laid-off workers within a year which can prove its low efficiency. However, The author fails to consider the different conditions between that year and last year and also the exact percentage of the successful cases of each company. For example, eight years ago there may economic crisis happened and demand of labors might very low but the percentage of successful cases achieved by WPF might even higher than DPF's last year. Therefore, it may unfair for the author to make such conclusion based on such fallible evidence.

To sum up, after examining all the factors involved intend to convince XYZ company to hire DPF on assisting its laid-off workers find new jobs, the author fails to consider hidden flaws behind the assumptions. Hence, to strengthen the recommendation, the author should set up a complete

Page 87: Y9 Argument Topics

research on every possible causes. Or the generosity of the company may be diminished by fallible decision.

Argument No. 88, 90

The following appeared in a memorandum written by the vice president of Nature's Way, a chain of stores selling health food and other health-related products.

"Previous experience has shown that our stores are most profitable in areas where residents are highly concerned with leading healthy lives. We should therefore build our next new store in Plainsville, which has many such residents. Plainsville merchants report that sales of running shoes and exercise clothing are at all-time highs. The local health club, which nearly closed five years ago due to lack of business, has more members than ever, and the weight training and aerobics classes are always full. We can even anticipate a new generation of customers: Plainsville's schoolchildren are required to participate in a 'fitness for life' program, which emphasizes the benefits of regular exercise at an early age."

The author of this argument avers that Nature's Way (NW), a chain of stores selling health food and other health related products, would profit successfully in the town of Plainsville. His conclusion is based on the assumption that Plainsville is clearly an area where people lead healthy lives. The apparent facts like- all time high sale of running shoes and exercise clothing, local health clubs having more members, other health classes being full and Plainsville school children being potential new customers, make him conclude that Plainsville is indeed a jackpot for NW. However, I disagree with the author because I can clearly see that his assumptions are erroneous. There is no evidence to prove what he avers.

Firstly, how can the author claim that the town of Plainsvelle is clearly a heath conscious area when there are no statistics to prove the same? The author has not surveyed the area to know their health lifestyle. He as not surveyed the town residents and school children if they are indeed interested in Nature Way's health products. Hence, he is terribly wrong to think NW will profit in Plainsville.

Secondly, the sales of running shoes and exercise clothing being on a all time high has got nothing to do with fitness. The shoes may be more comfortable than heels, hence women might be buying them.. The exercise clothes maybe on sale and hence they must be selling fast. Moreover, the exercise clothing could be on a mega sale because they weren't able to sell it before because of their unpopularity in Plainsville. The author has not considered these factors.

The local health club having more members may be because it is offering a huge discount. Again, having a health club membership does not mean that the residents actually are going there regularly. They maybe too lethargic to go even after taking a membership.

Also, the weight training and aerobics class being full does not go on to prove Plainsville residents are into fitnes. These classes may be rare and residents from other towns maybe visiting and hence they must be full. The author must find out what percent of Plainsvilee residents actually are a part of this class.

The assumption that school children would be the new generation of potential customers is baseless. School children might get dissuaded from fitness because they are coerced into this fit-

Page 88: Y9 Argument Topics

for-life program. Even if they develop an interest, it is their parents who should be interested in buying NW's products. The school children have no say in that.

Most importantly, even if we assume that Plainsville residents are into fitness. it does not go on to prove they will buy NW's health food..They may not be interested in NW's health related products either. In fact, maybe NW has an competitor in that area and Plainsville residents could favor the competitor. In such a case NW would never profit in Plainsville.

In conclusion, the author wrongly assumes a number of facts and I disagree with his conclusions. He must provide more evidence. He must create valid proof by holding surveys to convince me that NW would indeed succeed in Plainsville.

Argument No.91

Workers in the small town of Leeville take fewer sick days than workers in the large city of Masonton, 50 miles away. Moreover, relative to population size, the diagnosis of stress-related illness is proportionally much lower in Leeville than in Masonton. According to the Leeville Chamber of Commerce, these facts can be attributed to the health benefits of the relatively relaxed pace of life in Leeville.

Write a response in which you discuss one or more alternative explanations that could rival the proposed explanation and explain how your explanation(s) can plausibly account for the facts presented in the argument.

The Leeville Chamber of Commerce defends that due only to the relaxed pace of life in Leeville, its habitants are less frequently ill than the residents of Masonton, a large city in the same region. Although this argument has a certain logic to it, it is possible to find alternative explanations for the facts presented by the author. such as the differences in levels of pollution and disease exposure as well as in the nature of the work in each city.

First, a possible explanation for the fact that workers in Leeville take fewer sick days than workers in Masonton is the level of pollution in both cities. Typically, the level of air pollution in large cities is critical to people’s health as opposed to small towns where the air is usually close to being pure. Therefore it is reasonable to assume that the people from Masonton are more subject to having respiratory diseases which might stop them from going to work.

In addition, in large cities people are usually more exposed to diseases. With a larger population, people are often in crowded closed spaces, such as subways, where viruses and bacteria can easily spread. In contrast, in small towns these situations are not so common. Assuming that this holds true for Masonton and Leeville, it can also help to explain the difference in the frequency of absences related to illness.

Finally, the amount of stress-related illness in each city may be due not to the pace of life, but to the nature of the work in each town. It is possible that Masonton has a more industrial structure of production whereas Leeville has primarily family business. In this case, people in Masonton would be expected to work longer hours and to be more productive than in Leeville. Therefore, it is natural that habitants of Masonton are usually more stressed. Evidently, the pace of life is a consequence of the working habits of the population, but is not the main cause for the facts presented in the argument.

Page 89: Y9 Argument Topics

Thus, the argument is not completely sound. Although the pace of life may help explain the differences in the levels of stress and the frequency of sick days taken by workers in both cities, a more complete analysis of these differences would have to take into account the facts previously mentioned.

Argument No.93

The following appeared in a memorandum from the manager of WWAC radio station. "WWAC must change from its current rock-music format because the number of listeners has been declining, even though the population in our listening area has been growing. The population growth has resulted mainly from people moving to our area after their retirement, and we must make listeners of these new residents. But they seem to have limited interest in music: several local stores selling recorded music have recently closed. Therefore, just changing to another kind of music is not going to increase our audience. Instead, we should adopt a news-and-talk format, a form of radio that is increasingly popular in our area." Write a response in which you discuss what questions would need to be answered in order to decide whether the recommendation and the argument on which it is based are reasonable. Be sure to explain how the answers to these questions would help to evaluate the recommendation.

In this argument, the author concludes that WWAC radio station should turn to news-and-talk format instead of continue broadcasting music program. To substantiate the conclusion, the arguer points out that the elder are the major audience and assumes that they have no interest in music according to the business condition of music record stores. The evidence provides us with some possibility but is still far from adequate. To constitute a logical argument in favor of the conclusion, the author needs to provide more compelling support. Some questions remains to be answered before coming to the conclusion.

To begin with, the author has to answer is there any other reasons account for the poor condition of music-selling area. Is it the growing population downloading music from the Internet that causes the decrease in the number of the local stores selling recorded music? Perhaps elder residents want to save money, and thus, instead of purchasing music record, they are more willing to listen on the radio.

Even if stores selling music record closed due to lacking customers, we still need to doubt what kind of music-selling stores are these closed ones belong to. The author only points out that the number of local stores selling recorded music decrease but not telling us whether they sells rock music or pop music or classic music. Perhaps these days, people enjoy listening to pop music, making stores majoring in other fields hard to survive or perhaps sellers select other industry more profitable.

What’s more, the author suggests WWAC radio station turns to news-and-talk format. Have they made any investigation concerning the recent overall environment of the news-and-talk program or their competitors? It is likely that the growing number drew many radio stations to news-and-talk area, the market is already full and audiences begin to lose interest because all the programs seem to be alike.

Page 90: Y9 Argument Topics

There is still another question need to be answered. Are there any other newly-developed music radio station? As the author mentioned, the population in listening area grows while WWAC radio station loses a great number of audience. Is it possible that people are attracted to listen to other musical program? If it is true, rather than abandon the old area and turn to news-and-talk program, WWAC radio station had better improve their program to gain back there listeners.

In summary, though the conclusion seems plausible, in fact, it lacks credibility because the evidence cited in the analysis does not lend strong support to what the arguer maintains. The author fails to establish a causal relationship between the elder lacks interest in music and the fall of the number of stores selling recorded music. To make it logically acceptable, the author would have to provide more evidence concerning the questions mentioned above.

Argument No.94

The vice president for human resources at Climpson Industries sent the following recommendation to the company's president.

"In an effort to improve our employees' productivity, we should implement electronic monitoring of employees' Internet use from their workstations. Employees who use the Internet from their workstations need to be identified and punished if we are to reduce the number of work hours spent on personal or recreational activities, such as shopping or playing games. By installing software to detect employees' Internet use on company computers, we can prevent employees from wasting time, foster a better work ethic at Climpson, and improve our overall profits."

The above article was sent as a recommendation from the vice president for human resources at Climpson Industries to the company's president. The arguer wants to recommend the implementation of electronic monitoring of employee's internet use from their work stations to improve employees' productivity. The arguer gives various arguments in favor of his recommendation. The first argument which the arguer gives in support of his recommendation is the identification and punishment that needs to be levied on employees who use internet from their work stations. This would reduce the number of working hours spent on personal or recreational activities. The second argument presented by the arguer is that by installing software that detects employee's internet use on company computers wastage of time can be prevented and it would yield better profits for the company. The arguer fails to justify his stand and therefore his recommendation falls short of relevant proofs.

The first argument presented by the arguer is absolutely baseless as the arguer mentions about punishing those employees who use internet from their work place. The arguer has not mentioned in the argument why there was internet in the work place. The internet facility must be for employees to help them to perform their work better. The arguer has also not mentioned whether the employees were using internet in their rest hours. It is very much possible that the usage must be more during rest hours. Even if the employees are using more internet then that must be to aid their work. If spending time on internet was such a waste then instead of punishing employees internet should be removed from the work place. The arguer mentions about punishing those employees who would use internet for their personal use but this step would be too harsh on the employees. This would stop employees from using internet for office use also as it is very difficult to determine how much internet is used for office and personal use.

Page 91: Y9 Argument Topics

The second argument presented by the arguer is the software that would detect an employee's internet usage and help in stopping wastage of time and thus help in company profits. The arguer fails to bring into notice that if such a facility of using internet would be debarred then out of monotony the employee would not be able to work better which would result in decreased productivity and hence less profits. The arguer also mentions about good work ethics but fails to bring into notice that employees need recreation too and if there is no other source of recreation then there would be fall in productivity.

The arguer fails to convince us in regard to his recommendation. The recommendation has a very negative approach as it can be detrimental for the personal interest of an employee. Therefore, this recommendation should not be accepted as instead of increasing the productivity for the organization this step can decrease the productivity of the organization.

Argument No.95

The following appeared in a memo from the new vice president of Sartorian, a company that manufactures men's clothing.

"Five years ago, at a time when we had difficulty obtaining reliable supplies of high-quality wool fabric, we discontinued production of our popular alpaca overcoat. Now that we have a new fabric supplier, we should resume production. Given the outcry from our customers when we discontinued this product and the fact that none of our competitors offers a comparable product, we can expect pent-up consumer demand for our alpaca coats. Due to this demand and the overall increase in clothing prices, we can predict that Sartorian's alpaca overcoats will be more profitable than ever before."

Write a response in which you discuss what questions would need to be answered in order to decide whether the prediction and the argument on which it is based are reasonable. Be sure to explain how the answers to these questions would help to evaluate the prediction.

In the passage new vice president of Satrian, a company that manufactures men's clothing suggests that alpaca overcoat production which was discontinued five years ago because of difficulty for obtaining reliable supplies of high-quality wool fabric should resume because now they have a new fabric supplier. Also this will be more profitable than ever before. However author fails to give sufficient evidences to support this conclusion. In order to make the argument more cogent author should answer some questions.

Firstly author's claim that now they have new fabric supplier, however author fails to provide enough evidence how much reliable this supplier is? What are the chances that there would be no difficulty for obtaining the fabric in the future? By providing the answer for these question author will provide evidence which will strengthen the argument.

Also as per author claim that there was outcry from the customers five years back at the time when company discontinued production of alpaca overcoat, however author should provide evidence that alpaca overcoat is in demand today also. What is the demand of alpaca overcoat at present? If vice president answer these question that there is very high demand for alpaca overcoat at present also then this will help to robust the argument.

Also vice president mentioned that none of our competitors offers a comparable product, we can expect pent-up consumer demand for our alpaca coats, but it fails to support this premises with

Page 92: Y9 Argument Topics

enough evidence. What are other products in the market? how they are not comparable alpaca overcoat? And it also mentioned that "we can expect pent up customer demand", but there is cogent evidence provided to support this claim.

In conclusion, author should provide enough evidence to support the arguments. In order to support the arguments author should answer to above questions.

Argument No.97

"Over the past ten years, there has been a 20 percent decline in the size of the average audience at Classical Shakespeare Theatre productions. In spite of increased advertising, we are attracting fewer and fewer people to our shows, causing our profits to decrease significantly. We must take action to attract new audience members. The best way to do so is by instituting a 'Shakespeare in the Park' program this summer. Two years ago the nearby Avon Repertory Company started a 'Free Plays in the Park' program, and its profits have increased 10 percent since then. Therefore, I recommend that we start a 'Shakespeare in the Park' program of our own. If we do so, I predict that our profits will also see a significant increase."

In order to attract more audiences, marketing director of Classical Shakespeare Theatre (CST) asserts to commence a new program 'Shakespeare in the Park' . His assertion is based on success of the Avon Repertory Company (ARC), which have started such a program two years ago. Definitely, to assess this recommendation CST authorities would have find answers of following questions: To how much extent the success of ARC was contingent on 'Free plays in Park' program? Secondly, will 'Shakespeare in the Park' would be in agreement of audiences' taste?

Although there was a rise of 10% in profits of ARC after commencement of 'Free plays in Park'; but, this is not enough to prove that new program was solely responsible for this raised profit. It is also possible that some other schemes of ARC would actually had been responsible for this success. For example, reduction in irritating advertisements had pleased people to attend this program. In any case, if it is found that 'Free plays in Park' was not really responsible for thriving profits of ARC, director's recommendation would be severely undermined. While in opposite case, i.e. if evidences supporting that 'Free plays in Park' was really responsible for ARC's success, director's argument would seem cogent.

Moreover, it also important to see whether the starting this new program would be effective in attracting new audiences as expected or it. AVC had implemented 'Free plays in Park' two years earlier; since from that time might had changed drastically. In two years mood of public may be reluctant for theater plays; this new generation of audience may prefer digital media over conventional theater plays. In this case again, recommendation would be weakened; and starting 'Shakespeare in the Park' can be cause for CST to be in debts. However, if director is able to prove that public is interested in this theater theme (especially, through means of some survey) , director's recommendation would be strengthened in this case. Therefore, CST authorities should consider people's choice before arriving at any conclusion.

Conclusively, before accepting or rejecting this recommendation CST authorities need to consider following things:

First, significance of 'Free plays in Park' in profits of AVC. Second, prediction of audiences' interest for 'Shakespeare in Park' program. Perhaps, if these factors are found to be in favor of

Page 93: Y9 Argument Topics

recommendation, I think there is no harm to try it. However, if these factors are found to be against the argument, definitely, CST should not risk its funds for this program.

Argument No.98,99

The following is a recommendation from the Board of Directors of Monarch Books.

"We recommend that Monarch Books open a café in its store. Monarch, having been in business at the same location for more than twenty years, has a large customer base because it is known for its wide selection of books on all subjects. Clearly, opening the café would attract more customers. Space could be made for the café by discontinuing the children's book section, which will probably become less popular given that the most recent national census indicated a significant decline in the percentage of the population under age ten. Opening a café will allow Monarch to attract more customers and better compete with Regal Books, which recently opened its own café."

Write a response in which you discuss what questions would need to be answered in order to decide whether the recommendation is likely to have the predicted result. Be sure to explain how the answers to these questions would help to evaluate the recommendation.

According to the recommendation from the business manager of Monarch Books, a cafe should be opened in its store. This would attract more customers and give a better competition to Regal Books, which has recently opened up a cafe too.

Although, more people may be attracted to the book store by the opening up of a cafe, but they all need not be the customers willing to buy the books. There may be many who would just go there to relax, rather than show a genuine interest in buying the books. This may add up to the crowd thereby upsetting the actual customers.

Nevertheless, there has been no mention of the profits earned by regal books by virtue of their cafe. They might just be attracting crowd and not many customers among them. As a result, there may not be much increase in their sales. Before copying the cafe system from regal books, the locations of the two bookshops should be given a thorough consideration. The type of crowd more likely to visit Monarch Books should also be given a thought to. There is a possibility that Regal Books is situated near some college or university campus with a lot of youngsters around. Their preference for the bookstore with the cafe can be understood, as they are generally more experimental. What may have worked for Regal Books may not work for Monarch Books.

Monarch Books is known for its selection of books on all subjects and has built a large customer following in twenty years. Closing the children's book section, for aping the new cafe system may not be taken well and bring down its popularity level. Going by the indication given by national census on the decline in popularity of children's books, is not reliable since nothing is mentioned about the year in which this census was held. It may not be recent enough to be considered. Even if there is a decrease in the demand of children's books, it is not wise to discontinue this section, as that would be adding to the further declination of children's book section. Instead, more variety on the same could be made available, not withstanding the general trend of demand and supply.

The need to bring about changes in the present pattern arises only when there is an indication of Monarch Books losing its customers and drawing less profit. However, the business manager has

Page 94: Y9 Argument Topics

not thrown any light on such issues while giving his recommendations. On one hand the manager expresses the desire to give a good competition to Regal Books, and on the other hand he suggests opening up a cafe in the book store by copying Regal Books. This may not be appreciated by the customers.

Without considering the above stated points, opening up a cafe at the cost of children's book section is not at all advisable. Continuing with the children's section will not only maintain its reputation among children but surely among their elders (parents) too. Thus, the argument given is not based on ample reasoning and is unconvincing.